wills, trusts

205
Wills Outline Complete I. The Living and the Dead: Whose Money Is It? A. Introduction Shapira v. Union National Bank —Ohio Common Pleas Court, 1974 Issue: Whether a will that requires a son, in order to recover his portion of the estate, to marry a woman with two Jewish parents within 7 years of the testator’s death violates either the federal Constitution or the public policy of the State of Ohio? NO Holding: “[P]ublic policy should not, and does not preclude the fulfillment of Dr. Shapira’s purpose, and that in accordance with the weight of authority in this country, the conditions contained in his will are reasonable restrictions upon marriage, and valid.”-7 Rule: “Basically, the right to receive property by will is a creature of the law, and is not a natural right or one guaranteed or protected by either the Ohio or the United States Constitution.”-4 “[A] testator may legally entirely disinherit his children.”-4 “The great weight of authority in the United States is that gifts conditioned upon the beneficiary’s marrying within a particular religious class or faith are reasonable.”-4 1. Note on Terminology a. “A testator is a person who has written a will. A person who dies with a duly executed will dies testate. A person who dies without a will dies intestate. An intestate decedent’s property passes to that person’s heirs, who are designated by the jurisdiction’s statute on intestate succession.”-7 b. “[W]hen testator’s will disposes of property, the persons designated to take that property are not called heirs, but devisees (or sometimes legatees). A clause directing disposition of property is called a devise, a legacy, or a bequest. Historically, the word ‘devise’ was used to describe a disposition of money, and ‘bequest’ was used to describe a disposition of personal property other than money. Today, the terms are often used interchangeably.”-7 B. Inheritance Rights More Generally 1. The term “escheats” means “passes to.”-9 2. In Hodel v. Irving, “Congress had enacted the ‘Indian Land Consolidation Act,’ which provided that undivided fractional interests in tracts of land within a reservation would escheat if the interest represented two percent or less of

Upload: tomwinfrey

Post on 08-May-2015

5.446 views

Category:

Documents


3 download

TRANSCRIPT

Page 1: Wills, Trusts

Wills Outline Complete

I. The Living and the Dead: Whose Money Is It?A. Introduction

Shapira v. Union National Bank—Ohio Common Pleas Court, 1974Issue: Whether a will that requires a son, in order to recover his portion of the estate, to marry a woman with two Jewish parents within 7 years of the testator’s death violates either the federal Constitution or the public policy of the State of Ohio? NOHolding: “[P]ublic policy should not, and does not preclude the fulfillment of Dr. Shapira’s purpose, and that in accordance with the weight of authority in this country, the conditions contained in his will are reasonable restrictions upon marriage, and valid.”-7Rule: “Basically, the right to receive property by will is a creature of the law, and is not a natural right or one guaranteed or protected by either the Ohio or the United States Constitution.”-4“[A] testator may legally entirely disinherit his children.”-4“The great weight of authority in the United States is that gifts conditioned upon the beneficiary’s marrying within a particular religious class or faith are reasonable.”-4

1. Note on Terminologya. “A testator is a person who has written a will. A person who dies with a duly

executed will dies testate. A person who dies without a will dies intestate. An intestate decedent’s property passes to that person’s heirs, who are designated by the jurisdiction’s statute on intestate succession.”-7

b. “[W]hen testator’s will disposes of property, the persons designated to take that property are not called heirs, but devisees (or sometimes legatees). A clause directing disposition of property is called a devise, a legacy, or a bequest. Historically, the word ‘devise’ was used to describe a disposition of money, and ‘bequest’ was used to describe a disposition of personal property other than money. Today, the terms are often used interchangeably.”-7

B. Inheritance Rights More Generally1. The term “escheats” means “passes to.”-92. In Hodel v. Irving, “Congress had enacted the ‘Indian Land Consolidation Act,’ which

provided that undivided fractional interests in tracts of land within a reservation would escheat if the interest represented two percent or less of the total acreage in the tract and if the interest earned its owner less than $100 in the year before it was due to escheat. [Justice O’Connor, writing for the majority, held] ‘[T]he regulation here amounts to virtually the abrogation of the right to pass on a certain type of property – the small undivided interest – to one’s heirs. . . . Since the escheatable interests are not, as the United States argues, necessarily de minimis, . . . a total abrogation of these rights cannot be upheld. . . . The difference in this case is the fact that both descent and devise are completely abolished.’”-9

3. Mark L. Ascher, Curtailing Inherited Wealtha. “My major premise is that all property owned at death, after payment of debts and

administration expenses, should be sold and the proceeds paid to the United States government.”-10

4. Adam J. Hirsch & William Wang, A Qualitative Theory of the Dead Handa. “The traditional rationales for testamentary freedom are as varied as they are

controversial. Perhaps oldest is the notion that testators have a natural right to bequeath.”-12

b. “Another argument for freedom of testation, also premised upon the goal of wealth enhancement, is that such freedom supports, as it were, a market for the provision of social services.”-12

Page 2: Wills, Trusts

c. “A secondary justification for the right of testation is that it would in practice be difficult to curtail.”-13

d. “A final justification for freedom of testation, formulated with disarming unaffectedness by Professor Simes, is simply that the power to bequeath comports with political preferences . . . .”-13

5. Miscellaneousa. “In every state, the Statute of Wills permits a decedent to write a will disposing of

his property at death.”-15b. “The Shapira case indicates that courts typically give effect to testators’ stated

preferences, even when those preferences might be offensive to others.”-16c. In Riggs v. Palmer, the court held: “It was the intention of the law-makers that the

donees in a will should have the property given to them. But it never could have been their intention that a donee who murdered the testator to make the will operative should have any benefit under it. . . . No one shall be permitted to profit by his own fraud, or to take advantage of his own wrong, or to found any claim upon his own iniquity, or to acquire property by his own crime.”-16

Ford v. Ford—Ct. of App. of Md., 1986Issue: Whether a legatee who intentionally murders the testator while insane may recover property under the will of the victim? YESHolding: “[T]he slayer’s rule is simply not applicable when the killer was not criminally responsible at the time he committed the homicide.”-21“We hold that the slayer’s rule does not operate to preclude Pearl Rose Ford from inheriting under the will of her victim.”-21Rule: “We believe that for a homicide to be ‘felonious’ in the context of the slayer’s rule, it must be a felony for which the killer is criminally responsible under Maryland’s criminal insanity test. Therefore, if a killer is ‘insane’ at the time he killed, the killing is not felonious in the contemplation of the slayer’s rule. If the killing is not felonious, even though it may be intentional, the rule does not apply.”-21

C. Slayer Statutes and the Uniform Probate Code1. “Most states have enacted statutes to deal with the slayer-heir.”-242. “Perhaps the most comprehensive slayer statute is the one included in the Uniform

Probate Code (“UPC” or “the Code”).”-243. “The Uniform Probate Code was approved by NCCUSL in 1969, and a number of states

enacted probate codes based in part or in whole on the UPC.”-244. The Joint Editorial Board’s “efforts resulted in a number of amendments, and, in 1990, in

a wholesale revision of Article II of the Code, which deals with ‘Intestacy, Wills, and Donative Transfers.’ . . . A number of state legislatures have adopted the 1990 version of the Code, a number have retained their 1969 versions, and in other states, the Code has not been adopted, but has had a discernible impact on state legislation or case law.”-25

5. UPC § 2-803a. The term “‘disposition or appointment of property’ includes a transfer of an item of

property or any other benefit to a beneficiary designated in a governing instrument. ‘Governing Instrument’ means a governing instrument executed by the decedent. ‘Revocable,’ with respect to a disposition, appointment, provision, or nomination, means one under which the decedent, at the time of or immediately before death, was alone empowered, by law or under the governing instrument, to cancel the designation in favor of the killer, whether or not the decedent was then empowered to designate himself in place of his killer, and or the decedent then had capacity to exercise the power.”-25

b. Subsection (b) provides in summary: A person who “feloniously and intentionally” kills the decedent forfeits all benefits under a will, and if the decedent dies intestate, the decedent’s intestate estate passes as if the killer disclaimed his share.-25

2

Page 3: Wills, Trusts

c. Subsection (c) provides in summary: A person who meets the above standards revokes any revocable disposition of property in a governing instrument. Such a person also severs the interests of the decedent and killer in property jointly held at the time of death.-26

d. Subsection (g) provides in summary: A conviction for intentional murder conclusively establishes that the killer meets the requirements of this section. In the absence of a conviction, the court must determine, under the preponderance of the evidence standard, whether the killer would be criminally accountable for the felonious and intentional killing.-26

II. The Role of the Lawyer, and the Lawyer-Client RelationshipA. Introduction

1. “First, estates lawyers are sometimes litigators, representing clients with claims to a decedent’s estate. . . . Second, estates lawyers are sometimes policymakers – devising and evaluating rules for transmission of wealth from one generation to the next.”-27

2. “Perhaps the most important role of the estates lawyer, however, is the role of planner. Clients come to lawyers with objectives in mind: they want to assure that their property reaches its intended beneficiaries; they want to minimize their taxes; they want to protect beneficiaries from creditor claims.”-27

3. “Litigation is the estate planner’s enemy. In most cases, if a lawyer drafts a document that results in litigation, the lawyer has not done her job – even if the document is ultimately upheld and construed as the lawyer intended it to be construed.”-28

Hotz v. Minyard—S. Ct. of S.C., 1991Issue: Whether a genuine issue of material fact exists regarding whether Dobson breached his fiduciary duty to Judy by misrepresenting her father’s will, where Dobson had been a family attorney and accountant for 20 years, and had represented Judy in the past, but was technically representing only her father with respect to the will? YESHolding: “We find the evidence indicates a factual issue whether Dobson breached a fiduciary duty to Judy when she went to his office seeking legal advice about the effect of her father’s will.”-30“Similarly, we find evidence to present a jury issue whether Law Firm should be held vicariously liable for Dobson’s conduct since Dobson was acting in his capacity as a lawyer when he met with Judy to discuss the will in January 1985.”-31Rule: “A fiduciary relationship exists when one has a special confidence in another so that the latter, in equity and good conscience, is bound to act in good faith. . . . An attorney/client relationship is by nature a fiduciary one.”-30

4. Notesa. Rule 1.6 of the MRPC provides that a lawyer shall not reveal

information relating to representation of a client unless the client consents after consultation, except for a few exceptions.

b. Rule 1.7 provides that a lawyer shall not represent a client if representation of that client will be directly adverse to another client, unless the lawyer reasonably believes that the representation will not adversely affect the interest of the other client, and each client consults after consultation.-32

Barcelo v. Elliott—S. Ct. of TX, 1996Issue: Whether “an attorney who negligently drafts a will or trust agreement owes a duty of care to persons intended to benefit under the will or trust, even though the attorney never represented the intended beneficiaries”? NO

3

Page 4: Wills, Trusts

Rule: “At common law, an attorney owes a duty of care only to his or her client, not to third parties who may have been damaged by the attorney’s negligent representation of the client.”-35“We believe the greater good is served by preserving a bright-line privity rule which denies a cause of action to all beneficiaries whom the attorney did not represent.”-37“[A]n attorney retained by a testator or settlor to draft a will or trust owes no professional duty of care to persons named as beneficiaries under the will or trust.”-38

5. Notesa. “The malpractice ‘revolution’ of the last several decades has, in the vast majority of

states, displaced the privity rule endorsed in Barcelo v. Elliott.”-406. Martin D. Begleiter, Attorney Malpractice in Estate Planning – You’ve Got to Know

When to Hold Up, Know When to Fold Upa. “If the attorney’s negligence in estate planning causes loss to a beneficiary, the

attorney shall make good that loss. That is not to say that attorneys should make good all ‘losses’ to beneficiaries in wills they drafted . . . . Many acts or omissions do not constitute negligence, and the attorney is required only to use the degree of care and skill of reasonable lawyers under comparable circumstances.”-41

III. Probate and Non-Probate TransfersA. Probate: What Is It, and Who Needs It?

1. “[D]eath is the focal point for most gratuitous transfers of wealth.”-432. “If Jane Doe has not left a will, her closest relative, or one of her closest

relatives, will generally petition for an appointment as her personal representative (traditionally called her administrator). When more than one person is interested in becoming administrator, local statutes will generally prescribe priority among the applicants. If Jane Doe has left a will naming an executor, the executor will generally petition for ‘letters testamentary,’ which entitle the executor to serve as Jane’s personal representative. The personal representative then bears responsibility for collecting all of Jane’s assets. Because the personal representative has legal authority to act on behalf of Jane’s estate, a purchaser can rely on a deed executed by the personal representative, and a bank will be protected if the bank releases funds to the personal representative.”-44

3. “If Jane has left a will, the personal representative will offer the will for probate; she will have to prove that the will was properly executed, and she will provide notice to persons who might have reason to contest the will. If probate is uncontested, or if the personal representative overcomes any challenges to the will’s probate, the personal representative will distribute Jane’s assets to the persons named in the will – after paying taxes, creditor claims, and expenses of administering the estate. . . . That, in a nutshell, is the probate process.”-44

4. “If the property is not part of Jane’s estate, the property does not pass through probate.”

B. GiftsGruen v. Gruen—Ct. of App. of N.Y., 1986

Issue: Whether “a valid inter vivos gift of a chattel may be made where the donor has reserved a life estate in the chattel and the donee never has had physical possession of it before the donor’s death”? YRule: “[T]o make a valid inter vivos gift there must exist the intent on the part of the donor to make a present transfer; delivery of the gift, either actual or constructive to the donee; and acceptance by the donee.”-46“[T]he proponent of a gift has the burden of proving each of these elements by clear and convincing evidence.”-47

4

Page 5: Wills, Trusts

“An inter vivos gift requires that the donor intend to make an irrevocable present transfer of ownership; if the intention is to make a testamentary disposition effective only after death, the gift is invalid unless made by will.”-47“[T]he requirement of delivery is not rigid or inflexible, but is to be applied in light of its purpose to avoid mistakes by donors and fraudulent claims by donees.”-48“Accordingly, what is sufficient to constitute delivery ‘must be tailored to suit the circumstances of the case.’”-48“The rule requires that the delivery necessary to consummate a gift must be as perfect as the nature of the property and the circumstances and surroundings of the parties will reasonably permit.”-49“Acceptance by the donee is essential to the validity of an inter vivos gift, but when a gift is of value to the donee . . . the law will presume an acceptance on his part.”-49

1. Notesa. “Ordinarily, gifts are irrevocable – even if made on one’s deathbed. . . . But

suppose John unexpectedly recovers. Can he take the car back? Many courts would say yes. They would reason that John’s gift was made in contemplation of death, and included an implicit condition that the care would revert to John, if he recovered from the illness. Courts have labeled gifts like this one gifts causa mortis.”-50

C. Joint Interests With Right of Survivorship1. “Lifetime gifts are the most obvious sort of non-probate transfer.”-502. “[I]f two parties hold property as joint tenants with right of survivorship (or, in the case

of married couples, as tenants by the entirety), when the first of the two dies, the decedent’s share passes automatically to the survivor – that is the meaning of the words ‘with right of survivorship.’ If more than two parties share a joint tenancy with right of survivorship, at the death of the first to die, the decedent’s share is divided equally among the surviving joint tenants.”-51

3. “Many people – especially but not exclusively married couples – hold property in joint tenancies.”-51

4. “When a joint tenant or a tenant by the entirety dies, the transfer of property that occurs is a non-probate transfer.”-51

5. “Joint tenancies and tenancies by the entirety are not restricted to real property; many people hold bank accounts and brokerage accounts as joint tenants with right of survivorship. All joint tenancies – but particularly those in bank accounts – raise questions about the intent of the joint tenants.”-51

Franklin v. Anna National Bank—App. Ct. of Ill., 1986Issue: Whether the trial court erred in finding that Mrs. Goddard was the sole owner of the funds in a savings account in which she was a joint tenant where the decedent original owner of the account asked Mrs. Goddard to be a joint owner to help him, he evidenced an intent to remove her as joint tenant, Mrs. Goddard failed to contribute or remove money from the account, and the decedent did not give Mrs. Goddard any money while he was alive? YESHolding: “The money in account No. 3816 should have been found to be the property of the estate [of the decedent].”-54Rule: “The instrument creating a joint tenancy account presumably speaks the whole truth. In order to go behind the terms of the agreement, the one claiming adversely thereto has the burden of establishing by clear and convincing evidence that a gift was not intended. . . . Each case involving a joint tenancy account must be evaluated on its own facts and circumstances. . . . The form of the agreement is not conclusive regarding the intention of the depositors between themselves. . . . Evidence of lack of donative intent must relate back to the time of creation of the joint tenancy. . . . The decision of the donor, made subsequent to the creation of the joint tenancy, that he did not want the proceeds to pass to the survivor, would not, in itself, be sufficient to sever the tenancy. . . . However, it is proper to consider events occurring after creation of the joint account in determining

5

Page 6: Wills, Trusts

whether the donor actually intended to transfer his interest in the account at his death to the surviving joint tenant.”-53

6. Notesa. Reasons for joint accounts:

(i) “Depositor – especially an elderly or incapacitated depositor – fears that he or she will be physically unable to do his own banking, and, for reasons of convenience, wants another person’s name on the account.”-55

(ii) “Depositor wants to assure that the account passes to the joint account-holder at depositor’s death without the need to go through probate; depositor does not, however, want to confer on the joint account-holder any right to the money during depositor’s lifetime.”-55

(iii) “Depositor wants to confer on the joint-account holder all of the rights associated with joint ownership – including the unlimited right to withdraw and use the money on deposit.”-55

b. “Courts have traditionally treated joint accounts as giving each party a right only to the money he or she deposited in the account. . . . [I]f either party withdraws, without permission, more than the proportion he has deposited, the other party has a claim against her. Failure to advance the claim, however, would constitute ratification of the withdrawal.”-55

c. “Courts generally enforce survivorship provisions in joint bank accounts – assuming (1) that the depositor did not revoke the survivorship provision during his lifetime, as Mr. Whitehead did in Franklin . . . ; and (2) that decedent’s estate does not introduce clear evidence that the now-deceased depositor established the joint account only for convenience.”-56

d. “Courts reasoned that a [payable on death or POD] account did not transfer any interest to the beneficiary during the depositor’s lifetime. As a result, courts treated the designation as equivalent to a will – which becomes effective only at the property owner’s death. As we shall see, however, a will is valid only if executed in accordance with testamentary formalities. Since the depositor does not typically comply with those formalities when making a POD designation, courts often held the designation invalid.”-56

7. The Uniform Probate Code, Joint Accounts, and POD Accountsa. “The Uniform Probate Code provides that depositors may open either single-party

or multiple-party accounts, and provides that either type of account may have a POD designation, an agency designation, or both.”-57 See § 6-203

8. Beyond Bank Accounts: TOD Designations in Securities Registrationa. “UPC Section 6-302 authorizes registration of securities ‘in beneficiary form’

whenever a security is owned by one individual or by two or more individuals with right of survivorship. ‘Beneficiary form’ is defined as a registration which indicates ‘the intention of the owner regarding the person who will become the owner of the security upon the death of the owner.’”-57

9. Uniform Probate Codea. UPC § 6-305 provides that “Registration in beneficiary form may be shown by the

words ‘transfer on death’ or the abbreviation ‘TOD,’ or by the words ‘pay on death’ or the abbreviation ‘POD,’ after the name of the registered owner and before the name of a beneficiary.”-57

b. UPC § 6-306 provides that “The designation of a TOD beneficiary on a registration in beneficiary form has no effect on ownership until the owner’s death. A registration of a security in beneficiary form may be canceled or changed at any time by the sole owner or all then surviving owners without the consent of the beneficiary.”-58

6

Page 7: Wills, Trusts

D. The Nonprobate Revolution: Scope and Reasons1. Langbein, The Nonprobate Revolution and the Future of the Law of Succession – “The

probate system has earned lamentable reputation for expense, delay, clumsiness, makework, and worse.”-58

2. Notesa. “Although tax considerations remain paramount in the estate plans of many wealthy

individuals, tax considerations play little role in the decision to avoid the probate system.”-60

IV. Intestate SuccessionA. Introduction and Representative Statutes

1. “[T]he property of a person who dies without a valid will – that is, a person who dies intestate – will be distributed in accordance with the intestate succession statutes in effect in the jurisdiction.”-61

2. “[A]ll intestate succession statutes provide for distribution of an intestate decedent’s assets to close family members.”-62

3. “Still another factor dictates that intestate succession statutes should distribute property to close family members: administrative convenience.”-62

4. The Importance of Intestate Successiona. “First, . . . some people execute wills that are wholly invalid, or that

dispose of only part of decedent’s property. How is the remainder of decedent’s property to be distributed? By intestate succession.”-63

b. “Second, on many occasions, a will makes reference to the ‘heirs’ of a particular person – either the heirs of the testator himself, or the heirs of some other person. Who are a person’s heirs? The people who would succeed to that person’s estate by intestate succession.”-63

c. “Third, intestate succession is often important for determining who has standing to contest a will.”-63

d. “If the contestant would be entitled to take part of the estate by intestate succession, the contestant would have standing to contest. Otherwise, the contestant would have standing only if he had been the beneficiary of a prior will.”-63

e. In re Wendell’s Will – “More than 1,600 claimants sought to challenge the will. There were numerous disputes about the relationships between Ella Wendel and the various claimants. The Surrogate adopted a procedure which required those contending to be in the nearest degree of kinship to Ella Wendel to prove their claims first, because once the claim of a close relative was established, none of the more distant claimants would have standing to contest. The will contest was eventually settled upon payment of a sum to Ella Wendell’s closest relatives.”-63

5. Fundamentals of Intestate Successiona. The Share of the Surviving Spouse

(i) “The common law developed different rules for intestate succession to real property (often called rules of ‘descent’) and intestate succession to personal property (often called rules of ‘distribution’). The surviving spouse was not an ‘heir’ of decedent’s real property; instead, the spouse had ‘dower’ or ‘curtesy’ rights – lifetime interests – in decedent’s real property. By contrast, the surviving spouse was generally

7

Page 8: Wills, Trusts

entitled to share in distribution of decedent’s personal property.”-64

(ii) “Today, most states apply the same rules to ‘descent’ of real property and ‘distribution’ of personal property.”-64

(iii) “In most states, the statutory trend has been to treat the surviving spouse more generously than at common law. Indeed, as we shall see, in some states, the surviving spouse now takes the entire estate unless the decedent – or the surviving spouse – has children from outside the marriage.”-64

b. ‘Descendants’ Take to the Exclusion of ‘Collaterals’(i) “Another nearly universal feature of intestate succession

statutes is that decedent’s direct lineal descendants take to the exclusion of collateral relatives. Collateral relatives include brothers and sisters, nieces and nephews, cousins and all other relatives who are not direct lineal descendants.”-64

(ii) “[T]he established legal meaning of ‘issue’ includes more remote lineal descendants, including grandchildren and great-grandchildren.”-65

c. Distribution Among Collaterals(i) “If one or both of decedent’s parents are still alive, they

generally take to the exclusion of other relatives.”-65(ii) “If decedent’s parents are dead, which collateral relatives take?

Two factors are important. First, who is the common ancestor of the decedent and the collateral relative? Most intestate succession statutes hold that descendants of decedent’s parents inherit to the exclusion of relatives who are descended from decedent’s grandparents, but not decedent’s parents.”-65

(iii) “Whenever a statute refers to degrees of kinship, the statute is referring to the ‘Table of Consanguinity.’ . . . To determine how many degrees of kinship separate decedent and a particular relative, we must count ‘up’ from the decedent to the common ancestor, and then ‘down’ to the relative in question.”-65

(iv) “Many states . . . have enacted what are called ‘laughing heir’ statutes – statutes which preclude inheritance by relatives too remote from decedent.”

d. Only Blood Relatives (and the Surviving Spouse) Inherit(i) “Virtually all intestate succession statutes exclude relatives by

marriage, other than decedent’s own spouse.”-66e. A Bit More Terminology

(i) At common law, “[t]he persons entitled to take decedent’s real property were referred to as his heirs at law. The persons entitled to take his personal property were referred to as his next of kin. Today, the two terms are largely synonomous.”-67

6. Massachusetts General Laws, Ch. 190 – Intestate Succession Statutea. Section 1 provides that, after all debts and charges have been paid, and

the estate has been settled, the surviving husband or wife shall recover the following real and personal property not disposed of by will – (1) If the decedent left no issue, the surviving spouse takes up to $200,000, plus one-half of the remaining property; (2) if the decedent leaves issue, the surviving spouse shall take one-half of the personal

8

Page 9: Wills, Trusts

and real property; (3) if the decedent leaves no issue and no kindred, the surviving spouse shall take the whole.

b. Section 2 provides that personal property shall be distributed in accordance with the sections distributing real property.

c. Section 3 provides that real property shall be divided: (1) in equal shares to his children and to the issue of the deceased child by right of representation, and if there is no surviving child, then to all other lineal descendants. If such descendants are in the same degree of kinship, they receive equal shares; otherwise, they take according to their right of representation; (2) if decedent leaves no issue, the property goes in equal shares to the mother and father; (3) if he leaves no issue or mother, then to his father; (4) if he leaves no issue or father, then to his mother; (5) if he leaves no issue or parents, to his brothers and sister and their issue by right of representation; and if there are no surviving siblings, to the issue of deceased brothers and sisters; (6) if he leaves no issue, parents, siblings, or issue of siblings, then to his next of kin in equal degree; (7) if he leaves no kindred, widow or husband, his estate shall escheat to the commonwealth.

d. Section 8 provides that inheritance or right of representation is the taking by the descendants of a deceased heir of the same share or right in the estate of another person as their parent would have taken if living.-69

7. Uniform Probate Codea. Section 2-101(a) provides that any part of the decedent’s property not

provided for in a will passes to the decedent’s heirs as prescribed in the Code.

b. Subsection (b) provides that a decedent by will may expressly exclude or limit an individual’s right to recover by intestate succession.

c. Section 2-102(1) provides that a surviving spouse will recover the entire estate if: there is no surviving descendant OR parent of the decedent; OR all of the decedent’s surviving descendants are also descendants of the surviving spouse and there is no descendant of the surviving spouse who survives the decedent.

d. Subsection (2) provides that the surviving spouse recovers the first $200,000 plus ¾ of any balance of the intestate estate if ONLY a parent survives the decedent.

e. Subsection (3) provides that the surviving spouse recovers the first $150,000 plus ½ of the balance if the decedent leaves children and his spouse leaves children from a previous marriage.

f. Subsection (4) provides that the surviving spouse recovers the first $100,000 plus ½ of the balance if one or more of the decedent’s surviving descendants are not descendants of the surviving spouse.

g. Section 2-103 provides that if there is no surviving spouse, the following order of individuals is established: (1) to the decedent’s descendants by representation; (2) if there is no surviving descendant, then to the decedent’s parents or parent; (3) if there is no surviving descendant or parent, then to the decedent’s siblings by representation; (4) if there is no surviving descendant, parent, or sibling, half of the estate passes to the decedent’s paternal, and half to his maternal, relatives through his grandparents.

9

Page 10: Wills, Trusts

h. Section 2-105 provides that if there is no taker, the estate passes to the state.-70

V. The Share of the Surviving SpouseA. Introduction

Estate of Goick—S. Ct. of Montana, 1996Issue: Whether all or any of the decedent’s mother, sister, and brother lack standing to challenge the appointment of his not-quite ex-wife as personal representative? Sister and brother lack standing.Whether the lower court erred in granting summary judgment against the decedent’s mother on the ground that his not-quite ex-wife was the surviving spouse for intestate succession purposes? NOWhether the lower court erred in appointing the decedent’s not-quite ex-wife as his personal representative? NOHolding: “Wanda objected to Barbara’s appointment as PR, and for that reason she has standing to appeal the appointment. Michael’s brother and sister are neither creditors nor heirs of the estate, and therefore, they have no standing to appeal her appointment as PR.”-74“We therefore conclude that the District Court did not err in holding that Barbara was the surviving spouse for purposes of intestate succession and granting summary judgment in her favor.”-76Rule: If a surviving spouse is ineligible, a creditor has priority for appointment as personal representative.-73-74Creditors may object to the appointment of a personal representative.-74An oral agreement to divorce is not binding on a judge.-75Equitable estoppel requires detrimental reliance, and the maker of the representation must have intended the injured party to rely on the representation.“If a PR has not been named under will and there are no devisees, the decedent’s surviving spouse has priority for appointment.”-76

1. Notesa. “Uniform Probate Code section 2-802 provides that a person whose marriage

has been terminated by divorce or annulment does not qualify as a surviving spouse, and also provides that a surviving spouse does not include ‘an individual who was a party to a valid proceeding concluded by an order purporting to terminate all marital property rights.’”-77

b. “Generally, a personal representative is appointed to administer the estate – to collect assets, to pay debts and expenses, and to distribute the estate.”-77

c. “When a decedent leaves a will, the will generally appoints an ‘executor’ to perform these functions. When decedent dies intestate, a court will generally appoint a personal representative, often called an ‘administrator’ to perform the same functions.”-77

d. “[I]n most jurisdictions the personal representative is entitled to compensation out of the estate. In a large estate, the compensation can be quite significant.”

VI. The Share of Lineal DescendantsA. Introduction

1. “When an intestate decedent is not survived by a spouse, decedent’s lineal descendants (assuming decedent has lineal descendants) generally succeed to the entire estate. When an intestate decedent is survived by a spouse, the lineal descendants succeed to that portion of the estate which does not pass to the surviving spouse.”-78

2. “A first principle of intestate succession is that any living descendant of the decedent cuts off the right of the descendant’s own children to inherit.”-79

3. “Suppose, however, one of the decedent’s children – let’s say D – has died before X. How should X’s estate now be distributed? Under virtually any

10

Page 11: Wills, Trusts

intestate succession scheme, D’s child, here L, would succeed to D’s share.”-79

4. “Suppose now that X survived not only D, but all of his children. How should X’s estate now be distributed? At this point, statutes differ significantly in their approach. There are two alternatives:a. Strict ‘Per Stirpes’ Distribution – “The approach taken by the English

Statute of Distributions would divide X’s estate at the generation of children – the generation closest to the decedent – whether or not any children were actually living at X’s death. Then, the descendants of X’s children would take their parents share ‘by stocks,’ or ‘per stirpes.’ . . . [T]he case for a strict per stirpes distribution rests on a number of questionable assumptions: first, that there is a ‘normal’ death order; second, that X’s children would not consume their share of X’s estate during their own lifetimes, and third, that the children would in fact leave their shares of X’s estate to their own children.”-79

b. Modern ‘Per Stirpes’ Distribution – “The vast majority of American states have abandoned the strict per stirpes distribution in favor of what has sometimes been called a ‘modern’ per stirpes distribution. These states divide decedent’s property at the closest generation to the decedent in which there is at least one descendant living.”-80

5. Questionsa. In Professor Lawrence Waggoner’s “view, all grandchildren who are

entitled to share in X’s estate (those whose parents died before X) should take the same share. The 1990 version of the Uniform Probate Code endorsed Waggoner’s approach, which is embodied in section 2-106 [of the UPC].”-81

b. “The UPC’s representation scheme – although a significant break with tradition – has gained adherents even among states which have not adopted the Code wholesale.”-82

VII. The Share of Ancestors and Collateral HeirsA. Introduction

1. “As we have seen, when an intestate decedent has surviving descendants, those descendants almost always take to the exclusion of ancestors and collateral heirs. Many decedents, however, die without surviving descendants.”-83

2. “Most intestate succession statutes give preference to decedent’s parents over collateral relatives.”-83

3. “Generally, . . . descendants of parents (brothers, sisters, and their descendants) take to the exclusion of other collateral relatives. If there are no descendants of parents, descendants of grandparents (uncles, aunts, first cousins, and their descendants) take to the exclusion of descendants of more remote ancestors.”-83

4. “[W]hen decedent is survived by descendants, issue of deceased descendants always take, no matter how many generations remote from testator. With collateral descendants, however, many statutes cut off the right to take by stocks at some point.”-84

In re Estate of Martineau—S. Ct. of N.H., 1985Issue: Whether the trial court erred in interpreting § 561:3 to mean that any collaterals up to the fourth degree of kinship may take by representation where the statute provided that no representation shall be allowed among collaterals beyond the degree of brothers’ and sisters’ grandchildren, but there are no brothers’ or sisters’ grandchildren? NO

11

Page 12: Wills, Trusts

Holding: “We conclude that all of the appellant’s arguments are unavailing, and we hold that the probate judge was correct in his decree that the first cousins were entitled to share in the paternal moiety by right of representation.”-87

5. Notesa. “The New Hampshire intestate succession statute, like many but not

all intestate succession statutes, provided that when a decedent is survived only by grandparents or descendants of grandparents, half of the estate passes to the issue of paternal grandparents and the other half passes to the issue of maternal grandparents.”-88

b. “Note that in most states, if the intestate succession statute precludes representation, it does so only with respect to more distant collateral relatives.”-89

6. Note on Escheata. “When decedent dies intestate and without heirs, the estate typically

escheats to the state, or to a governmental entity designated by statute.”-90

b. “Courts often strain to avoid escheat – despite the advent of ‘laughing heir’ statutes.”

c. In Estate of Brunel, “[t]he court held that because second cousins had been entitled to inherit before introduction of the UPC provision, they still qualified as ‘heirs.’”-90

VIII.Defining the Modern Family: Halfbloods, Adoptees, and Non-Marital Children

A. Halfbloods1. “Suppose two people share one parent, but not the other. The two are

‘halfbloods.’”-912. “The modern trend is to treat halfbloods equally with wholebloods.”-913. UPC § 2-107 provides that “Relatives of the half-blood inherit the same

share they would inherit if they were of the whole blood.”-914. “In a number of states, if decedent is survived by relatives of the half-

blood and relatives of the whole-blood in the same degree, the relatives of the half-blood take half as much as the relatives of the whole blood.”-91

5. “In still other jurisdictions, halfbloods take equally with wholebloods ‘unless the inheritance come to the intestate by descent, devise or gift of some one of his ancestors, in which case all those who are not of the blood of such ancestors must be excluded from such inheritance.”-92

B. AdoptionEstates of Donnelly—S. Ct. of Wash., 1972

Issue: May an adopted child inherit from her natural grandparents where the adoptor was the natural mother’s second husband? NOHolding: “The chain of inheritance was broken by respondent’s adoption.”-95Rule: Adopted children may not inherit from their natural parents or any other ancestors in their natural family by operation of law.

1. UPCa. Section 2-114(a) provides that a person is a child of his or her

natural parents regardless of their marital status. Subsection (b) provides that an adopted person is the child of his or her adopted parents, and not his or her natural parents; but the adoption of the child by the spouse of his or her natural parent has no effect on the child’s ability to inherit by intestate succession.-97

12

Page 13: Wills, Trusts

b. Section 2-113 provides that a person related to the decedent through two lines of relationship is entitled to only one share based on the relationship that would entitle the individual to a larger share.-97

2. Inheritance From Stepparents and Foster Parentsa. “We have seen that if a stepparent adopts her spouse’s child, the

child is treated as part of the stepparent’s natural family for intestate succession purposes. Suppose, however, the stepparent does not adopt the child, but otherwise treats the child as her own. Can the child inherit from the stepparent? The answer is generally no.”

b. “[V]irtually all states give stepchildren no right to inherit from stepparents. California is an important exception.”-98

c. California Probate Code § 6454 provides that children may inherit through a foster or stepparent if both: (1) the relationship began during the child’s minority and continued throughout the joint lifetime of the parties; and (2) it is established by clear and convincing evidence that the foster parent or stepparent would have adopted but for a legal barrier.-98

Estate of Brittin—App. Ct. of Ill., 1996Issue: Whether the natural children of an adult adoptee are descendants of the adopting parent for purposes of intestate succession? YESWhether the legislature, in enacting the statute granting an adopted child the status of a descendant of the adopting parent, intended to limit succession rights of the adoptee’s children to the natural children of a child adopted as a minor and to exclude the natural children born to the adult adoptee prior to his adoption by the adopting parent? NOHolding: “[T]he plain language of the statute indicates that the legislature intended to use the word ‘child’ in its relational sense; referring to the parent-child relationship between the adoptee and the adopting parent. The word ‘child’ as used here, cannot be interpreted fairly as meaning a minor . . . .”-Rule: The children of adult adoptees may recover under their adopting parent’s estate.

3. Equitable Adoptiona. “The equitable adoption doctrine rests on the fiction that when a

caretaker takes a child into her home, she impliedly agrees with the child’s natural parents that she will adopt the child. If the parties rely on that agreement, the child will be entitled to whatever benefits flow from status as the caretaker’s child even if the caretaker never performs the implied ‘agreement.’ Thus, the equitable adoption doctrine would permit the child to succeed to a share of the caretaker’s property by intestate succession.”-104

b. In O’Neal v. Wilkes, “Hattie sought a share of Cook’s estate as an adopted daughter – even though Cook had never taken any formal steps to adopt her. . . . A majority of the Georgia Supreme Court rejected Hattie’s intestate succession claim. Why? Because virtual adoption rests on the theory that the child’s natural parents contracted with the caretaker that the caretaker would adopt the child. For a contract – even an implied contract – to be enforceable, the parties to the contract had to have authority to make it. But who had authority to contract for Hattie’s adoption? Not her mother, who was dead. Not her paternal aunt, who, in the court’s view, had never acquired a relationship as a legal custodian. Since no one had authority to contract with Cook, Cook

13

Page 14: Wills, Trusts

had not impliedly contracted to adopt Hattie, and Hattie was not entitled to inherit.”-104

c. “In Board of Education of Montgomery County v. Browning, “[t]he court reaffirmed equitable adoption doctrine, but held that the doctrine only permits the adopted child to inherit from the adoptive parent, not through the adoptive parent, from the adoptive parent’s relatives.”-105

d. Notes(i) “By one estimate, nearly one million black children do not

live with a biological parent, and few of them have been formally adopted by their caretakers.”-105

(ii) “Even when the court recognizes equitable adoption doctrine, the adoptive ‘parent’ is not entitled to inherit from the adopted ‘child.’”-106

C. Non-Marital Children1. Trimble v. Gordon—“An Illinois statute provided that a non-marital child

could inherit from its father only if the parents had legitimated the child by marrying each other and the father had acknowledged the child during his lifetime. A girl sought to inherit from her intestate father when there had been neither marriage nor acknowledgement. The father had, however, been ordered to support his daughter after a court proceeding in which the court found him to be the girl’s father. The Illinois courts, relying on the state’s statute, held that she was not entitled to inherit from her father. The United States Supreme Court reversed, holding the statute unconstitutional. . . . [T]he Court concluded that the Illinois statute was infirm because it excluded some categories of non-marital children from inheriting even though their inheritance rights could have been recognized without jeopardizing the orderly settlement of estates.”-106

2. “Subsequently, in Lalli v. Lalli, . . . the Supreme Court upheld a New York statute which permitted a non-marital child to inherit from its father only when there had been a declaration of paternity before the father’s death. The Court held that this procedural requirement bore a substantial relation to the state interest in assuring the orderly settlement of estates.”-107

Wingate v. Estate of Ryan—S. Ct. of N.J., 1997Issue: Whether the twenty-three year from birth statute of limitations in the Parentage Act precludes the P’s claim to establish heirship? NOHolding: “We hold that N.J.S.A. 3B:23-19 controls and that the 1991 amendment to N.J.S.A. 3B:5-10 did not change the Probate Code’s statute of limitations. That statute allows an out of wedlock child of a deceased parent to file an heirship claim with the personal representative of the decedent’s estate within the time the court has deemed reasonable for the filing of claims.”-114

3. Notesa. “Historically, state statutes often required proof of paternity during the

father’s lifetime, and a number of statutes still make it more difficult for a non-marital child to establish paternity if the child made no effort to establish paternity during the father’s lifetime.”-114

b. In New York, § 4-1.2 provides that the only way to establish proof of paternity is through an order of filiation obtained during the father’s lifetime.

c. In California, the statute “precludes a child from establishing paternity after the father’s death unless paternity is presumed to exist under the Uniform Parentage Act or one of the following conditions exist: (1) A

14

Page 15: Wills, Trusts

court order was entered during the father’s lifetime declaring paternity; (2) paternity is established by clear and convincing evidence that the father has openly held out the child as his own; (3) it was impossible for the father to hold out the child as his own and paternity is established by clear and convincing evidence.”-115

d. The Uniform Parentage Act, § 4, provides that a man is presumed to be the natural father if: he and the child’s mother are or have been married and the child is born during the marriage or within 300 days after the marriage is terminated; if before the child’s birth the natural mother and man attempted to marry, etc.-116

e. The Uniform Parentage Act, § 6, provides that a child, his natural mother, or a man presumed to be his father may bring an action at any time for declaring paternity or non-paternity.-117

4. The Presumption of Legitimacya. “At common law, a man married to a child’s mother at the time of the

child’s birth was presumed to be the child’s father. The presumption could be rebutted only by proof of impotence, sterility, or lack of access during the period of gestation. In a number of states, the common law presumption has been explicitly codified. The Uniform Parentage Act codifies the presumption, and makes the presumption conclusive unless, within five years of the child’s birth, the child, the mother, or the mother’s husband brings an action to establish that the husband was not the child’s father.”-118

5. Inheritance From a Non-Marital Childa. Section 6452 of the California Probate Code provides that if a child is

born out of wedlock, neither a natural parent nor a relative of that parent inherits from or through the child on the basis of the parent and child relationship unless both: (1) the parent or relative of the parent acknowledged the child; and (2) the parent or relative contributed to the support or care of the child.-119

D. The Impact of Modern Reproductive Technology1. Introduction

a. “The Uniform Probate Code – like other state statutes – announces that every person is a ‘child of his [or her] natural parents, regardless of their marital status.’ . . . But who is the ‘natural’ parent of a child born when its mother has been artificially inseminated by sperm drawn from a sperm donor.”-121

2. Sperm Donors and Egg Donorsa. The UPA, § 5, provides that where a married woman is artificially

inseminated, her husband is treated as the natural father of the child even if the sperm is not his. However, the husband must, along with the wife, sign a consent form. Unmarried couples are not entitled to the benefit of this provision. The section goes on to provide that the donor of semen provided to a licensed physician for use in artificial insemination to any married woman other than his wife is treated as if he is not the natural father.-121

b. “The Uniform Status of Children Assisted Conception Act (1988), . . . goes beyond the Uniform Parentage Act, and provides that a donor of an egg or sperm is not the parent of a child conceived through assisted conception. The statute, however, has been enacted only in two states, Virginia and North Dakota.”-123

15

Page 16: Wills, Trusts

3. Deceased Donorsa. “The common law has always assumed . . . that no additional children

could be born to a person after the person’s death. . . . With the advent of frozen sperm, and even frozen embryos, the common law’s assumption is no longer accurate. Should intestate succession law adjust to this new reality? Probably not. If we preserved a share of a decedent’s estate for future children who might be born from decedent’s sperm or egg, we would have to deprive current heirs of the right to enjoy decedent’s estate in the here and now.”-123

Kane v. Superior Court—Ct. of App. of Cal., 1995Issue: Whether the trial court erred in issuing an order directing the administrator of the decedent’s estate to deliver 3 of the 15 vials of the decedent’s frozen semen to his mistress, where the decedent bequeathed to the mistress all 15 vials of semen, the settlement agreement entered into between the mistress and the decedent’s natural wife did not speak to the distribution of the semen, and the mistress was 40 years old? NO

IX. Simultaneous DeathA. Introduction

Estate of Villwock—Ct. of App. of Wis., 1987Issue: Whether the trial court erred in finding that Roy died before his wife where Roy’s official time of death is 11 minutes after his wife’s official time of death, but the physician who treated Roy testified that the later time was the result only of a cautionary attempt to exhaust all possibilities of a premature finding of death? NOWhether, in light of the finding that Roy died first, the Simultaneous Death Act is applicable? NOHolding: “Because the trial court found that Roy died before his wife, the provisions of the Uniform Simultaneous Death Act . . . are inapplicable.”-127

1. Uniform Probate Codea. UPC, § 2-104, provides that an individual who fails to survive the

decedent by 120 hours is deemed to have predeceased the decedent for purposes of, inter alia, intestate succession.

2. Notesa. “The current version of the Uniform Simultaneous Death Act does

not include a 120-hour survivorship requirement. In many states, however, an older version of the statute remains the law.”-131

b. “By requiring an heir to survive for 120 hours, UPC 2-104 is designed both to effectuate decedent’s intent and to avoid litigation over the precise moment of a decedent’s death.”-131

c. “The Uniform Probate Code, . . . [s]ection 2-702(a), provides that for purposes of the Code itself, a person who is not established to have survived an event by 120 hours is deemed to have predeceased the event.”-131

d. “Section 2-702(c) deals with a comparable problem: how should we decide what to do with property held by co-owners with right of survivorship when survivorship is unclear? The statute provides that, unless there is clear and convincing evidence that one co-owner survived, one-half of the property should be distributed as if each co-owner survived.”-131

e. “Section 2-702(d) provides that survival by 120 hours is not required if ‘the governing instrument contains language dealing explicitly with simultaneous deaths or deaths in a common disaster and that language is operable under the facts of the case.’”-131

16

Page 17: Wills, Trusts

X. Disclaimer (Renunciation)A. Introduction

Estate of Baird—S. Ct. of Wash., 1997Issue: Whether an anticipatory disclaimer of an expectancy interest in an estate is valid and effective where the purpose of the disclaimer is to avoid debt obligations in bankruptcy, and to transfer the expectancy interest to one’s heirs? NOHolding: “We hold that as a matter of law the instrument executed by James Baird on March 8, 1994 is invalid . . . because at that time he did not have an ‘interest,’ nor was he a ‘beneficiary.’ In sum, [the statute in question] does not authorize anticipatory disclaimers of expectation interests.”-136Rule: “The doctrine of relation ‘is a legal fiction invented to promote the ends of justice . . . . It is never allowed to defeat the collateral rights of third persons, lawfully acquired.’”-135“An intestate interest is created only upon the death of the creator of the interest, i.e., the death of the intestate.”-135

1. Notesa. “Most courts have used the ‘relation back’ theory – discussed in

Baird – to permit disclaiming beneficiaries to avoid creditor claims.”-136

b. “Two federal Courts of Appeals have held that a disclaimer executed before the disclaiming heir petitions for bankruptcy is effective to cut off any rights of the bankruptcy trustee.”-137

c. “Many public assistance programs impose eligibility limits. If an applicant for public assistance disclaims an interest in an estate, do the disclaimed assets count toward the eligibility threshold? Courts have almost invariably held that the disclaimed assets do count toward the eligibility threshold.”-137

d. “Most disclaimers today are tax driven. . . . But consider one simple [tax] advantage: by disclaiming assets, a beneficiary may be able to funnel assets to his own children without paying estate taxes. At the same time, if the beneficiary’s children are in a lower income tax bracket, the disclaiming beneficiary may also be able to reduce his family’s income tax burden.”-138

2. The Mechanics of Disclaimera. UPC § 2-801 provides that a person or the representative of a

person to whom an interest in or with respect to property or an interest therein devolves, may disclaim it in whole or in part by filing a written disclaimer, regardless of any anti-disclaimer provision in the governing instrument.

3. Note on Assignment of Expectancy Interestsa. “[A]n heir is not an heir until death, but merely an ‘heir apparent.’

Why? Because events that might occur during decedent’s lifetime could cut off the heir apparent’s interest. For instance, if decedent’s heir apparent is her brother, decedent could give birth to a child who would prevent the brother from ever becoming an heir.”-141

b. “Perhaps more important, courts were unwilling to recognize property interests in heirs apparent because decedent could always write a will cutting out the heir apparent.”-141

c. “[W]hen an heir apparent (or a will beneficiary) receives consideration for assignment of an expectancy, courts may treat the assignment as a contract to transfer decedent’s property when the

17

Page 18: Wills, Trusts

property passes to the heir, and may enforce the contract in equity.”-142

XI. AdvancementsA. Introduction

1. “At common law, a substantial gift to a child by a parent raised a presumption that the gift was an advancement of the child’s inheritance, and that the gift should be charged against the child’s intestate share. The presumption was rebuttable, but the burden of proof was on the child to demonstrate that the parent did not intend an advancement.”

2. UPC § 2-109 provides that if an individual dies intestate as to all or a portion of his estate, property that the decedent gave during his or her lifetime to an individual who is an heir, at the time of decedent’s death, is treated as an advancement against the heir’s intestate share ONLY if: (1) the decedent declared that the gift is an advancement in a contemporaneous writing; or (2) the decedent’s contemporaneous writing otherwise indicates that the gift is to be taken into account in computing the division of property.

3. “Note that the UPC (and many state statutes) reverse the common law presumption that a lifetime gift should be treated as an advancement. . . . The UPC requires a writing to establish that a gift is an advancement.”-143

4. Even if the advancement doctrine is applicable, and X is deemed to have received part or all of his estate prior to the death of the decedent, he is NOT required to give back any amount of the advancement above that amount which he would have been entitled to had he not received the advancement.-144

XII. Protection of the FamilyA. Introduction

1. “A small percentage of testators . . . would prefer to disinherit close family members, and particularly spouses.”-145

2. “Within the United States, virtually every state protects a surviving spouse against disinheritance. In most ‘common law’ states, that protection takes the form of a statutory right to a fixed share of the decedent spouse’s estate.”-145

3. “The protection against disinheritance enjoyed by the surviving spouse does not typically extend to other relatives. . . . As we shall see, however, many states protect children against inadvertent disinheritance.”-145

B. An Introduction to the Elective Share1. “In most American states, common law concepts of dower and curtesy

form the historical basis for modern law protections of the surviving spouse. Dower gave a widow a life interest in one-third of her deceased husband’s lands; curtesy gave the husband a life interest in all of his wife’s lands, but the husband acquired a curtesy interest only if children were born to the marriage.”-145

2. “[M]ost states enacted ‘elective share’ statutes which permitted the surviving spouse to elect to take a statutory percentage (generally one-third) of the decedent spouse’s probate estate even if the decedent spouse tried to limit the surviving spouse to a smaller share. The elective share generally applies to both personal property and land. In most states, the

18

Page 19: Wills, Trusts

elective share has emerged as a substitute for dower and curtesy, and those common law interests have been abolished.”-146

3. “[E]arly elective share statutes were premised on the notion that the surviving spouse – generally the wife – deserved and needed support from her husband, who was generally the family breadwinner, and who generally held title to most of the couple’s assets. Most recent elective share statutes embrace a conception of marriage as an equal partnership in which each spouse may contribute in different ways, but in which the parties agree to ‘pool their fortunes.’ Equality and sharing – not dependence and need – serve as the foundation for many of the more modern elective share statutes – including the Uniform Probate Code.”-147

4. “The most recent version of the Uniform Probate Code . . . [entitles] the surviving spouse to far less than one-third of the estate when the parties were recently married, and increasing the share of the surviving spouse with each additional year of marriage, until the share reaches a maximum of 50% of the estate when the spouses have been married for at least 15 years. . . . In addition, the Code, like some other state statutes, guarantees the surviving spouse $50,000 even if the elective share percentage would give the surviving spouse a smaller amount.”-147

C. Traditional Elective Share StatutesSullivan v. Burkin—S. Jud. Ct. of Mass., 1984

Issue: “[W]hether the assets of the decedent’s inter vivos trust are to be considered in determining the ‘portion of the estate of the deceased’ in which Mary A. Sullivan has rights”? NOHolding: “We conclude that, whether or not Ernest G. Sullivan established the inter vivos trust in order to defeat his wife’s right to take her statutory share in the assets placed in the trust and even though he had a general power of appointment over the trust assets, Mary A. Sullivan obtained no right to share in the assets of that trust when she made her election . . . .”-150Rule: “We conclude that . . . we should adhere to the principles expressed in Kerwin v. Donaghy . . . that deny the surviving spouse any claim against the assets of a valid inter vivos trust created by the deceased spouse, even where the deceased spouse alone retained substantial rights and powers under the trust instrument. For the future, however, as to any inter vivos trust created or amended after the date of this opinion, we announce that the estate of a decedent, for the purposes [of the statute in question] shall include the value of assets held in an inter vivos trust created by the deceased spouse as to which the deceased spouse alone retained the power during his or her life to direct the disposition of those trust assets for his or her benefit, as, for example, by the exercise of a power of appointment or by revocation of the trust.”-149

1. Notesa. Newman v. Dore – In this case, the wife, who was 40 years

younger than the octogenarian husband, “before his death, brought a separation action, alleging that his sexual perversions made him impossible to live with. Decedent had responded by bringing an annulment action of his own. As in Sullivan v. Burkin, decedent transferred most of his property into a revocable inter vivos trust. The Court of Appeals held the device ineffective to cut off the wife’s elective share rights.”-152

D. Modern Elective Share Statutes1. “The court in Sullivan v. Burkin suggested that the problem of

disinheritance by lifetime transfers is best dealt with by legislation. But what sort of legislation? Some states have enacted statutes that focus on the intent of the person making the gift; others focus on the character of

19

Page 20: Wills, Trusts

the transfer – was it ‘illusory.’ Still others statutes – including the pioneering New York statute and the UPC – take a bright-line approach and simply prescribe a list of inter vivos transfers whose value must be included in the estate for elective share purposes.”-152

2. Statutes that Focus on Fraudulent Intenta. Tennessee Code Ann., § 31-1-105 provides: “Any conveyances made

fraudulently to children or others, with an intent to defeat the surviving spouse of his distributive or elective share, is voidable at the election of the surviving spouse.”-153

b. Missouri Revised Statutes, § 474.150 provides: any gift, whether made while testate or intestate, in fraud of the marital rights of the surviving spouse, may be recovered from the donee and treated as a testamentary disposition and applied to the spouse’s share. Any conveyance of real estate made by a married person without the joinder or written express assent of his spouse, is deemed to be in fraud of the marital rights of the spouse.

3. The UPC’s ‘Augmented Estate’a. New York’s “legislature enacted a statute designed to protect the

surviving spouse against specified lifetime transfers, and, at the same time to reduce uncertainty about the validity of other lifetime transfers. The statute required that specified lifetime transfers be added to the value of decedent’s probate estate for purposes of computing the surviving spouse’s elective share.”-156

b. “The drafters of the 1993 version of the UPC attempted to implement the partnership theory of marriage – although some commentators contend that the Code’s principal foundation continues to be support of the surviving spouse.”-156

c. “In general outline, here is how the UPC works: in any marriage that has lasted for at least 15 years, the Code essentially treats all of the property of both spouses (and most of the property either spouse has transferred without consideration) as a pot of resources owned equally by the two spouses. Therefore, once we figure out what is in the pot at the death of the first spouse to die, if the various inter vivos and testamentary transfers have left the surviving spouse with less than half of the pot, the surviving spouse has a right to elect. Thus, if the surviving spouse holds the bulk of the pot as her own assets, she will not generally have a right to elect, even if her spouse left her none of his assets. By contrast, if the surviving spouse held no assets in her own name, she will have a right to elect unless the decedent spouse left her with at least 50% of his own assets.”-156

d. “For shorter marriages, the surviving spouse’s elective share percentage is smaller, and some of the surviving spouse’s own assets are excluded from the elective share calculation.”157

e. “The UPC, by limiting surviving spouse protection until the marriage has endured for a significant period, takes account of a common situation: a late in life marriage between spouses each of whom have had children before entering into the marriage, and each of whom would like those children to be the principal beneficiaries of their respective estates.”-157

f. A Statutory “Roadmap”

20

Page 21: Wills, Trusts

(i) “In determining whether a surviving spouse has a right to elect, one must start by taking three basic steps: (1) compute the value of the decedent spouse’s ‘augmented estate’: the pot of money the spouses have a right to share; (2) compute the surviving spouse’s elective share (the statutory percentage to which the spouse is entitled, given the length of the marriage, multiplied by the value of the augmented estate); and (3) determine whether the dispositions already made for the surviving spouse, taken in combination with the spouse’s own assets, were sufficient to eliminate any right to elect.”-157

(ii) Section 2-202(a) of the UPC provides a schedule listing the elective-share percentage based on the number of years that the couple was married. Subsection (b) provides that if the augmented estate amounts to less than $50,000, the surviving spouse is entitled to an additional elective-share amount equal to $50,000, minus the sum of the amounts described in §§ 2-207 and 2-209. Subsection (c) provides that if the right of election is exercised by the surviving spouse, the surviving spouse’s homestead allowance, exempt property, and family allowance, are not charged against it but are in addition to the elective share and supplemental elective share amounts. Subsection (d) provides that the right of a surviving spouse domiciled outside of this state to take an elective share is governed by the domicile of the decedent at the time of his death.

(iii) Section 2-203 provides: the value of the augmented estate consists of the sum of the values of all property, real or personal, that constitute the decedent’s net probate estate, the decedent’s nonprobate transfers to others, the decedent’s nonprobate transfers to the surviving spouse, and the surviving spouse’s property and nonprobate transfers to others.

(iv) Section 2-204 provides: the value of the decedent’s probate estate is to be reduced by funeral and administration expenses, homestead allowance, family allowances, exempt property, and enforceable claims.

(v) Section 2-205 provides: nonprobate transfers made by the decedent include: (1) property owned or owned in substance by the decedent immediately before his death that passed outside probate, including: property over which the decedent held a presently exercisable general power of appointment; the decedent’s fractional interest in property held in joint tenancy with a right of survivorship; ownership interest in accounts or property held in POD, TOD, or co-ownership registration with the right of survivorship; proceeds of insurance on the life of the decedent payable to anyone other than the decedent’s estate or the surviving spouse; (2) property transferred in any of the following forms during marriage: any irrevocable transfer in which the decedent retained the right to the possession or enjoyment, or income from, the property; any transfer in which the decedent created a power over income or property, exercisable by the decedent alone, or in conjunction with any

21

Page 22: Wills, Trusts

other person, to or for the benefit of the decedent; (3) property passing during marriage and during the two-year period next preceding the decedent’s death as a result of a transfer by the decedent if the transfer was one of the following types: any property passing as a result of the termination of a right or interest in, or power over, property that would have been included in the augmented estate, if the right or interest or power had not terminated until the decedent’s death; any transfer of or relating to an insurance policy on the life of the decedent if the proceeds would have been included in the augmented estate had the transfer not occurred; any transfer of property made to or for the benefit of a person other than the decedent’s surviving spouse.

(vi) Section 2-206 provides: the value of the augmented estate includes all nonprobate transfers made by the decedent to the surviving spouse at his death.

(vii) Section 2-207(a) provides: the surviving spouse’s property is included in the augmented estate. Subsection (b) provides that the property included under this section is valued at the time of the decedent’s death. Subsection (c) provides that the value of property under this section is reduced by enforceable claims against the surviving spouse.

(viii) Section 2-208(a) provides: the value of the property is excluded from the decedent’s nonprobate transfers to others to the extent that the decedent received full and adequate consideration for the transfer, or the property was transferred with the written joinder of, or written consent of, the surviving spouse. Subsection (b) provides that the value of the property includes any commuted value of any present or future interest. Subsection (c) provides that no property may be included more than once even if it would be included under two or more sections.

(ix) Section 2-209(a) provides that the following amounts are applied first to satisfy the elective share amount and to reduce or eliminate any contributions due from the decedent’s probate estate and recipients of the decedent’s nonprobate transfers to others: (1) amounts included in the augmented estate under 2-204 which pass or have passed to the surviving spouse; and (2) amounts included in the augmented estate under 2-207 up to the applicable percentage thereof. The “applicable percentage” is twice the elective share percentage set forth in the schedule in section 2-202(a) appropriate to the length of time the couple was married.

g. “Under the UPC, a surviving spouse always has a formal right to take an elective share. If, however, the decedent spouse has provided adequately for the surviving spouse, the surviving spouse will not benefit by electing, and the decedent spouse’s estate plan will remain intact.”-165

4. Note: Protecting the Spouse with Life Interests In Trust (With an Aside on Disclaimer)a. “How does the UPC treat life interests in trust?”-166

22

Page 23: Wills, Trusts

b. “[I]t is clear that the drafters intended that if decedent provided for the surviving spouse by creating life interests in trust, the surviving spouse could disclaim those interests. As a result, neither the trust principal nor the present value of the surviving spouse’s income interest would be applied against the elective share. The surviving spouse would be entitled to take the elective share outright. This result is consistent with the ‘partnership’ theory of marriage on which the Code is based.”-167

5. Note: Exercise on Behalf of a Dead or Incapacitated Surviving Spousea. “UPC section 2-212(a) does permit a conservator, guardian, or agent

under the authority of a power of attorney to exercise the power on the incapacitated spouse’s behalf. However, if the election is exercised on behalf of an incapacitated person, the elective share amounts must be placed in a custodial trust for the benefit of the incapacitated person. If the surviving spouse regains capacity, he may terminate the trust and take the property outright. If the spouse does not regain capacity, the trust property must be distributed to the beneficiaries of the decedent spouse’s estate, not the surviving spouse’s estate.”-169

6. Waiver of Elective Share RightsGeddings v. Geddings—S. Ct. of S.C., 1995

Issue: Whether the trial court erred in finding that the decedent did not fully disclose the value of his estate prior to his wife’s signing a “waiver of right to elect and of other rights,” thereby invalidating the waiver? NOHolding: “There was sufficient evidence supporting the factual findings of the probate court concurred in by the circuit court.”-171Rule: “The right of election of a surviving spouse may be waived by a written contract signed by the party waiving after fair disclosure.”-170

a. UPC § 2-213 provides that the right of election of a surviving spouse may be waived in whole or in part by the surviving spouse by a written agreement, contract, or waiver signed by the surviving spouse. But a waiver is not enforceable if the surviving spouse proves that: she did not execute the waiver voluntarily; or the waiver was unconscionable when executed, and before the execution she was not provided a fair and reasonable disclosure of the property or financial obligations of the decedent; did not voluntarily and expressly waive, in writing, any right to disclosure of the property or financial obligations of the decedent beyond the disclosure provided; and did not have, or reasonably could not have had, an adequate knowledge of the property or financial obligations of the decedent.

b. Notes(i) Rule 1.7 of the MRPC provides that a lawyer shall not

represent a client if representation of that client will be directly adverse to another client, unless: the lawyer reasonably believes the representation will not adversely affect the relationship with the other client; and each client consults after consultation.

XIII.Other Protections for the Surviving SpouseA. Homestead Allowance, Exempt Property, and Family Allowance

1. “The debtor’s clothing is often beyond the reach of creditors . . . . Many states also provide a limited exemption for the family homestead . . . . Why? For at least three reasons: (1) to preserve the debtor’s basic human

23

Page 24: Wills, Trusts

dignity; (2) to relieve taxpayers of the obligation to provide for insolvent debtors; and (3) to promote efficiency: items of personal property are likely to be worth more to the debtor than to anyone else and exempting them from creditor execution assures that they stay with the person who values them most.”-175

2. “The surviving spouse, and often minor children, remain entitled to a homestead exemption, and also remain entitled to keep some personal property away from decedent’s creditors. The form of those exemptions differs significantly from state to state.”-175

3. Section 2-402 of the UPC provides: a decedent’s surviving spouse is entitled to a $15,000 homestead allowance; or, if there is no spouse, the minor children are entitled to the $15,000, divided between the total number of such children. The homestead allowance is exempt from the claims of creditors and has priority over all other claims. The allowance is in addition to any share of the estate passing to the spouse or children by will or by intestate succession or elective share.

4. Section 2-403 of the UPC provides that in addition to the homestead allowance, the surviving spouse is entitled to up to $10,000 in household furniture, automobiles, furnishings, appliances, and personal items. If there is no spouse, the children of the decedent (not necessarily minor) are entitled to the property jointly. These rights are in addition to any share passing to the spouse or children through will, intestate succession, etc.

5. “[T]he UPC’s ‘Homestead Allowance’ is significantly different from the homestead allowance in most other state statutes. The UPC allowance applies even if decedent did not own a home; it gives the surviving spouse (or a minor child, if there is no surviving spouse) a flat sum of money, exempt from all creditor claims.”-176

6. “Probate is not instantaneous. Estate assets are generally not distributed for months after decedent’s death. For that reason, many lawyers counsel that each spouse should have title to some money so that the survivor will have instant access to funds at decedent’s death.”-177

7. In most states, “the surviving spouse and minor children are entitled to support from the estate during the period of administration.”-177

8. Section 2-404 of the UPC provides that in addition to the rights to a homestead allowance and exempt property, the decedent’s surviving spouse and minor children whom the decedent was obligated to support, and who were in fact being supported, are entitled to a reasonable allowance in money out of the estate for the period of administration, which period may continue for no longer than one year. The money is payable to the surviving spouse, if living; otherwise to the children, for their care and custody. The family allowance is exempt from and has priority over all creditor claims except the homestead allowance. The family allowance is not chargeable against any benefit or share passing to the spouse or children by will, intestate succession, etc.

B. Protection Against Inadvertent Disinheritance: The Problem of the Pre-Marital Will1. “In a few states, legislatures have assumed that testators who fail to

change their pre-marital wills do so inadvertently. Statutes in those states provide that marriage automatically revokes a pre-marital will. As a result, decedent dies intestate, and the surviving spouse takes his intestate share. . . . In other states, legislatures have declined to assume that failure

24

Page 25: Wills, Trusts

to change a pre-marital will is inadvertent, and have left the elective share provisions as the principal protection for the surviving spouse.”-178

2. “The UPC . . . assumes that a pre-marital will does reflect the testator’s intent to the extent that it benefits the testator’s issue from previous relationships. The Code presumes, however, that the testator’s intent to benefit others was negated when she remarried. That presumption is rebuttable.”-178

3. Section 2-301 of the UPC provides: if a testator’s surviving spouse married the testator AFTER the testator executed his or her will, the surviving spouse is entitled to receive, as an intestate share, no less than the value of the share of the estate he or she would have received had the testator died intestate, unless: (1) it appears from the will or other evidence that the will was made in contemplation of the testator’s marriage to the surviving spouse; (2) the will expresses the intention that it is to be in effect notwithstanding any subsequent marriage; or (2) the testator provided for the spouse by transfer outside of the will and the intent of the testator is shown to be that the transfer is made in lieu of a testamentary provision. In satisfying the share provided by this section, devises made by the will to the testator’s surviving spouse, if any, are applied first.

XIV. The Community Property SystemA. Introduction

1. “Eight states (Arizona, California, Idaho, Louisiana, Nevada, New Mexico, Texas and Washington) have community property laws. Wisconsin has enacted the Uniform Marital Property Act which contains many basic principles of Community Property.”-179

2. In community property states “there is no dower, curtesy or elective share in the probate estate.”-180

3. “The community property system . . . assumes that property acquired during marriage (other than by gift or inheritance) is the product of joint efforts of the husband and wife. Each, therefore, has a half share. Property acquired before marriage or by gift or inheritance is the separate property of the respective spouse. Management of community property varies with each state, but usually is either ‘equal’ . . . or ‘joint.’”-180

4. “Each spouse has a power of testamentary disposition only over his or her half of the community property.”-180

5. “[I]f the money came in before the marriage or by gift or inheritance, the surviving spouse gets nothing – there is no right to separate property.”-180

6. Transmutation is “the power of the husband and wife to change property of one kind into property of another kind (community/separate) by agreement or transfer.”-180

7. “[I]f a husband earns money while the couple is domiciled in a community property state and invests the money in stock in his name, and the couple moves to a common law state where the husband dies, the wife still has an equitable claim to a half interest in the stock as her community property. And, in many common law states, she will also have an elective share in the deceased husband’s half interest! UPC section 2-202 blocks such a double interest.”-180

8. “Quasi-community property” is treated as if it were community property in which each spouse has a one-half interest at the death of the owner even when it was earned in a common law state.-181

25

Page 26: Wills, Trusts

9. “The comparable concept under the UMPA section 18, is ‘deferred marital property.’”

XV. Protection of Children: Pretermitted Child StatutesA. “[O]nly Louisiana provides children with statutory protection against

intentional disinheritance. Most states, however, protect children against unintentional disinheritance. The statutes – often called pretermitted child statutes – fall into two broad categories: those which protect only children born after execution of the testator’s will, and those which protect all children who have been unintentionally disinherited.”-181

Estate of Glomset—S. Ct. of Okla., 1976Issue: Whether the trial court erred in determining that the decedent’s omission of the appellee from his will was unintentional? NOWhether extrinsic evidence is admissible to determine the testator’s intent? NOHolding: “[W]e must find that Appellee is a pretermitted heir and entitled to inherit her proportionate share of her deceased father’s estate” because her disinheritance was unintentional.-183Rule: “We have previously held that if there are no uncertainties appearing on the face of the will, extrinsic evidence is not admissible.”-182Under the statute, where a testator unintentionally disinherits one of his own children by failing to provide for them in his or her will, such children are entitled to receive the same share that they would have received if the testator had died intestate.-182Dissent: Extrinsic evidence should have been admissible to show that the appellee’s relationship with the decedent was strained. Also, the only child mentioned in the will, since the appellee is permitted to take, will receive nothing. This goes against the testator’s clearly expressed intent.-184

1. Notesa. “Massachusetts-type statutes permit the excluded child to inherit

‘unless it appears that such omission was intentional.’ In most jurisdictions (but not Oklahoma), extrinsic evidence is admissible to prove that the omission was intentional.”-184

b. “By contrast, other jurisdictions have Missouri-type statutes, which permit all children ‘not named or provided for’ in the will to take a share of the deceased parent’s estate. By definition, in states with Missouri-type statutes, extrinsic evidence is not generally available to establish testator’s intent.”-185

c. Section 2-302 of the UPC provides that: if a testator fails to provide in his or her will for any children born or adopted after the execution of the will, the omitted child receives a share in the estate as follows: (1) if the testator had no child living when he executed the will, an omitted after-born or after-adopted child receives a share of the estate equal in value to that which he or she would have received had the testator died intestate, unless the will devised all or substantially all of the estate to the other parent of the child and the other parent survives the testator; (2) if the testator had one or more children living when he executed the will, and the will devised property or an interest in property to one or more of the then living children, an omitted child is entitled to share in the testator’s estate as follows: (i) the portion of the testator’s estate in which the omitted after-born or after-adopted child is entitled to share is limited to devises made to the testator’s then living children under the will; (ii) the omitted child is entitled to receive the share that he would have received had the testator included all omitted children with the children to whom devises

26

Page 27: Wills, Trusts

were made under the will and have given an equal share of the estate to each child; (iii) the interest granted an omitted after-born or after-adopted child must be of the same character, whether equitable or legal, present or future, as that devised to the testator’s then living children under the will; (iv) in satisfying a share provided by this paragraph, devises to the testator’s children who were living when the will was executed abate ratably. In abating the devises of the then-living children, the court shall preserve the character of the testamentary plan adopted by the testator, to the extent feasible. Subsection (a) above does not apply if: (1) it appears from the will that the omission was intentional; or (2) the testator provided for the omitted children through transfer outside the will and the intent of the testator was to provide the transfer in lieu of a testamentary provision. If the testator believed that the child was dead, and for that reason excluded him from the will, that child is entitled to a share as above.

2. Heir Freight: How the Strange Life of a DHL Founder Left His Estate a Messa. The best way to prevent omitted children from taking under a

decedent’s estate is to declare in the will that such children are disinherited, even though they may exist.

3. Protection of Children Against Intentional Disinheritancea. “[M]ost American jurisdictions purport to give testators complete

freedom to disinherit their children, so long as they do so explicitly. And, of course, many testators disinherit their children in favor of their surviving spouses – especially when the spouse is also the parent of testator’s children. In that situation, courts routinely honor the disinheritance of children.”-190

XVI. WillsA. Execution of Wills

1. Will Formalities: What are They and Why Have Them?a. Section 2-502 of the UPC provides that a will must be: (1) in writing;

(2) signed by the testator or in the testator’s name by some other individual in the testator’s conscious presence and by the testator’s direction; and (3) signed by at least two individuals, each of whom signed within a reasonable time after he or she witnessed either the signing of the will as described in paragraph (2) or the testator’s acknowledgement of that signature or acknowledgement of the will.

b. “Why should the legislature be so insistent on formalities when wills are involved? The standard answer focuses on four functions allegedly served by the Statute of Wills: (1) the protective function; (2) the ritual function; (3) the evidentiary function; and (4) the channeling function.”-194

c. “The formalities of the Statute of Wills are said to protect the testator against several things including: fraud, undue influence, mistake, and fraudulent suppression of a valid will after the testator dies.”-194

d. “The formalities of the Statute of Wills do provide a ritual function. Many people understand will-signing as a serious piece of human business that one gives some consideration to, that one dresses up for and that one enters an office building to accomplish.”-194

27

Page 28: Wills, Trusts

e. “There is no quarreling with the evidentiary function of the formalities required by the Wills Act. Requiring a writing gives us a physical record of the testator’s wishes. Requiring a formal event surely creates a piece of evidence about which to testify, and requiring witnesses certainly has the effect of providing witnesses to testify about that evidence.”-195

f. “Simply put, channeling provides us with standard expressions of a testator’s intent. Channeling encompasses these ideas: (1) formalities mark an instrument with a safe conduct pass that assures a swift passage through the legal system; (2) formalities provide a known ‘channel,’ or canal, for an instrument to travel efficiently through the legal system and into the safe harbor of admission to probate; and (3) the mysteries created by the formalities channel testators to lawyers, who are trained in helping people think about their property, preparing wills, presiding over their execution, storing them, etc.”-195

Morris v. West—Ct. of App. of TX, 1982Issue: Whether the trial court erred in holding that the witnesses were not “in the presence” of the testator at the time they signed the will where they were in the same suite of offices, but were not in the same room, and could not see or be seen by the testator? NORule: “To be within the testator’s presence the attestation must occur where testator, unless blind, is able to see it from his actual position at the time, or at most, from such position as slightly altered, where he has the power readily to make the alteration without assistance.”-198

g. Questions(i) “When a will is held ineffective for failure to comply with

testamentary formalities, testator’s estate is distributed intestate unless testator had executed a prior will that has never been effectively revoked.”-199

h. Notes(i) “[S]ection 2-502 of the UPC essentially omits the presence

requirement.”(ii) “A testator must sign his will. Almost any imaginable

signature will do if it makes a visible impression on the paper and the testator intended the impression to be his signature. A testator can use her first name, initials, nickname or mark. . . . The purpose of signing is to show the will is final, to show the testator’s decision to give the will life.”-200

(iii) “Most wills acts allow the testator to sign by proxy if the proxy signs at the direction or request of the testator and in the testator’s presence.”-200

(iv) “[T]here is no need to sign at the end as long as the signature is intended to give life to the will. This is the position of the drafters of the 1990 UPC in the comment following 2-502. Signature at the end is preferable because it creates a presumption that the testator intended her signature to give life to the will. Signature in another part of the will requires the proponent to prove intent.”-201

i. The Execution Ceremony(i) “In general, . . . all execution ceremonies should include the

following steps: The will should be in final form, with pages numbered and securely fastened. It should contain an attestation clause and, perhaps, a self-proving affidavit.”

28

Page 29: Wills, Trusts

(ii) “Use the proper number of witnesses. In most states the minimum is two. In a few states the minimum is three. Use three if there is contact with a three witness state, or risk of a contest. The drafter often acts as one of the witnesses.”-202

(iii) “The witnesses should be findable, locally available and likely to survive the testator. This will make it easier to probate the will. People who take under the will should not be in the room during the ceremony, with the possible exception of the testator’s spouse. . . . If devisees are present, there is a risk of a claim of undue influence.”-202

(iv) “Testator, witnesses and attorney should be able to observe and hear each other. The attorney should ask the testator if it is her will and if she wants to sign it. When the attorney is sure the witnesses have heard the answer she should proceed to the signing. She should suggest the testator sign, and point out the place to sign, while getting the attention of the witnesses. The attorney should then ask the testator to declare to the witnesses that the instrument is her will and to request that the witnesses attest and subscribe the will. . . . As they sign, the signature of the testator should be visible to them.”-202

j. Note on Safekeeping(i) “The testator has an absolute right to her original will, and

most clients leave the lawyer’s office with the original will in hand.”-202

2. Witnessesa. “The Statute of Wills typically requires witnesses; the statute does not

expressly tell the testator, or the testator’s lawyer, whom to choose as witnesses.”-203

b. “[I]n choosing witnesses, the lawyer should keep in mind the possibility of a future contest, and should pick witnesses who will be able to offer credible testimony when the will is offered for probate. The more likely a contest, the more careful the lawyer should be in choosing witnesses. If the will is a routine one, in which testator leaves her estate to the natural objects of her bounty, the lawyer need not be as particular in choosing witnesses as when the will disinherits relatives with some expectation of an inheritance.”-204

c. NY Witness Competency Statute, § 3-3.2 provides that an attesting witness to a will to whom a beneficial disposition or appointment of property is made is a competent witness if there are at least two additional attesting witnesses not receiving a beneficial interest.

d. Notes(i) “Attest means to bear witness and competent means qualified

to testify in court concerning the material facts of execution.”-205

(ii) “An attestation clause is a paragraph of boilerplate that states the circumstances of the execution and provides a place for the witnesses to sign as attesting witnesses. It is not required in any state, but it yields a presumption of due execution and that the events described in the clause actually occurred. An attestation clause is understood to be part of the will. Every

29

Page 30: Wills, Trusts

well-drafted will contains an attestation clause or its equivalent.”-205

(iii) “When a witness signs an attestation clause she is expressing a present intent to act as a witness to the will. She is attesting. When she signs a self-proving affidavit she is swearing to the validity of an act already performed. She is testifying. To the uninitiated, the acts look the same and so does the boilerplate.”-205

(iv) “The attestation clause is the more important of the two. . . . There is nothing to probate if there is no attestation, unless there is a legal doctrine of forgiveness available.”-206

(v) “If there is an attestation clause there is a rebuttable presumption that the formalities recited in the clause were, in fact, performed. The clause can also be used to impeach a hostile witness or to inspire a wavering witness to remember.”-206

3. Salvage Doctrines: Substantial Compliance and the UPC’s Dispensing Power

In re Alleged Will of Ranney—S. Ct. of NJ, 1991Issue: “[W]hether an instrument purporting to be a last will and testament that includes the signature of two witnesses on an attached self-proving affidavit, but not on the will itself, should be admitted to probate”? YESHolding: “We are unable to conclude that a will containing the signatures of witnesses only on such an affidavit literally complies with the attestation requirements of NJSA 3B:3-2 as signatures on a will. We further hold, however, that the will may be admitted to probate if it substantially complies with these requirements.”-210 BUT,“If, after conducting a hearing in solemn form, the trial court is satisfied that the execution of the will substantially complies with the statutory requirements, it may reinstate the judgment of the surrogate admitting the will to probate.”-214Rule: In order to literally comply with the statutory requirements, every will must be signed by at least two persons who witnessed the testator signing and acknowledging the validity of his will.-210“[I]n limited circumstances a will may be probated if it substantially complies with [the] requirements.”“Generally, when strict construction would frustrate the purposes of the statute, the spirit of the law should control over its letter. Accordingly, we believe that the Legislature did not intend that a will should be denied probate because the witnesses signed in the wrong place.”-213“When formal defects occur, proponents should prove by clear and convincing evidence that the will substantially complies with statutory requirements.”-213

a. Questions(i) “The 1990 version of the UPC § 2-503 provides: ‘Although a

document . . . was not executed in compliance with § 2-502 [enumerating the wills formalities], the document . . . is treated as if it had been executed in compliance with that section if the proponent of the document . . . establishes by clear and convincing evidence that the decedent intended the document to constitute (i) the decedent’s will.’”-215

(ii) ‘Dispensing Power’ is “a judicial power to admit a document to probate even when the document lacks even the basic formalities required by UPC § 2-502.”-215

4. Holographic Wills

30

Page 31: Wills, Trusts

a. “[A] holographic will is an unwitnessed will, handwritten and signed by the testator. Holographic wills are recognized in about half the states.”-215

Zhao v. Wong—Cal. Ct. of App., 1995Issue: Whether a piece of paper that says: All Tai-Kin Wong’s (arrow) Xi Zhao. My best half, which is signed and dated by the decedent, constitutes a valid will? NOHolding: “This series of words contains no recognizable subject, no verb and no object. . . . We conclude that it simply does not contain words sufficient to constitute a valid will.”-220“The document in this case . . . simply does not contain operative words legally sufficient to accomplish a transfer of property upon death.”-221Rule: “No particular words are required to create a will. ‘Thus, a letter or other informal document will be sufficient if it discloses the necessary testamentary intention, i.e., if it appears that the decedent intended to direct the final disposition of his property after his death. The surrounding circumstances may be considered in reaching a conclusion on this issue.’”-219“[T]he document must contain some indication that it is intended to convey something upon death, or it is not a will.”-220“But when the document itself does not express an intention to convey property upon death, it does not exhibit the intent necessary for a will.”-220

b. Notes(i) “In some states a holographic will must be entirely in the

handwriting of the testator, while in others a holograph only has to be substantially in the handwriting of the testator.”-223

(ii) “A testator must sign a holographic will personally – proxy or rubber stamp signatures are not allowed. In some states the testator may sign anywhere in the will and in others she must sign her holographic will at the end. Sign anywhere is the UPC rule, and the better rule.”-224

B. What Constitutes the Will?1. Introduction

a. “The doctrine of integration permits us to treat the four pages [of a will with only one signature] as a single ‘integrated’ will.”-226

b. “The doctrine of incorporation by reference permits a court to give effect to a will which disposes of property in accordance with an unattested document, so long as the document was in existence at the time the will was executed, and so long as the document is sufficiently identified in the will.”-226

c. “The doctrine of facts of independent significance permits a court to give effect to events which would change the disposition of testator’s estate after execution of testator’s will – so long as those events have significance apart from a change in testator’s dispositive scheme.”-226

2. Integration and Incorporation by Referencea. “If the lawyer assures that all pages are consecutively numbered,

initialed by the testator, and securely affixed, courts will generally have little difficulty treating the pages, taken together, as a single, integrated will.”-226

Estate of Norton—S. Ct. of N.C., 1991Issue: Whether “the Court of Appeals erred in affirming the trial court’s decision to award judgment notwithstanding the verdict in favor of respondents because propounder failed to satisfy the legal requirements of testamentary incorporation by reference or otherwise prove that the eight-page document as a whole constituted the final testamentary instrument of Lawrence Norton”? NO

31

Page 32: Wills, Trusts

Whether there is reliable evidence that the six pages were in existence at the time of the codicil, that a “clear and distinct” reference was made to the codicil such as to provide “full assurance” that the six pages were intended to be incorporated in the testamentary wishes of decedent Lawrence Norton? YES/NOHolding: “It is apparent that sufficient evidence exists to satisfy the first Watson requirement that the extrinsic document be in existence at the time of the creation of the codicil.”-230“However, as to the second Watson requirement (that ‘the reference . . . be in terms so clear and distinct that . . . full assurance’ is provided that the six page document was intended to be incorporated, . . . we conclude that propounder’s claim fails.”-231Rule: “It is well recognized in this state that a will, properly executed, may so refer to another unattested will or other written paper or document as to incorporate the defective instrument and make the same a part of the perfect will, the conditions being that the paper referred to shall be in existence at the time the second will be executed, and the reference to it shall be in terms so clear and distinct that from a perusal of the second will, or with the aid of parol or other proper testimony, full assurance is given that the identity of the extrinsic paper has been correctly ascertained.”-230

Clark v. Greenhalge—S. Jud. Ct. of Mass., 1991Issue: Whether “a probate judge correctly concluded that specific, written bequests of personal property contained in a notebook maintained by a testatrix were incorporated by reference into the terms of the testatrix’s will”? YESRule: “A properly executed will may incorporate by reference into its provisions any ‘document or paper not so executed and witnessed, whether the paper referred to be in the form of . . . a mere list or memorandum, . . . if it was in existence at the time of the execution of the will, and is identified by clear and satisfactory proof as the paper referred to therein.’”-235“The ‘cardinal rule in the interpretation of wills, to which all rules must bend, is that the intention of the testator shall prevail, provided it is consistent with the rules of law.’”-236“The intent of the testator is ascertained through consideration of ‘the language which the testatrix has used to express her testamentary designs,’ . . . as well as the circumstances existing at the time of the execution of the will.”-236“[O]ne who seeks equity must do equity and . . . a court will not permit its equitable powers to be employed to accomplish an injustice.”-237

b. Notes(i) “The incorporation by reference doctrine is explicitly

recognized in the vast majority of states.”-238c. UPC § 2-510 provides: “A writing in existence when a will is

executed may be incorporated by reference if the language of the will manifests this intent and describes the writing sufficiently to permit its identification.”-238

d. UPC § 2-513 provides: “Whether or not the provisions relating to holographic wills apply, a will may refer to a written statement or list to dispose of items of tangible personal property not otherwise specifically disposed of by the will, other than money. To be admissible under this section as evidence of the intended disposition, the writing must be signed by the testator and must describe the items and the devisees with reasonable certainty. The writing may be referred to as one to be in existence at the testator’s death; it may be prepared before or after the execution of the will; it may be altered by the testator after its preparation; and it may be a writing that has no significance apart from its effect on the dispositions made by the will.”

3. Facts of Independent Significance and Pour-Over Trustsa. “Suppose . . . that testator’s will provides ‘I leave my 100 shares of

IBM stock to the oldest of my sisters living at the time of my death.’

32

Page 33: Wills, Trusts

After execution of the will, testator’s oldest sister dies. At testator’s death, does his second-oldest sister, his oldest surviving sister, take the stock? The answer is yes. . . . The doctrine of facts of independent significance permits a court to give effect to dispositions like the one to testator’s oldest surviving sister.”-240

b. UPC § 2-512 provides: “A will may dispose of property by reference to acts and events that have significance apart from their effect upon the disposition made by the will, whether they occur before or after the execution of the will or before or after the testator’s death. The execution or revocation of another individual’s will is such an event.”-240

c. “[T]he doctrine of facts of independent significance is relevant in two kinds of cases: first, when testator’s will makes reference to facts or events of independent significance to determine the beneficiaries of the will (as when testator’s will makes a devise to his oldest surviving sister); and second, when testator’s will makes reference to facts or events of independent significance to determine the property that an ascertained beneficiary will receive (I leave any automobile I may own at my death to my brother, Bob).”-241

d. Pour-Over Trusts(i) “Perhaps the most significant application of the doctrine of

facts of independent significance arises when testator directs that a portion of his estate be distributed to an inter vivos trust created by Testator, or by someone else. . . . If the trust were not modifiable or revocable, we could uphold testator’s disposition to the trust by using the incorporation by reference doctrine.”-241

(ii) “Nevertheless, if the trust is funded with $1,000,000 in assets, a will provision pouring part of testator’s estate into the trust would be upheld by application of the doctrine of facts of independent significance. Be sure you understand why: note that testator would not modify or revoke the trust just to change the disposition of his estate; a modification of the trust would immediately affect disposition of the $1,000,000 in the trust. The disposition would have an important present effect, not merely an effect at testator’s death.”-242Clymer v. Mayo—S. Jud. Ct. of Mass., 1985

Issue: Whether the trial court erred in holding that the decedent’s trust was validly created despite the fact that it was not funded until her death? NORule: “[W]e agree with the court’s conclusion that ‘the statute is not conditioned upon the existence of a trust but upon the existence of a trust instrument.”-244“In Second Bank-State St. Trust Co. v. Pinion, . . . we upheld a testamentary gift to a revocable and amendable inter vivos trust established by the testator before the execution of his will and which he amended after the will’s execution. Recognizing the importance of the pour-over devise in modern estate planning, we explained that such transfers do not violate the Statute of Wills despite the testator’s ability to amend the trust and thereby change the disposition of property at his death without complying with the statute’s formalities. ‘We agree with modern legal thought that a subsequent amendment is effective because of the applicability of the established equitable doctrine that subsequent acts of independent significance do not require attestation under the statute of wills.’”-243

(iii) Note

33

Page 34: Wills, Trusts

A. “Every state has enacted a version of the Uniform [Testamentary Additions to Trusts] Act, or an equivalent statute validating will provisions which ‘pour’ assets into inter vivos trusts.”-245

(iv) UPC § 2-511 provides: “(a) A will may validly devise property to the trustee of a trust established or to be established (i) during the testator’s lifetime by the testator, by the testator or some other person, or by some other person, including a funded or unfunded life insurance trust, although the settlor has reserved any or all rights of ownership of the insurance contracts, or (ii) at the testator’s death by the testator’s devise to the trustee, if the trust is identified in the testator’s will and its terms are set forth in a written instrument, other than a will, executed before, concurrently with, or after the execution of the testator’s will or in another individual’s will if that other individual has predeceased the testator, regardless of the existence, size or character of the corpus of the trust. The devise is not invalid because the trust is amendable or revocable, or because the trust was amended after the execution of the will or the testator’s death.”-245

B. Construction Problems Created by the Time Gap Between Will Execution and Death1. “Abatement rules determine the order of priority among various

devisees when the value of the estate is insufficient to satisfy all of the devises in the will. The doctrine of ademption applies when testator has devised a particular piece of property – a diamond ring, for instance – which testator disposes of after executing the will. When a devisee named in the will dies before testator’s death, the devise generally lapses unless the jurisdiction’s antilapse statute preserves the devise for the devisee’s descendants.”-246

2. “One point bears emphasis: all of these rules are rules of construction. They represent the law’s best guess about what testator would have wanted when testator did not explicitly consider the situation that actually occurred. Testator (and her lawyer) can avoid the impact of each of these doctrines by appropriate language in the will itself.”-246

3. AbatementIn re Estate of Potter—Dist. Ct. of App. of Fla., 1985

Issue: Whether the trial court erred in abating the decedent’s gift to her daughter (the house) simultaneously with the abatement of the gift to her son (an equal amount of cash) where the daughter’s gift was specific, and the son’s gift was general? YESHolding: “We hold that the trust provision in favor of Mrs. Potter’s son constituted a general legacy while the will provision in favor of Mrs. Potter’s daughter constituted a specific legacy. Thus, under the circumstances, the general legacy abated prior to the specific legacy with the result here being that Mrs. Potter’s daughter should receive the Pompano Beach residence.”-249Rule: “A specific legacy is a gift by will of property which is particularly designated and which is to be satisfied only by the receipt of the particular property described.”-247“A general legacy is one which may be satisfied out of the general assets of the testator’s estate instead of from any specific fund, thing or things. It does not consist of a gift of a particular thing or fund or part of the estate distinguished and set apart from others of its kind and subject to precise identification. A general legacy has a prerequisite of designation by quantity or amount. The gift may be either of money or other personal property.”-248

34

Page 35: Wills, Trusts

“[G]eneral devises abate before specific devises.”-248“Abatement is the reduction of a legacy or devise on account of the insufficiency of the estate of a testator to pay all his debts, all the costs of administration, and all the legacies in full.”-248

a. Notes(i) “In many cases, the need for abatement arises when

creditors assert claims against the estate. . . . Creditor claims generally enjoy priority over claims of estate beneficiaries. As a result, if decedent makes $200,000 in specific and general devises, and his net estate after payment of creditor claims is only $50,000, the devises will abate in the order specified in Potter.”-250

b. UPC § 3-902 provides: “(a) Except as provided in subsection (b) and except as provided in connection with the share of the surviving spouse who elects to take an elective share, shares of distributees abate, without reference or priority as between real and personal property, in the following order: (1) property not disposed of by will; (2) residuary devises; (3) general devises; (4) specific devises. For purposes of abatement, a general devise charged on any specific property or fund is a specific devise to the extent of the value of the property on which it is charged, and upon the failure or insufficiency of the property on which it is charged, a general devise to the extent of the failure or insufficiency. Abatement within each classification is in proportion to the amounts of property each of the beneficiaries would have received if full distribution of the property had been made in accordance with the terms of the will. (b) If the will expresses an order of abatement, or if the testamentary plan or the express or implied purpose of the devise would be defeated by the order of abatement stated in subsection (a), the shares of the distributees abate as may be found necessary to give effect to the intention of the testator. (c) If the subject of a preferred devise is sold or used incident to administration, abatement shall be achieved by appropriate adjustments in, or contribution from, other interests in the remaining assets.”-251

c. Note on Demonstrative Devises(i) The ‘demonstrative devise’ is “a devise of a

particular amount of money to be drawn from a specified fund. Suppose, for instance, testator devises to his brother ‘$20,000, and I direct that my stamp collection be sold to satisfy this devise.’ What happens if testator’s estate is insufficient to satisfy all devises, and if the stamp collection itself is only worth $10,000? The devise to the brother is a demonstrative devise. The demonstrative devise is treated as a specific devise up to the value of the stamp collection, and as a general devise for the balance. Thus, $10,000 of the devise abates with specific devises, while the remaining $10,000 abates with general devises. . . . On the other hand,

35

Page 36: Wills, Trusts

if the estate is sufficient to satisfy all devises, the brother will take all $20,000, even if the stamp collection itself is worth only $10,000.”-251

(ii) “Sophisticated lawyers rarely draft demonstrative devises.”-252

d. Note on Exoneration of Specific Devises(i) If one received a house with a mortgage under a

will, “[t]he common law rule of exoneration held that a specific devisee is entitled to have the mortgage paid at the expense of the residuary estate unless it appeared, from the will itself or surrounding circumstances, that testator intended the devisee to take subject to the mortgage.”-252

(ii) “Increasingly, however, courts and legislatures have adopted the opposite presumption – a presumption of nonexoneration. That is, the specific devisee takes subject to a mortgage lien unless testator’s contrary intent appears from the will or surrounding circumstances. Section 2-607 of the UPC . . . provides: ‘A specific devise passes subject to any mortgage interest existing at the date of death, without right of exoneration, regardless of a general directive in the will to pay debts.’”-253

e. Note on Apportionment of Taxes(i) UPC § 3-916(b) provides: “Except as provided in

subsection (i) and, unless the will otherwise provides, the tax shall be apportioned among all persons interested in the estate. The apportionment is to be made in the proportion that the value of the interest of each person interested in the estate bears to the total value of the interests of all persons interested in the estate. The values used in determining the tax are to be used for that purpose. If the decedent’s will directs a method of apportionment of tax different from the method described in this Code, the method described in the will controls.”-253

(ii) “[W]ills often include a ‘direction against apportionment’ – an instruction that tax claims should be treated as claims against the estate, reinstating the common law abatement rules for tax claims, and putting tax claims on the same footing as other claims.”-253

(iii) “Two other points are worth noting. First, the base for the federal estate tax includes not only testator’s net probate estate, but also certain lifetime transfers. In that situation, it is important for the drafter of the will (and also the lifetime instruments) to apportion tax liability in a manner that reflects testator’s intent.”-254

36

Page 37: Wills, Trusts

(iv) “Second, in addition to estate taxes, many states impose inheritance taxes. These taxes are not assessed on the size of the decedent’s estate, but on the size of the devise received by each beneficiary. . . . [E]ach recipient is primarily liable for the tax on her share. Again, testator’s lawyer may draft around the statutory solution.”-254

4. Ademptiona. “The doctrine of ademption provides, in general terms, that the

specific devisee is entitled to nothing if the specifically devised property is not in testator’s estate at testator’s death. We say that the specific devise has been adeemed by testator’s disposal of the specifically devised property.”-255

b. “The premise on which the ademption doctrine rests is this: by making a specific devise, testator expressed a desire that devisee have particular property – not the value of that property; if testator had wanted the devisee to have the value of the property, testator would have made a general devise, or a demonstrative devise, not a specific one.”-255

McGee v. McGee—S. Ct. of R.I., 1980Issue: Whether a bequest involving the distribution of all shares held in the stock of a particular company and all of the money in the name of the testator on deposit in any banking institution is a specific legacy? YESWhether the specific bequest was adeemed by virtue of the fact that $30,000 was removed from a bank account for tax purposes, and the same $30,000 was used to purchase government bonds? YESRule: “[A] testamentary gift of specific real or personal property may be adeemed – fail completely to pass as prescribed in the testator’s will – when the particular article devised or bequeathed no longer exists as part of the testator’s estate at the moment of his death because of its prior consumption, loss, destruction, substantial change, sale, or other alienation subsequent to the execution of the will. In consequence, neither the gift, its proceeds, nor similar substitute passes to the beneficiary, and this claim to the legacy is thereby barred.”-258“The principle of ademption by extinction has reference only to specific devises . . . .”-258“A specific legacy, as the term imports, is a gift or bequest of some definite specific thing, something which is capable of being designated and identified.”-258“[T]he designation and identification of the specific legacy in a testator’s will describe the gift in a manner that serves to distinguish it from all other articles of the same general nature and prevents its distribution from the general assets of the testator’s estate.”-258“[M]oney payable out of a fund – rather than out of the estate generally – described with sufficient accuracy and satisfiable only out of the payment of such fund, . . . or a bequest of money deposited in a specific bank, . . . is, as a rule, a specific legacy.”-258The “modern theory” or “in specie” test “focuses on two questions only: (1) whether the gift is a specific legacy and, if it is, (2) whether it is found in the estate at the time of the testator’s death.”-259The “form and substance rule” provides that “a substantial change in the nature or character of the subject matter of a bequest will operate as an ademption; but a merely nominal or formal change will not.”-259

c. Notes(i) “Like a majority of courts, the court in the McGee

case adopts the ‘identity’ theory of ademption rather than the ‘intent’ theory. As a result, the court did not focus on whether testator would have wanted the devise adeemed. Instead, the identity theory

37

Page 38: Wills, Trusts

requires the court to focus only on whether the devise was specific (and, of course, on whether the property was a part of the testator’s estate at testator’s death).”-261

(ii) “In many jurisdictions, if a court-appointed conservator sells or transfers specifically-devised property, the specific devise is not adeemed.”-261

(iii) “Wisconsin . . . has, by statute, adopted an ‘intent’ theory of ademption. . . . A number of statutes, including the Wisconsin statute, provide exceptions in cases where a conservator transfers property of an incompetent, and where insurance proceeds remain after adeemed property has been destroyed.”-262

d. UPC § 2-606 provides: (a) A specific devisee has a right to the specifically devised property in the testator’s estate at death and: (1) any balance of the purchase price, together with any security agreement, owing from a purchaser to the testator at death by reason of sale of the property; (2) any amount of a condemnation award for the taking of the property unpaid at death; (3) any proceeds unpaid at death on fire or casualty insurance on or other recovery for injury to the property; (4) property owned by the testator at death and acquired as a result of foreclosure, or obtained in lieu of foreclosure, of the security interest for a specifically devised obligation; (5) real or tangible personal property owned by the testator at death which the testator acquired as a replacement for specifically devised real or tangible personal property; and (6) unless the facts and circumstances indicate that ademption of the devise was intended by the testator or ademption of the devise is consistent with the testator’s manifested plan of distribution, the value of the specifically devised property to the extent of the specifically devised property is not in the testator’s estate at death and its value or its replacement is not covered by paragraphs (1) through (5). (b) If specifically devised property is sold or mortgaged by a conservator or by an agent acting within the authority of a durable power of attorney for an incapacitated principal, or if a condemnation award, insurance proceeds, or recovery for injury to the property are paid to a conservator or to an agent acting within the authority of a durable power of attorney for an incapacitated principal, the specific devisee has the right to a general pecuniary devise equal to the net sale price, the amount of the unpaid loan, the condemnation award, the insurance proceeds, or the recovery. (c) The right of a specific devisee under subsection (b) is reduced by any right the devisee has under subsection (a). (d) For the purposes of the references in subsection (b) to a conservator, subsection (b) does not apply if after the sale, mortgage, condemnation, casualty, or recovery, it was adjudicated that the testator’s incapacity ceased and the testator survived the adjudication by one year. (e) For the purposes of

38

Page 39: Wills, Trusts

the reference in subsection (b) to an agent acting within the authority of a durable power of attorney for an incapacitated principal, (i) ‘incapacitated principal’ means a principal who is an incapacitated person, (ii) no adjudication of incapacity before death is necessary, and (iii) the acts of an agent within the authority of a durable power of attorney are presumed to be for an incapacitated principal.-263

e. “UPC section 2-606(a)(6) explicitly adopts the intent theory of ademption, and creates a presumption against ademption unless the facts show that ademption was intended by testator or consistent with testator’s plan of distribution.”-263

f. Changes in Form of Securities(i) UPC § 2-605 provides: (a) If a testator executes a

will that devises securities and the testator then owned securities that meet the description in the will, the devise includes additional securities owned by the testator at death to the extent the additional securities were acquired by the testator after the will was executed as a result of the testator’s ownership of the described securities and are securities of any of the following types: (1) securities of the same organization acquired by reason of action initiated by the organization or any successor, related, or acquiring organization, excluding any acquired by exercise of purchase options; (2) securities of another organization acquired as a result of a merger, consolidation, reorganization, or other distribution by the organization or any successor, related, or acquiring organization; or (3) securities of the same organization acquired as a result of a plan of reinvestment. (b) Distributions in cash before death with respect to a described security are not part of the devise.

g. Note on Ademption by Satisfaction(i) “If testator’s will devises her ‘portrait of mom’ to

her sister, and testator then gives the portrait to her sister, the devise to the sister is ‘adeemed by satisfaction.’ The sister has received all that the testator intended to giver her; she is entitled to nothing more at testator’s death.”-265

(ii) “What if the devise is not specific? Testator’s will devises ‘to each of my three children, one third of my estate.’ One year before testator’s death, he gives $30,000 to one of his three children. Should that gift be charged against the recipient’s share of testator’s estate? In general, the answer is no – unless testator has provided, either in a will or in a writing contemporaneous with the gift – that the gift is designed to satisfy the devise in the will.”-266

(iii) UPC § 2-609 provides: (a) Property a testator gave in his or her lifetime to a person is treated as a

39

Page 40: Wills, Trusts

satisfaction of a devise in whole or in part, only if (i) the will provides for deduction of the gift, (ii) the testator declared in a contemporaneous writing that the gift is in satisfaction of the devise, or (iii)) the devisee acknowledged in writing that the gift is in satisfaction of the devise or that its value is to be deducted from the value of the devise. (b) For purposes of partial satisfaction, property given during lifetime is value as of the time the devisee came into possession or enjoyment of the property or at the testator’s death, whichever occurs first. (c) If the devisee fails to survive the testator, the gift is treated as a full or partial satisfaction of the devise, as appropriate, in applying Sections 2-603 and 2-604, unless the testator’s contemporaneous writing provides otherwise.-266

5. Lapsea. “The common law has generally assumed that when a devisee

dies before testator, the testator would not have wanted the devised property to pass to the devisee’s descendants or heirs. As a result, a devise ‘lapses’ if the devisee predeceases testator; the devisee’s heirs and descendants take nothing. In most American jurisdictions, the common law rule remains intact . . . .”-266

b. “In every American state except Louisiana, the legislature has enacted an ‘antilapse’ statute . . . . The theory behind antilapse statutes is that when a testator leaves property to a sufficiently close relative, testator would want the issue of that devisee to take the property if the devisee predeceases the testator.”-267

c. “Antilapse statutes vary significantly in operation. In a few states, the antilapse statute applies to all devisees, even to those of non-relatives. . . . In other states, the statute saves only devises to issue, or to issue and siblings. . . . In still other states, the statute saves devises to all ‘relatives’ . . . or to a class of relatives defined more broadly than issue and siblings.”-267

d. Beneficiaries of Antilapse Statutes(i) “Antilapse statutes – except in Maryland – preserve

the devise only for the issue of the deceased devisee, not for the deceased devisee’s will beneficiaries.”-267

e. Specific and General Devises – “If a specific or general devise to an individual beneficiary lapses, the devised property generally passes into the residue of testator’s estate.”-268

f. Residuary Devises – “If testator devises the residue of her estate to a single devisee, and if the devise lapses, the residue passes by intestate succession.”

(i) “Legislation in a number of states – together with the UPC – embodies the modern view that if a devise to one residuary devisee fails, that devisee’s share passes to the other residuary devisees.”

g. Representative Antilapse Statutes

40

Page 41: Wills, Trusts

(i) New Hampshire, New York, and Virginia have antilapse statutes.

h. Class Gifts(i) “At common law, if a member of the class

predeceased testator, that member’s devise lapsed, and the remaining members of the class divided the lapsed devise. Thus, if Testator’s will devises $50,000 ‘to my brother’s children,’ and one of the brother’s children predeceases Testator leaving issue, the common law rule would exclude the deceased child’s issue, and divide the property among the brother’s surviving children.”-270

(ii) “Most antilapse statutes apply to class gifts as well as gifts to individuals. . . . Even when an antilapse statute does not expressly apply to class gifts, courts typically hold that the statute was intended to apply to class gifts as well as to gifts to individuals.”

(iii) “Gifts to multi-generational classes automatically include issue of deceased class members. Drafters should use multi-generational language unless the testator wants to exclude issue of deceased class members.”-271

i. Note on Void Devises(i) “At common law, a lapsed devise was a devise to a

person who died between the time of will execution and the time of testator’s death. By contrast, if the devise was to a person who had died before the time of will execution, the devise was a ‘void’ devise.”

(ii) “Modern antilapse statutes generally apply to save both lapsed and void devises.”-272

j. Problem Areas: When Does the Will Override the Antilapse Statute?

(i) “Remember that antilapse statutes are constructional rules. By using express language in a will, testator (and testator’s lawyer) can assure that an antilapse statute will not apply.”-272

Estate of Rehwinkel—Ct. of App. of Wash., 1993Issue: Whether the trial court erred in holding that the testator intended to avoid the application of the antilapse statute by providing in her will only for “those of the following who are living at the time of my death, share and share alike”? NOHolding: “We hold that the language of distribution to those ‘who are living at the time of my death’ manifests the testator’s clear intent that the antilapse statute not apply. Accordingly, the gift to Fossum’s mother lapsed when she predeceased the testator.”-275Rule: “The paramount duty of the court is to give effect to the testator’s intent. . . . Such intention must, if possible, be ascertained from the language of the will itself and the will must be considered in its entirety and effect must be given every part thereof.”-273“A presumption arises in favor of the operation of the antilapse statute.”-273“A testator is presumed to be aware of the antilapse statute; further, it is presumed that the testator intended the statute to apply unless a contrary intent is shown. . . . The intent on the part of the testator to preclude operation of the statute must be clearly shown.”-273

41

Page 42: Wills, Trusts

“Where the testator uses words of survivorship indicating an intention that the devisee shall take the gift only if he survives the testator, the statute does not apply.”-274

k. Notes(i) UPC § 2-702(b) provides: “Except as provided in

subsection (d), for purposes of a provision of a governing instrument that relates to an individual surviving an event, including the death of another individual, an individual who is not established by clear and convincing evidence to have survived the event by 120 hours is deemed to have predeceased the event.”-276

Estate of Ulrikson—S. Ct. of Minn., 1980Issue: Whether the antilapse statute “applies where the residuary estate is given to a brother and sister, ‘and in the event that either one of them shall predecease me, then to the other surviving brother or sister,’ but in fact both brother and sister predecease the testatrix, the brother leaving issue”? YESHolding: “In this case, we hold the words of survivorship to be effective only if there are survivors. Since there are no survivors in this case, the anti-lapse statute is free to operate.”-278

l. The UPC’s Antilapse Provision(i) “In 1990, . . . the drafters of the UPC chose to

replace the relatively concise antilapse statute with a new one, renumbered as section 2-603, which is far more comprehensive – and controversial.”-279

(ii) UPC § 2-603 (b) provides: If a devisee fails to survive the testator and is a grandparent, a descendant of a grandparent, or a stepchild of either the testator or the donor of a power of appointment exercised by the testator’s will, the following apply: (1) Except as provided in paragraph (4), if the devise is not in the form of a class gift and the deceased devisee leaves surviving descendants, a substitute gift is created in the devisee’s surviving descendants. They take by representation the property to which the devisee would have been entitled had the devisee survived the testator. (2) Except as provided in paragraph (4), if the devise is in the form of a class gift, other than a devise to ‘issue,’ ‘descendants,’ ‘heirs of the body,’ ‘heirs,’ ‘next of kin,’ ‘relatives,’ or ‘family,’ or a class described by language of similar import, a substitute gift is created in the surviving descendants of any deceased devisee. The property to which the devisees would have been entitled had all of them survived the testator passes to the surviving devisees and the surviving descendants of the deceased devisees. Each surviving devisee takes the share to which he or she would have been entitled had the deceased devisees survived the testator. Each deceased devisee’s surviving descendants who are substituted for the deceased devisee take by representation the share to which the deceased devisee would have been entitled had

42

Page 43: Wills, Trusts

the deceased devisee survived the testator. For the purposes of this paragraph, ‘deceased devisee’ means a class member who failed to survive the testator and left one or more surviving descendants. (3) For the purposes of section 2-601, words of survivorship, such as in a devise to an individual ‘if he survives me,’ or in a devise to ‘my surviving children,’ are not, in the absence of additional evidence, a sufficient indication of an intent contrary to the application of this section. (4) If the will creates an alternative devise with respect to a devise for which a substantial gift is created by paragraph (1) or (2), the substitute gift is superseded by the alternative devise only if an expressly designated devisee of the alternative devise is entitled to take under the will. (5) Unless the language creating a power of appointment expressly excludes the substitution of the descendants of an appointee for the appointee, a surviving descendant of a deceased appointee of a power of appointment can be substituted for the appointee under this section, whether or not the descendant is an object of the power.-280

m. Notes(i) “UPC section 2-603 operates to save devises to two

classes of people: (1) grandparents and their descendants; and (2) stepchildren. Few other antilapse statutes protect devises to stepchildren.”-281

C. Construction Problems More Generally1. Jane Baron – Intention, Interpretation, and Stories

a. “All courts concede . . . that some will provisions are ambiguous. What techniques are acceptable to resolve those ambiguities? First, a careful reading of the will as a whole often sheds light on testator’s meaning, even though a particular devise, read in isolation, appears ambiguous. Courts are most receptive to interpretive arguments that do not require them to look beyond the ‘four corners’ of the dispositive document – here, the will. Second, in somewhat more limited circumstances, courts are willing to look at extrinsic evidence – evidence found outside the will itself – to resolve ambiguities.”-283

2. Reading the Will as a Wholea. “When a will provision creates an ambiguity obvious on the

face of the provision, courts will, as a matter of course, look to the rest of the will to resolve the ambiguity.”-284

b. Where “the devise of personal property is ambiguous on its face; in doctrinal terms, the ambiguity is ‘patent.’”-284

Matter of Marine Midland Bank, N.A.—Ct. of App. of NY, 1989Issue: Whether the term “surviving children” in the testator’s will can or should be reasonably construed to include grandchildren where one of the testator’s two children predeceased him, leaving issue? NO

43

Page 44: Wills, Trusts

Holding: “In concluding that appellants are not entitled to inherit under the relevant gift provision of this disputed will, we reaffirm two basic principles governing the adjudication of decedents’ estates by courts: our primary function is to effectuate the testator’s intent and the words used to express that intent are to be given their ordinary and natural meaning.”-284Rule: “[T]he word ‘children’ will be given its ordinary and natural meaning unless the will as a whole shows an unmistakable intent that different or remoter persons or classes should be included.”-285“[A] testator’s intent, as ascertained ‘from the words used in the will . . . according to their everyday and ordinary meaning’ reigns supreme.”-286“Courts . . . are not privileged to put contrary or even additional words into a testator’s actual written expression in order retrospectively to effectuate their own notions of ‘fair’ or ‘equitable’ distribution of estates.”-286

3. Using Extrinsic Evidencea. “Why not permit all [extrinsic] evidence to be admitted in the

hope that some of the evidence may increase the court’s understanding of the language of the will? One answer is that admitting all potentially useful evidence would be inefficient. . . . Another answer is that many kinds of extrinsic evidence are easily fabricated, especially after testator’s death.”-291

b. “Sometimes, the problem with extrinsic evidence is not fabrication, but relevance. . . . The question is testator’s intention when testator executed his will; not his intention at the time he spoke to his daughter. Testator might have changed his mind about his stepson after execution of the will, but that change in mind will not affect disposition of his estate unless testator has executed a will memorializing the change.”-291

c. Is the Will Ambiguous?(i) “Courts often say that extrinsic evidence is

admissible to shed light on testator’s intent only when the will itself is ambiguous. Mahoney v. Grainger . . . is a leading case. Testator, a 64-year old woman, consulted a lawyer about preparation of her will. She told the lawyer that her 25 first cousins were her closest relatives, and that she wanted them to share equally. She neglected to tell the lawyer that she also had an elderly aunt. The lawyer drafted the will to leave testator’s residuary estate to ‘my heirs at law living at the time of my decease, absolutely; to be divided among them equally, share and share alike. . . .’ At testator’s death, her cousins sought to introduce extrinsic evidence to establish that testator intended them – and not the aunt – to take under the will. The court rejected their effort and permitted the aunt to take.”-292

Estate of Carroll—Ct. of App. of Mo., 1989Issue: Whether the trial court erred in refusing to consider extrinsic evidence tending to show that the testator meant to provide property in his will for both his own nieces and nephews and those of his wife, where the will itself provided “my nieces and nephews that are living as of the date of my death”? NO

44

Page 45: Wills, Trusts

Rule: “’Words with a well-known technical meaning should be construed according to their technical meaning unless a contrary meaning clearly appears from the context of the will.,’ after considering the will as a whole.”-294“It is an inflexible rule of construction that the words of the will shall be interpreted in their strict and primary sense, and in no other, although they may be capable of some popular or secondary interpetation.”-294“The general rule is that the testator must be presumed to have used words in his will in their ordinary or primary sense and meaning . . . . ‘Niece’ and ‘nephew’ in their primary and ordinary sense mean the immediate descendants of the brothers and sisters of the person named.”-294

(ii) NotesA. “Some other courts have been more willing to

admit extrinsic evidence to establish that the meaning testator attached to the words he used was different from the apparently ‘clear’ meaning of those words.”-296

B. “The Restatement (Third) of Property [§ 11.1]. . . endorses this more ‘liberal’ approach . . .: ‘An ambiguity in a donative document is an uncertainty in meaning that is revealed by the text or by extrinsic evidence other than direct evidence of intention contradicting the plain meaning of the text.’ Section 11.2(a) goes on to provide that ambiguities should be ‘resolved by construing the text of the donative document in accordance with the donor’s intention, to the extent that the donor’s intention is established by a preponderance of the evidence.’ Finally, section 10.2 provides: ‘In seeking to determine the donor’s intention, all relevant evidence, whether direct or circumstantial, may be considered. Thus, the text of the document and relevant extrinsic evidence may both be considered.”-296

d. Testator’s Circumstances and Behavior(i) “Although the will appears clear on its face – the

word ‘daughter’ is not inherently ambiguous – ambiguity arises when we try to match the words in the will with the facts in the world. Lawyers often say that the will includes a ‘latent’ ambiguity. When the will includes a latent ambiguity, extrinsic evidence . . . is generally admissible.”-297

(ii) “In cases of latent ambiguity, . . . courts routinely admit evidence of testator’s circumstances or behavior to enable the court to correlate the language of the will with the facts of the world.”-297

Estate of Gibbs—S. Ct. of Wis., 1961Issue: Whether the trial court properly considered extrinsic evidence in determining that the respondent-former-employee (Robert W. Krause) of the testators should take under their will where the will explicitly mentioned Robert J. Krause of 4708 N. 46th Street, and where Robert J. Krause was not affiliated in any way with the testators? YES

45

Page 46: Wills, Trusts

Rule: “Under rules as to construction of a will, unless there is ambiguity in the text of the will read in the light of surrounding circumstances, extrinsic evidence is inadmissible for the purpose of determining intent. A latent ambiguity exists where the language of the will, though clear on its face, is susceptible of more than one meaning, when applied to the extrinsic facts to which it refers.”-299“There are two classes of latent ambiguity. One, where there are two or more persons or things exactly measuring up to the description in the will; the other where no person or thing exactly answers the declarations and description of the will, but the two or more persons or things answer the description imperfectly. Extrinsic evidence must be resorted to under these circumstances to identify which of the parties, unspecified with particularity in the will, was intended by the testator.”-299“We conclude that details of identification, particularly such matters as middle initials, street addresses, and the like, which are highly susceptible to mistake, particularly in metropolitan areas, should not be accorded such sanctity as to frustrate an otherwise clearly demonstrable intent. Where such details of identification are involved, courts should receive evidence tending to show that a mistake has been made and should disregard the details when the proof establishes to the highest degree of certainty that a mistake was, in fact, made.”-301

(iii) NotesA. Moseley v. Goodman – “Testator’s will left

$20,000 to ‘Mrs. Moseley’ and $20,000 to ‘Mrs. Moseley’s housekeeper.’ Testator was in the business of selling cigars. He bought cigars made by R.L. Moseley. At testator’s death, R.L. Moseley’s wife sought to exclude the following evidence: testator referred to one Trimble, the salesman from whom he purchased Moseley cigars, as “Moseley,’ and referred to Trimble’s wife as “Mrs. Moseley.’ When testator became ill, he rented a room at an apartment house owned by Trimble, and was attended there by Mrs. Trimble and by Mrs. Anna Lang, a housekeeper employed by Mrs. Trimble. Held, the evidence was admissible, and Mrs. Trimble and Mrs. Lang were entitled to the $20,000 devises.”-302

B. “The ambiguity in Estate of Gibbs was latent because it did not appear on the face of the document; the ambiguity became apparent only with reference to extrinsic evidence. A patent ambiguity, by contrast, is one that is apparent on the face of the will. For instance, if the will leaves testator’s house to John in one provision of the will, and leaves the same house to John in another provision, the ambiguity is patent.”-302

C. “Although courts have traditionally been ready to admit extrinsic evidence to resolve latent ambiguities, some courts have been more hesitant to admit extrinsic evidence to resolve patent ambiguities. Patent ambiguities are, in cause and effect, quite similar to mistakes, and . . . courts have historically been unwilling to admit extrinsic evidence to reform mistakes in wills.”-302

46

Page 47: Wills, Trusts

e. Testator’s Unattested StatementsBritt v. Upchurch—S. Ct. of N.C., 1990

Issue: Whether “the Court of Appeals erred (i) in holding that the affidavit of the attorney who drafted the testator’s will was admissible at trial to show the testator’s intent”? YESWhether the trial court erred in granting summary judgment for the plaintiff? NOHolding: “Since the affidavit contains the impressions of the attorney as to Mr. Hartman’s intent concerning who was to get lot 37, it is inadmissible as extrinsic evidence to explain the latent ambiguity in the will.”-307Rule: “The general rule in North Carolina is that a latent ambiguity presents a question of identity and that extrinsic evidence may be admitted to help identify the person or the thing to which the will refers. . . . This extrinsic evidence is admissible ‘to identify a person or thing mentioned therein.’ This evidence is not admissible ‘to alter or affect the construction’ of the will. . . . ‘Surrounding circumstances as well as the declarations of the testator are relevant to the inquiry.’ ‘Surrounding circumstances’ do not refer to the intent of the testator, rather these circumstances mean the ‘facts of which the testator had knowledge when she made her will.’ . . . ‘Declarations of intent by a testator . . . are not admissible to control the construction of his will or to vary, contradict, or add to its terms.”-305

(i) NotesA. “Direct statements of testator’s intention are routinely

admitted in cases of equivocation: when testator’s description of a person or property could fit one of two people or things equally well. Suppose, for instance, testator leaves $10,000 to ‘my cousin Sam.’ Testator has two cousins – one named Samuel and the other named Samantha. Testator’s statement to his wife (or his lawyer) that he had just executed a will leaving $10,000 to Samantha would be admissible to resolve the ambiguity.”-309

B. “The Restatement (Third) of Property: Donative Transfers abandons altogether the prohibition on admission of direct statements of intent.”

4. Correcting MistakesGifford v. Dyer—S. Ct. of R.I., 1852

Issue: Whether a will that does not provide for an only child is sufficient, by itself, to revoke any right to the testator’s property that the only child may have had?Holding: “It is very apparent in the present case, that the testatrix would have made the same will, had she known her son was living. She did not intend to give him anything, if living.”-310Rule: “[W]here the testator revokes a legacy, upon the mistaken supposition that the legatee is dead, and this appears on the face of the instrument of revocation, such revocation was held void.”-311

a. Notes(i) “Gifford v. Dyer and Witt v. Rosen are cases in

which testator has allegedly formed a mistaken impression about the world around her (or him). In Gifford, testator mistaken believed her son was dead; in Witt, testator mistakenly (or so the beneficiaries asserted) believed he had made gifts to the beneficiaries. Cases like these are often referred to as cases involving ‘mistake in the inducement.’ The allegation made by the excluded beneficiaries is, in effect, ‘if testator only knew the true facts, testator would have left us money.’”-311

47

Page 48: Wills, Trusts

(ii) “Courts are understandably reluctant to honor mistake in the inducement claims because every testator is under some misimpressions at the time the testator executes his or her will. . . . If we were to hold that ‘mistakes’ of this sort were sufficient to upset a duly executed will provision, many wills would be denied probate or subject to reformation.”-311

Knupp v. District of Columbia—D.C. Ct. of App., 1990Issue: Whether the trial court erred in refusing to admit extrinsic evidence showing that the lawyer drafting the will unintentionally omitted the name of a legatee? NORule: “’[I]f the language ‘upon its face and without explanation, is doubtful or meaningless’ . . . a court may examine extrinsic evidence in order to understand the will.’’”-313“Certain conditions must be present to warrant the introduction of extrinsic evidence. First, there must be some ambiguity in order for a court to consider extrinsic evidence. . . . In addition, in all cases in which such evidence is received, it can be ‘utilized only for the purpose of interpreting something actually written in the will and never to add provisions to the will.’”-313

b. Notes(i) “The Restatement (Third) of Property: Donative

Transfers . . . would disapprove the result in the Knupp case, and would permit extrinsic evidence to reform a mistaken omission in a will.”-315

D. Revocation of Wills1. “Wills are ambulatory. The testator can always revoke her will and, in

some situations, the government can revoke a will by presumption, to carry out the testator’s imagined intent. And, a revocation may be total or partial.”-315

2. “There are only three ways to revoke a will. They are: (1) revocation by a physical act to the original will – either to the paper the will is written on or to the writing on the paper; (2) revocation by subsequent instrument – either by an express clause of revocation or by inconsistent subsequent provisions; and (3) revocation by operation of law due to a change in the circumstances of the testator.”-316

3. “Problems are most likely to occur in one of three situations: if the testator walks away with the original will and does (or might have done) something physical to the will that is ambiguous; if the testator does something unequivocal to a copy of the will; and if the testator creates an ambiguous holographic writing on the original will.”-316

4. UPC § 2-507 provides: “(a) A will or any part thereof is revoked: (1) by executing a subsequent will that revokes the previous will or part expressly or by inconsistency; or (2) by performing a revocatory act on the will, if the testator performed the act with the intent and for the purpose of revoking the will or part of it another individual performed the act in the testator’s conscious presence and by the testator’s direction. For purposes of this paragraph, ‘revocatory act on the will’ includes burning, tearing, canceling, obliterating, or destroying the will or any part of it. A burning, tearing, or canceling is a ‘revocatory act on the will,’ whether or not the burn, tear, or cancellation touched any of the words on the will. (b) If a subsequent will does not expressly revoke a previous will, the execution of the subsequent will wholly revokes the previous will by inconsistency if the testator intended the

48

Page 49: Wills, Trusts

subsequent will to replace rather than supplement the previous will. (c) The testator is presumed to have intended a subsequent will to replace rather than supplement a previous will if the subsequent will makes a complete disposition of the testator’s estate. If this presumption arises and is not rebutted by clear and convincing evidence, the previous will is revoked; only the subsequent will is operative on the testator’s death. (d) The testator is presumed to have intended a subsequent will to supplement rather than replace a previous will if the subsequent will does not make a complete disposition of the testator’s estate. If this presumption arises and is not rebutted by clear and convincing evidence, the subsequent will revokes the previous will only to the extent the subsequent will is inconsistent with the previous will; each will is fully operative on the testator’s death to the extent they are not inconsistent.”-316-17

5. Revocation by Physical Acta. “In light of the formalities that attend execution of wills, few

testators are likely to burn, tear or obliterate their wills unless they intend to revoke.”-317

First Interstate Bank of Oregon v. Henson-Hammer—Ct. of App. of Or., 1989Issue: Whether the trial court erred in holding that the presumption of revocation of the testator’s will was overcome where the original will was not found upon the testator’s death, but a copy of the will did in fact exist? NORule: “When a will that was last known to be in the custody of the testator or in a location to which he had ready access cannot be found after his death, it is presumed to have been destroyed with the intention of revoking it. The strength of the presumption, however, depends on the control that the decedent possessed over the repository and whether others had access to it.”-319“[T]he strength of the presumption of revocation varies with the facts.”-319

b. Notes(i) “If the presumption is rebutted, as in the principal

case, the proponent of the will (in all likelihood the nominated executor) still has to prove the contents of the missing, but not revoked, will.”-321

(ii) “[S]tates may require specific forms of proof of the will’s contents or establish special procedures for proving the will. Statutes establishing these special procedures are called ‘lost and destroyed will statutes.’”-321

(iii) “At present, the rule is clear – acts done to a copy of the will are irrelevant.”-321

(iv) “If, for some reason, a testator signs two original wills with the same exact terms, the two wills are ‘duplicate originals.’”-321

(v) “If the ‘lost’ duplicate original was the only one in the testator’s custody, courts generally indulge in the presumption that testator destroyed the duplicate original, and thereby revoked the will.”

(vi) “Virtually all states recognize proxy revocation by physical act. The testator must intend a revocation and the act must be done by the proxy in the testator’s presence and by her direction.”-322

49

Page 50: Wills, Trusts

(vii) “Most states allow partial revocation of a will by physical act, on the basis that the local statute expressly or impliedly sanctions such revocations.”-322

6. Revocation by Subsequent Written Instrumenta. “Lawyers rarely supervise revocation by physical acts.

Typically, a lawyer supervises revocation of an old will by preparing a new will which expresses the client’s current testamentary intent.”-323

Wolfe’s Will—S. Ct. of N.C., 1923Issue: Whether a will that explicitly bequeaths “all my effects to my brothers and sisters” supersedes a separate will, drafted approximately 15 days earlier, that provides for the bequest of a specific parcel of land to another person? MaybeRule: “A will may be revoked by a subsequent instrument executed solely for that purpose, or by a subsequent will containing a revoking clause or provision inconsistent with those of the previous will, or by any of the other methods prescribed by law; but the mere fact that a second will was made, although it purports to be the last, does not create a presumption that it revokes or is inconsistent with one of prior date.”-324“[I]n the construction of wills the primary purpose is to ascertain and give effect to the testator’s intention as expressed in the words employed, and if the language is free from ambiguity and doubt, and expresses plainly, clearly, and distinctly the maker’s intention, there is no occasion to resort to other means of interpretation.”-325

b. Questions(i) “[T]he decedent’s last will and testament may . . .

be the combination of two or more instruments. The combination often consists of a will plus one or more codicils.”-326

7. Revocation by Operation of Lawa. UPC § 1-201(19) provides: “’Governing instrument’ means a

deed, will, trust, insurance or annuity policy, account with POD designation, security registered in beneficiary form (TOD), pension, profit-sharing, retirement, or similar benefit plan, instrument creating or exercising a power of appointment or a power of attorney, or a dispositive, appointive, or nominative instrument of any similar type.”-327

b. UPC § 2-804(b) provides: “Except as provided by the express terms of a governing instrument, a court order, or a contract relating to the division of the marital estate made between the divorced individuals before or after the marriage, divorce, or annulment, the divorce or annulment of a marriage: (1) revokes any revocable (i) disposition . . . of property made by a divorced individual to his [or her] former spouse in a governing instrument and any . . . disposition . . . in a governing instrument to a relative of the divorced individual’s former spouse, . . . (iii) nomination in a governing instrument, nominating a divorced individual’s former spouse or a relative of the divorced individual’s former spouse to serve in any fiduciary or representative capacity, including a personal representative, executor, trustee, conservator, agent, or guardian.”-328

c. Notes

50

Page 51: Wills, Trusts

(i) “In some states the marriage revokes the entire will. Then Fred would get his intestate share and the balance of Sally’s estate would also pass by intestacy. In other states, Fred would be a ‘pretermitted’ or ‘omitted’ spouse. Then the will would not be revoked. Rather, Fred would get his omitted spouse’s share and the balance of Sally’s probate estate would pass under her will.”

8. Revival and Dependent Relative Revocationa. “Revocation of testator’s last will does not reinstate a prior will

. . . [b]ecause reinstatement of the prior will would require testamentary formalities, and the act of burning, tearing, or mutilating the 1996 will was not accompanied by those formalities. Thus, if, in 1999, testator were to sign the [explicitly revoked] 1990 will again, in front of witnesses, testator would have effectively reinstate the will. Similarly, if testator were to execute a codicil to the previously revoked 1990 will, testator would have ‘republished’ the original will by codicil, and the 1990 will would be reinstated.”-330

b. UPC § 2-509 provides: (a) If a subsequent will that wholly revoked a previous will is thereafter revoked by a revocatory act under Section 2-507(a)(2), the previous will remains revoked unless it is revived. The previous will is revived if it is evident from the circumstances of the revocation of the subsequent will or from the testator’s contemporary or subsequent declarations that the testator intended the previous will to take effect as executed. (b) If a subsequent will that partly revoked a previous will is thereafter revoked by a revocatory act under Section 2-507(a)(2), a revoked part of the previous will is revived unless it is evident from the circumstances of the revocation of the subsequent will or from the testator’s contemporary or subsequent declarations that the testator did not intend the revoked part to take effect as executed. (c) If a subsequent will that revoked a previous will in whole or in part is thereafter revoked by another, later, will, the previous will remains revoked in whole or in part, unless it or its revoked part is revived. The previous will or its revoked part is revived to the extent it appears from the terms of the later will that the testator intended the previous will to take effect.-331

c. “Suppose testator executes a will, and then a codicil. If testator subsequently revokes the codicil, has testator also revoked the will? The standard answer is ‘no.’ Revocation of the codicil leaves the will, without the revoked codicil, intact.”-332

Carter v. First United Methodist Church of Albany—S. Ct. of Ga., 1980Issue: Whether the trial court erred in admitting the testator’s will to probate, where part of the will was crossed out, as if revoked, and the will was accompanied by a handwritten document, not signed or witnessed, purporting to be the testator’s last will and testament? NORule: “As a general rule, the burden is on a person attacking a paper offered for probate as a will to sustain the grounds of his attack. But by express provision of our statute, where a will has been canceled or obliterated in a material part, a presumption of revocation arises, and the burden is on the

51

Page 52: Wills, Trusts

propounder to show that no revocation was intended. Where the paper is found among the testator’s effects, there is also a presumption that he made the cancellations or obliterations.”-333 McIntyre v. McIntyre.“The doctrine of dependent relative revocation (conditional revocation) has been stated by this court as follows: ‘It is a doctrine of presumed intention, and has grown up as a result of an effort which courts always make to arrive at the real intention of the testator. . . . The mere fact that the testator intended to make a new will, or made one which failed of effect, will not alone, in every case, prevent a cancellation or obliteration of a will from operating as a revocation. If it is clear that the cancellation and the making of the new will were parts of one scheme, and the revocation of the old will was so related to the making of the new as to be dependent upon it, then if the new will be not made, or if made is invalid, the old will, though canceled, should be given effect, if its contents can be ascertained in any legal way. But if the old will is once revoked – if the act of revocation is completed – as if the will be totally destroyed by burning and the like, or if any other act is done which evidences an unmistakable intention to revoke, though the will be not totally destroyed, the fact that the testator intended to make a new will, or made one which cannot take effect, counts for nothing.”-334“[I]f the purpose of the doctrine is to effect testator’s intent, there is no point in distinguishing between revocation by physical act and by subsequent instrument.”-335

d. Notes(i) “The premise underlying DRR is that testator’s

revocation of his will was based on a mistake – often a mistake about the effect of the revocation. Since testator was laboring under the burden of a mistake when he ‘revoked’ his will, courts ignore the ‘revocation,’ and treat the revoked will as if it were still in effect.”-336

(ii) “Mrs. Tipton’s revocation was conditioned on her belief that her new disposition would be given effect. Since that disposition would not be given effect, the condition did not occur, and the revocation was therefore ineffective.”-336

E. Limits on the Power to Revoke: Joint Wills and Will Contracts1. “In general, a testator is free to revoke her will at any time before her

death.”-337Estate of Wiggins—S. Ct. of N.Y., App. Div., 1974

Issue: Whether the execution of two codicils after the death of a spouse with whom the decedent drafted a joint will providing for the distribution of all of the couples’ property may alter the terms of the original joint will? NO Holding: “We hold that the parties entered into a binding contract for the distribution of their collective assets, that Mrs. Wiggins’ promised performance became irrevocable upon the death of her husband in 1948 and that the entire estate of the two parties as it existed at the time of Mrs. Wiggins’ death is to be distributed in accordance with the will, the bequests contained in the later codicils notwithstanding.”-Rule: “A joint will is an instrument that when viewed solely as a will is revocable at pleasure, but when considered as a contract, if supported by adequate consideration, may be enforceable in equity.”-340

2. Notesa. “When parties execute joint wills, courts often find contract

obligations even when the language imposing binding obligations is less than crystal clear.”-

b. “Suppose a husband and wife execute separate wills. Can those wills embody contract obligations? The answer is yes. If the parties are sufficiently explicit, courts will enforce contracts created by separate wills.”-343

52

Page 53: Wills, Trusts

c. “Difficult questions arise when two spouses execute separate wills with the same dispositive provisions – often called ‘reciprocal’ or ‘mutual’ wills. In general, ‘the existence of reciprocal wills is not sufficient by itself to imply an agreement between the testators that the wills were to be irrevocable.’ Walleri v. Gorman.”-343

d. UPC § 2-514 provides: “A contract to make a will or devise, or not to revoke a will or devise, or to die intestate, if executed after the effective date of this Article, may be established only by (i) provisions of a will stating material provisions of the contract, (ii) an express reference in a will to a contract and extrinsic evidence proving the terms of the contract, or (iii) a writing signed by the decedent evidencing the contract. The execution of a joint will or mutual wills does not create a presumption of a contract not to revoke the will or wills.”-343

Shimp v. Huff—Ct. of App. of Md., 1989Issue: Whether “Lester Shimp’s second wife, upon his death, is entitled to receive an elective share and a family allowance under Maryland Code . . . § 3-203 and § 3-201 of the Estates and Trusts Article when Lester had previously contracted, by virtue of a joint will with his first wife, to will his entire estate to others”? YESHolding: “[W]e find that respondents’ rights under the contract were limited by the possibility that the survivor might remarry and that the subsequent spouse might elect against the will. Consequently, we conclude that their claims under the contract are subordinate to Lisa Mae’s superior right to receive her elective share.”-350Rule: “In those cases where the decedent has executed a will conforming to the contract, the claimants cannot seek specific performance, and courts, therefore, do not use equitable powers in resolving these cases. Instead, courts have analyzed the conflicting claims by characterizing the competing claimants as either creditors or legatees and evaluating their claims under the applicable priority rules. In a number of cases involving divorce settlements, the courts have found that where the decedent executes a will, which conforms to the terms of a contract, the beneficiaries take as legatees under the will and not as contract creditors. . . . Consequently, because the applicable statutes give a higher priority to a surviving spouse’s elective share than to testamentary bequests, the courts upheld the surviving spouse’s claim over the claims of the contract beneficiaries.”-348“[W]e find the question of priorities between a surviving spouse and beneficiaries under a contract to make a will should be resolved based upon the public policy which surrounds the marriage relationship and which underlies the elective share statute.”-349“In executing a will a testator is presumed to know that a spouse might renounce the will, thus extinguishing or reducing legacies contained in the will, and if the testator does not provide for this contingency then the beneficiaries under the will might lose the property left them.”-350

3. Notesa. Hughes v. Frank—“Alden, a difficult woman afflicted with

Parkinson’s disease, became dependent on Hughes, a caretaker. Alden never paid Hughes a salary, but, according to Alden’s bookkeeper, she promised that Hughes would receive everything upon Alden’s death. Alden’s will, however, left Hughes only $125,000, while Alden’s estate was worth $2.1 million. The court concluded that Alden and Hughes had entered into a binding oral contract, and that Hughes was entitled to specific performance.”-352

XVII. Contesting the WillA. Introduction

53

Page 54: Wills, Trusts

1. “Even if the will complies with statutory formalities, contestants – those seeking to establish that the will is invalid – may argue that the will, or a part of the will, should not be admitted to probate because the testator lacked testamentary capacity, or because the will’s dispositive provisions were unduly influenced by one of the will beneficiaries.”

B. Testamentary Capacity1. “It is hornbook law that testator can only execute a valid will if testator

has ‘testamentary capacity.’ The UPC provides that ‘an individual 18 or more years of age who is of sound mind may make a will.’”-357

Barnes v. Marshall—S. Ct. of Mo., 1971Issue: Whether the trial court erred in holding that the testator was not of sound mind when he executed two codicils shortly before his death where he acted crazy, and several witnesses attested to that fact? NHolding: “It is sufficient to say that we think testator’s stated views on government, religion, morals, and finances go beyond the classification of peculiarities and eccentricities and are sufficient evidence from which a jury could reasonably find he was of unsound mind.”-362Rule: “The rule is well settled that, ordinarily, before a lay witness will be permitted to give his opinion that a person is of unsound mind, he must first detail the facts upon which he bases such opinion, but if he expresses an opinion that such person is of sound mind, he is not required to detail the facts upon which he founds his opinion. The reason for the rule is obvious. An opinion that a person is of unsound mind is based upon abnormal or unnatural acts and conduct of such person, while an opinion of soundness of mind is founded upon the absence of such acts and conduct.”-363

2. Notesa. “One justification for invalidating the will of a person who lacks

capacity is that the will does not reflect testator’s ‘true’ desires – the desires testator would have had were it not for testator’s ‘illness.’”-366

b. “Another justification for requiring capacity rests on the notion that family members are entitled to an inheritance – either because they have relied on the inheritance during their lifetime, or because they have developed expectations about inheritance which ought not to be disappointed unless testator has a rational basis for disinheriting them.”-366

c. “In many capacity cases, the proponent of the will tries to rebut evidence of incapacity by showing that even if testator often lacked capacity, testator executed the will during a ‘lucid interval’ – a period during which testator did have sufficient understanding to make a will.”-366

d. In Daley v. Boroughs, “[t]he trial court had upheld testator’s will against a capacity challenge. The Arkansas Supreme Court . . . upheld the will, relying on testimony of lay witnesses, combined with the possibility of a lucid interval.”-367

e. “Typically, testator’s heirs, and any beneficiaries who would take larger amounts under prior wills, have standing to contest. That is, people with a financial interest in invalidating the last will have standing to contest the will.”-367

f. “Jurisdictions divide about the right of an administrator or executor under a prior will to contest a subsequently executed will. Trustees under prior wills generally can contest because they have a financial interest as title holders. Creditors generally have not standing to contest, because the contest will not affect the size of the estate. Courts divide, however, about whether the creditors of an heir, or of the beneficiary of a prior will, have standing to contest.”-367

54

Page 55: Wills, Trusts

g. “Testamentary capacity is a legal concept, not a medical one. Typically, the articulated standard for capacity is relatively low.”-367

h. “Indeed, in a number of cases, courts have upheld testator’s capacity to make a will even after a guardian or conservator has been appointed to manage testator’s affairs.”

i. “As Barnes v. Marshall indicates, courts treat testamentary capacity as a question of fact. In most states, juries decide capacity questions. Juries are notoriously sympathetic to disinherited relatives.”-368

j. “When a jury finds that testator lacks capacity, what recourse is available to the proponents of the will? . . . [T]hey will have to argue that there was no evidence to support the jury verdict, or in other words, that ‘reasonable jurors could not have differed’ as to testator’s capacity.”-368

k. “In some states, proponents may be able to argue that the jury’s verdict was ‘against the great weight and preponderance’ of the evidence. . . . Alternatively, they can take two other avenues pursued by the lawyers in Barnes v. Marshall: they can argue that the trial court improperly admitted certain testimony, or they can argue that the trial court issued improper instructions to the jury. Note that these approaches may only entitle proponents to a new trial, not to a judgment admitting the will to probate – although appellate courts often find a way to direct a verdict for proponents. Note also that some states – California is a notable example – ban juries from will contest cases.”-368

l. “Courts differ about who bears the burden of proof on the capacity issue. Some states hold that the proponents of the will bear the burden of proving capacity. . . . Others hold that the contestants bear the burden of proving incapacity.”-368

m. “UPC § 3-407 provides that contestants have both the initial burden of proof and the ultimate burden of persuasion on issues of incapacity.”-369

n. “Incapacity cases frequently turn on testimony of witnesses who have no legal or medical expertise. . . . Such evidence is routinely admitted – subject to the limitation discussed in Barnes: the witness may not state a conclusion about testator’s capacity without relating the facts on which the conclusion is based.”-369

o. “In broad terms, medical testimony is of two types: testimony by physicians who examined decedent, and testimony by physicians who didn’t.”-369

p. “When a physician has examined testator close to the time of will execution, the physician’s testimony is always relevant and admissible.”-369

In re Hargrove’s Will—N.Y. S. Ct., App. Div., 1941Issue: Whether the decedent possessed a rational basis for the exclusion of his two children from his will, or, alternatively, was of unsound mind by virtue of an insane delusion that his children were not in fact his? Rational BasisHolding: “When consideration is given . . . to his unfortunate experiences in connection with his divorce, the fact of his belief in his wife’s infidelity certified to by her under oath in her divorce proceeding and confirmed by the formal affidavit left by him with his executor, it cannot be said that his belief on this subject was entirely without reason although possibly mistaken.”-372Rule: “The law is that assuming the decedent was mistaken in his belief that he was not the father of the children of his divorced wife, that fact would not necessarily establish testator’s incapacity. . . .

55

Page 56: Wills, Trusts

‘Delusion is insanity where one persistently believes supposed facts which have no real existence except in his perverted imagination, and against all evidence and probability, and conducts himself, however logically, upon the assumption of their existence. But, if there are facts, however insufficient they may in reality be, from which a prejudiced or narrow or a bigoted mind might derive a particular idea or belief, it cannot be said that the mind is diseased in that respect. The belief may be illogical or preposterous, but it is not, therefore, evidence of insanity in the person. Persons do not always reason logically or correctly from facts, and that may be because of their prejudices, or of the perversity or peculiar construction of their minds. Wills, however, do not depend for their validity upon the testator’s ability to reason logically, or upon his freedom from prejudice.’”-372

3. Notesa. Insane delusion: “If a person persistently believes supposed facts,

which have no real existence except in his perverted imagination, and against all evidence and probability, and conducts himself, however logically, upon the assumption of their existence, he is, so far as they are concerned, under a morbid delusion; and delusion in that sense is insanity. Such a person is essentially mad or insane on those subjects, though on other subjects he may reason, act and speak like a sensible man.”-373

b. “[C]ourts are reluctant to reform wills based on alleged mistakes of fact – largely because all of us are laboring under mistakes of fact every day.”-373

Gonsalves v. Superior Court—Ct. of App. of Ca., 1993Issue: “Does an attorney who fails to investigate a client’s testamentary capacity incur liability to a person disinherited by the will drawn by the attorney”? NORule: “[I]ntended beneficiaries of a will who lose their testamentary rights because of failure of the attorney who drew the will to properly fulfill that attorney’s obligations under his or her contract with the testator may recover damages as third party beneficiaries and also on a tort liability theory for breach of duty owed directly to the beneficiaries.”-377“[A]ttorney liability should not be extended in favor of those who are disinherited by the will of a testator without testamentary capacity.”-377“The primary duty of the attorney is to the client and is fulfilled if the attorney, convinced of testamentary capacity by his or her own observations and experience with the client, draws the will as requested. We conclude that an attorney who fails to investigate the testamentary capacity of his or her client is not liable in tort to a former beneficiary disinherited by the will drawn by the attorney. The attorney obviously is not liable in contract since the disinherited person is not a third party beneficiary of the contract between attorney and testatrix.”-378

C. Undue Influence1. “When contestants challenge a will based on ‘undue influence’ they argue

that the written will does not reflect the testator’s ‘true’ intent. The contestants generally argue that the written will reflects the successful effort of a will beneficiary to substitute his own wishes for those of a testator susceptible to the beneficiary’s influence. Contestants need not argue that testator lacked capacity to make a will.”-379

Haynes v. First National State Bank of N.J.—S. Ct. of N.J., 1981Issue: Whether “the will is invalid on the grounds of ‘undue influence’ attributable to the fact that the attorney, who advised the testatrix and prepared the testamentary instruments, was also the attorney for the principal beneficiary, the testatrix’s daughter, in whom testatrix had reposed trust, confidence and dependency”? YESRule: “[I]t is generally presumed that ‘the testator was of sound mind and competent when he executed the will.’ . . . If a will is tainted by ‘undue influence,’ it may be overturned. ‘Undue influence’ has been defined as ‘mental, moral or physical’ exertion which has destroyed the ‘free agency of a testator’ by

56

Page 57: Wills, Trusts

preventing the testator ‘from following the dictates of his own mind and will and accepting instead the domination and influence of another.’ . . . When such a contention is made ‘the burden of proving undue influence lies upon the contestant unless the will benefits one who stood in a confidential relationship to the testatrix and there are additional circumstances of a suspicious character present which require explanation. In such case the law raises a presumption of undue influence and the burden of proof is shifted to the proponent.’”-386“The first element necessary to raise a presumption of undue influence, a ‘confidential relationship’ between the testator and a beneficiary, arises ‘where trust is reposed by reason of the testator’s weakness or dependence or where the parties occupied relations in which reliance is naturally inspired or in fact exists.’”-386“The second element necessary to create the presumption of undue influence is the presence of suspicious circumstances which, in combination with such a confidential relationship, will shift the burden of proof to the proponent.”-386“In this jurisdiction, once a presumption of undue influence has been established the burden of proof shifts to the proponent of the will, who must, under normal circumstances, overcome that presumption by a preponderance of the evidence.”-387“It has been often recognized that a conflict on the part of an attorney in a testimonial situation is fraught with a high potential for undue influence, generating a strong presumption that there was such improper influence and warranting a greater quantum of proof to dispel the presumption.”-388“Accordingly, it is our determination that there must be imposed a significant burden of proof upon the advocates of a will where a presumption of undue influence has arisen because the testator’s attorney has placed himself in a conflict of interest and professional loyalty between the testator and the beneficiary.”-389“Hence, the presumption of undue influence created by a professional conflict of interest on the part of an attorney, coupled with confidential relationships between a testator and the beneficiary as well as the attorney, must be rebutted by clear and convincing evidence.”-390

2. Notesa. “[I]f a will beneficiary enjoys a confidential relationship with testator,

and there is reason to believe that the beneficiary used the testator’s confidence and trust to benefit herself, the beneficiary has unduly influenced the testator.”-392

b. “The mere fact that a person who enjoys a confidential relationship with testator is also a will beneficiary does not, therefore, create a presumption of undue influence. Instead, as the court in the Haynes case indicates, suspicious circumstances must be present.”-392

c. “If, for instance, the will beneficiary had no discussions with testator about disposition of testator’s estate, no presumption of undue influence arises. See, e.g., Andrews v. Rentz.”-392

d. “Courts often treat dispositions as suspicious when made to persons who do not appear to be the natural objects of testator’s bounty. . . . Circumstances may be suspicious even if the parties accused of undue influence benefit only by collecting fees as fiduciaries rather than beneficial interests in testator’s estate.”-392

e. “Even when the persons accused of undue influence are natural objects of testator’s bounty – usually close relatives – courts may find suspicious circumstances when, as in the Haynes case, testator’s will has changed dramatically over time to benefit the supposed influencer.”-392

f. “Courts often require evidence of ‘weakened intellect’ or ‘weakened mental state’ before they will sustain a finding of undue influence. . . . At the very least, courts treat the physical and mental condition of

57

Page 58: Wills, Trusts

testator ‘as it affects his or her ability to withstand the influence’ as a relevant factor in undue influence cases.”-393

g. “Rule 1.8(c) of the ABA’s Model Rules provides: ‘A lawyer shall not prepare an instrument giving the lawyer or a person related to the lawyer as a parent, child, sibling, or spouse any substantial gift from a client, including a testamentary gift, except where the client is related to the donee.’”-393

Will of Moses—S. Ct. of Miss., 1969Issue: Whether the trial court erred in finding that the testator had been subject to undue influence on the part of her boyfriend/lawyer, who was made the sole beneficiary of her estate, where the testator was a 57 year-old alcoholic, and the boyfriend/lawyer is 42 years old? NOHolding: “The chancellor was justified in finding that the physical absence of Holland during Mrs. Moses’ brief visit to the office of the attorney who wrote the will did not suffice to abate or destroy the presumption of undue influence.”-397

3. Notesa. “Although the relationship between husband and wife is often a

relationship of trust and confidence, courts rarely find that the marital relationship, combined with a substantial devise to the spouse, gives rise to a presumption of undue influence.”-400

b. “Because of the fiduciary relationship between lawyer and client, courts look upon wills in which the lawyer is named as beneficiary with great disfavor, and the lawyer is generally unable to overcome the presumption of undue influence – even if the lawyer did not draft the will herself.”-401

c. “Occasionally, the lawyer gets to keep the devise. Thus, in Vaupel v. Barr, . . . where testator made her lawyer, a long-time friend, the residuary beneficiary of her estate, the court went so far as to grant summary judgment to the lawyer-beneficiary, noting that when the testator informed the lawyer, who had drafted her previous wills, that she wanted to make him a beneficiary, the lawyer immediately informed testator that another lawyer would have to draft the will.”-402

d. “Testators often enjoy confidential relationships with their clergymen, and unscrupulous clergymen may have a particular hold on aged testators – the promise of eternal salvation. Indeed, in days past, the mortmain statutes routinely invalidated excessive gifts to religious organizations. . . . Although mortmain statutes are on the wane, courts continue to scrutinize gifts to religious organizations for signs of undue influence. The spiritual advisor need not benefit personally for the gift to be suspect.”-402

e. “A disposition in favor of a nursing home operator is inherently suspicious.”-402

f. “Courts generally acknowledge that a presumption of undue influence can be rebutted by proof that testator acted after obtaining the counsel of persons independent of the alleged influencer.”-403

g. “After studying 160 undue influence cases decided over a five-year period, Professor Melanie Leslie concluded: ‘Courts often evaluate potential beneficiaries from their own perspective, as opposed to that of a testator, thus appearing less concerned with effectuating testamentary intent than in forcing the testator to distribute his or her estate in accordance with prevailing notions of morality.’”-403

58

Page 59: Wills, Trusts

h. Ray D. Madoff, Unmasking Undue Influence(i) “[T]he undue influence doctrine is akin to the doctrine of fraud

and duress in its attempt to protect testators’ rights to freely dispose of their property.”-403

(ii) “[F]amily protection is built into the very fabric of undue influence doctrine. Rather than resulting from a misapplication of the doctrine, . . . the correct application of the doctrine imposes a preference for the biological family over non-family members. The doctrine does not act to protect the intent of the testator, but rather to protect the testator’s biological family from disinheritance.”-404

D. FraudIn re Roblin’s Estate—S. Ct. of Or., 1957

Issue: Whether the trial court erred in dismissing a will contest on the ground of fraud where the contestant’s father left him only one dollar, and the contestant’s sister, who had a closer relationship to her father, received the remainder of his estate, where the sister made false statements to the father regarding the fact that she received only a diamond ring, while her brother received the remainder under his mother’s will? NORule: “Courts set aside wills whose provisions reflect the testator’s belief in false data arising from fraudulent misrepresentation made to him by a beneficiary.”-405“Fraud which causes testator to execute a will consists of statements which are false, which are known to be false by the party who makes them, which are material, which are made with the intention of deceiving testator, which deceive testator, and which cause testator to act in reliance upon such statements.”-406“The statement must be false and known to be such by the maker.”-406“We must distinguish between a belief in the literal truth and falsity of a statement and that type of belief in falsity that underlies the fraudulent misrepresentation.”-406“The speaker must intend to deceive, and succeed.”-406“The misrepresentation must cause the testator to act upon it. In other words, the will must be the fruit of the fraud.”-406

4. Notesa. “The constructive trust, as we have already seen is a remedial device

designed to prevent a person from unjustly retaining title to property. In Judge Cardozo’s famous words: ‘A constructive trust is the formula through which the conscience of equity finds expression. When property has been acquired in such circumstances that the holder of the legal title may not in good conscience retain the beneficial interest, equity converts him into a trustee.’”-408

b. “The constructive trust has also been invoked to aid beneficiaries of an unexecuted will who contend that beneficiaries of an earlier will fraudulently prevented testator from revoking the old will and executing the new one.”-408

c. “Latham v. Father Divine . . . presents the most graphic illustration of the use of the constructive trust to prevent fraud. Testator had written a will benefiting Father Divine, the leader of a religious cult, and his followers. Shortly before testator’s death, she had lawyers prepare a new will leaving property to her first cousins. The cousins alleged that Father Divine and his followers ‘conspired to kill, and did kill, the deceased by means of a surgical operation performed by a doctor engaged by the defendants . . . without the consent or knowledge of the relatives of the deceased.’ The cousins sought imposition of a

59

Page 60: Wills, Trusts

constructive trust on estate proceeds payable to Father Divine on the ground that he and his followers had fraudulently prevented execution of the new will. The Court of Appeals held that the complaint was sufficient to withstand a motion to dismiss.”-409

E. Preparing for the Contest: The Lawyer’s Role1. “The estate of Seward Johnson – heir to the Johnson & Johnson

pharmaceuticals fortune – became the subject of a nasty contest between Johnson’s children and his wife. The contest featured major New York law firms on each side of the controversy, and generated a popular book by a former legal affairs columnist for the New York Times.”-409

2. In the will contest, Johnson’s wife “the principal beneficiary of his will, agreed to pay $40 million out of the estate to settle the contest brought by Johnson’s children, and in which the estate was depleted by another $25 million to pay both sides’ legal fees.”-409

3. Notesa. “Because the ‘worst evidence’ principle of American probate

law requires the testator to be dead before the court decides whether he was capable when he was alive, good planners take steps to generate and preserve favorable evidence of the testator’s capacity and independence.”-411

b. “Another common precaution is to arrange for persons who are likely to survive the testator to inform themselves about his condition at the time of the making of the will, so that they can testify about the subject in the event of contest. Such persons might include both medical experts new to the testator, and persons who have been long familiar with him and are thus able to form a comparative view of his capacity at the time of the making of the will.”-412

c. “A third avenue of defense is to include a no-contest clause in the will.”-412

d. “What is a no-contest clause (also called an in terrorem clause because of its objective – striking terror in the heart of potential contestants)? In the Haynes case, lawyer Buttermore inserted no-contest clauses in Mrs. Dutrow’s will and in the trust instruments he drafted for Mrs. Dutrow. The clause included in one of the trust instruments, which was nearly identical to the clause in Mrs. Dutrow’s will, was as follows: ‘If any beneficiary under this trust shall contest the validity of, or object to this instrument, or attempt to vacate the same, or to alter or change any of the provisions hereof, such person shall be thereby deprived of all beneficial interest thereunder and of any share in this Trust and the share of such person shall become part of the residue of the trust, and such person shall be excluded from taking any part of such residue and the same shall be divided among the other persons entitled to take such residue.’”-413

e. “No-contest clauses . . . require the contestant to make an evaluation of the likelihood that the contest will prove successful. If the likelihood is small, the contestant might

60

Page 61: Wills, Trusts

choose not to contest, in order to preserve whatever provisions testator made for the contestant.”-414

f. “The New Jersey courts had long held no-contest clauses enforceable, subject to exceptions not relevant in the Haynes case. In 1977, however, after Dutrow’s death, the New Jersey legislature enacted a statute patterned on section 3-905 of the UPC, reproduced below.”-44

g. UPC § 3-905 provides: “A provision in a will purporting to penalize any interested person for contesting the will or instituting other proceedings relating to the estate is unenforceable if probable cause exists for instituting proceedings.”-414

h. “The UPC’s approach to no-contest clauses has not yet won general acceptance. . . . In both California and New York, no-contest clauses are enforceable even against contestants who have probable cause. . . . Both states make exceptions for contests based on allegations that the will was the product of forgery or that the will was revoked.”-415

i. “During the 1970s, there was a significant movement toward a system of ante-mortem probate. A number of scholars suggested that a procedure permitting evaluation of testator’s will before testator’s death would reduce the incidence of will contests by disappointed heirs seeking compromise settlements.”-416

j. “Three states – Arkansas, North Dakota, and Ohio – enacted ante-mortem probate statutes in the late 1970s.”-416

k. Aloysius A. Leopold & Gerry Beyer, Ante-Mortem Probate: A Viable Alternative

(i) “Despite the greater awareness of the ante-mortem probate alternative in Ohio, it nonetheless appears that the statute is infrequently used. In the first eight years of its availability, approximately eight ante-mortem probate cases were filed in Franklin County, one of Ohio’s largest counties.”-417

l. “Why did ante-mortem probate not garner more significant support?: ‘A failing of living probate is the high price it exacts from testators in return for an ephemeral assurance that their wills are secure from challenge. That price includes physical and mental examination, participation in the court proceeding, the cost of an attorney and of a guardian ad litem (depending upon the proposal adopted), and the risk of family disharmony. These costs will inevitably deter many testators from using the proceeding regardless of the benefits obtained.’”-417

F. Special Problems Affecting Gay and Lesbian Testators1. “[B]ecause homosexual testators cannot marry, and thereby assure an

intestate share for their life partners, the need to plan may be even greater than in more traditional families.”-417

Will of Kaufmann—S. Ct. of N.Y., App. Div., 1964Issue: Whether the trial court erred in holding that the testator was subject to undue influence of his partner where the partner became the sole beneficiary after the testator changed his will, the partner

61

Page 62: Wills, Trusts

exercised significant influence over the testator’s business affairs, and the testator left a letter explicitly explaining why the partner was made the beneficiary? NORule: “There are two principal categories of undue influence in the law of wills, the forms of which are circumscribed only by the ingenuity and resourcefulness of man. One class is the gross, obvious and palpable type of undue influence which does not destroy the intent or will of the testator but prevents it from being exercised by force and threats of harm to the testator or those close to him. The other class is the insidious, subtle and impalpable kind which subverts the intent or will of the testator, internalizes within the mind of the testator the desire to do that which is not his intent but the intent and end of another.”-424

2. Notes – Planning Strategies for Same-Sex Couplesa. “If testator believes that members of his family of origin would

be receptive, testator can enter into contracts with them by which they release their right to contest testator’s will. These contracts can provide testator with some reassurance against contest – but only if family members are sufficiently open-minded to make such agreements – the very situation in which contest is least likely.”-427

b. “Many jurisdictions place no explicit restrictions on adult adoptions. By adopting one’s partner, one might assure that the partner would inherit by intestate succession even if a will contest ensued. Moreover, if the testator has an adopted ‘child,’ the ‘child’ would, in most jurisdictions, take to the exclusion of collateral relatives and parents. As a result, no one other than the adoptee would have standing to contest the will [unless testator left prior wills benefiting other parties].”-427

c. “In Adoption of Robert Paul P., . . . [t]he Court of Appeals . . . rejected the adoption petition, concluding that ‘adoption is not a means of obtaining a legal status for a nonmarital sexual relationship – whether homosexual or heterosexual.’”-427

d. “On the other hand, in In re Adoption of Swanson, . . . the court issued a decree of adoption between a 65-year old man and his 51-year old companion of 17 years.”-428

e. “The Internal Revenue Code permits a testator to take a charitable deduction for the remainder interest in a charitable remainder annuity trust or a charitable remainder uni-trust. . . . By creating a charitable remainder trust, the gay testator may kill two birds with one stone: first, the testator may obtain a charitable deduction for part of his estate and, second, the testator may obtain an important ally in any will contest – the holder of the charitable remainder, who will want to defend the trust against contest by testator’s legal heirs. Indeed, the state Attorney General, as enforcer of charitable trusts, may also come to the aid of the testator’s named executor.”-428

f. “If testator and a partner choose to hold assets in their joint names, many lawyers recommend that testator’s will expressly indicate that the joint ownership arrangement was intended to convey ownership rights on the surviving partner, and was not intended merely as a matter of convenience. Because non-probate transfers tend to be less susceptible to after-death contests, many lawyers advise gay clients to make liberal use

62

Page 63: Wills, Trusts

of testamentary substitutes, which take assets out of the probate process. . . . Funded revocable trusts can be especially helpful.”-429

XVIII. TrustsA. Introduction

1. “A trust is an entity in which ownership is divided between the trustee (who is sometimes said to hold ‘legal’ title to the trust property) and the beneficiaries (said to hold ‘beneficial’ title). The trustee – not the beneficiary – has the right to manage the trust property, but also has the obligation to manage the property in the beneficiary’s interest – not the trustee’s own interest. In other words, if you wanted to buy some property held in trust, you would go to the trustee; if you were looking for a rich friend, you would look to the trust beneficiary.”-445

2. “The creator of a trust (generally called the ‘settlor’ or sometimes ‘trustor’) may want to leave property to people – whether adults or children – with limited capacity to manage money. By creating a trust, the settlor can repose management responsibilities in a trustee who is (the settlor hopes) up to the task.”-445

3. “Many trusts (called ‘testamentary trusts’) are created by will. In recent decades, however, many settlors have created trusts (called ‘inter vivos’ trusts’ or ‘living trusts’) while they are alive. A primary motivation for some of these trusts has been avoidance of probate.”-445

4. “Although changes in the tax laws have made it somewhat more difficult (and somewhat less lucrative) than it once was to use trusts as a means for avoidance of federal estate and income taxation, opportunities still exist to reduce tax liability through use of trusts.”-446

5. Note on Trusts and Legal Life Estatesa. “Settlors often create trusts to maintain control over property

(and, indirectly, over people) past their own deaths.”-446b. “A number of factors counsel against creating legal life estates.

Rarely would it be advisable for a lawyer to recommend creating a legal life estate rather than a trust.”-446

c. “First, when a life tenant and remaindermen share ownership of property, no one person has power to sell a fee simple interest in the property. . . . By contrast, if a trustee owns the property under a properly drafted instrument which includes a power of sale, the trustee could exchange the unproductive property for other property that better meets the needs of the intended beneficiary or beneficiaries.”-446

d. “Second, a legal life tenant has no power to lease the property for a period beyond the expiration of the life estate. . . . Again, by conveying the property in trust, and conferring on the trustee a power to lease the property, the settlor avoids this difficulty.”-447

e. Third, when ownership is divided between legal life tenant and remaindermen, the life tenant’s management responsibilities are often murky. . . . By contrast, when the owner of property creates a trust, the owner vests decision-making power in the trustee. The trustee decides what repairs are necessary and

63

Page 64: Wills, Trusts

whether sale or lease of the property would be advisable. With respect to potential buyers and tenants, the trustee has the powers of a fee owner.”-447

f. “For the property owner seeking to divide ownership over time, the trust is virtually always the preferred mechanism.”-447

B. Creation of Trusts1. Trust Requisites: The Beneficiary, the Property, and the Trustee

a. “If Ann, after consulting her lawyer, decides to create a trust – either during her lifetime or by will – her lawyer will draft a trust instrument in which she describes with care the trust beneficiaries, the trust property, and the trustee (or trustees). No one is likely to challenge the trust instrument for want of adequate property, beneficiaries, or trustee.”-447

b. “It is established law that every trust must have at least one beneficiary (see Restatement of Trusts (Second), sec. 112) and trust property (see id., sec. 74).”-448

c. The Trustee(i) “To the outside world, the trustee appears as the

‘owner’ of the trust property. . . . [T]he trustee manages the trust property. . . . Moreover, the trustee is a fiduciary. As a fiduciary, she must act in the interests of the trust beneficiaries, not in self-interest.”-448

(ii) “Statutes and case law provide a background definition for trustee duties – a ‘default’ law of fiduciary obligation – but the settlor of the trust can vary those obligations by contract, or by the terms of the trust instrument. Indeed, contract has come to play an increasingly important role in defining the trustee’s duties.”-448

(iii) John H. Langbein, The Contractarian Basis of the Law of Trusts – “The rules of trust law apply only when the trust instrument does not supply contrary terms.”-448

(iv) “A trust will not fail for want of a trustee. A vacancy in the office of trustee might arise for a number of reasons: (1) the trust instrument might not name a trustee; (2) the trustee named might fail to qualify (refusal to accept the position, death of a testamentary trustee before effective date of the trust, refusal of court to confirm the appointment because of incompetence, etc.), or might die, be removed, or resign after creation of the trust; (3) the trustee might lack legal capacity to hold in trust, as an unincorporated association would in some states.”-449

(v) “When the trust instrument does not account for these contingencies, a court will appoint a trustee to fill the vacancy and assure continuation of the trust. Obviously, a well-drafted trust instrument provides

64

Page 65: Wills, Trusts

for these contingencies; if it doesn’t, the client didn’t get what he paid for.”-449

(vi) “A trust settlor may name the sole trustee one of the beneficiaries of the trust, and may name the sole beneficiary one of the trustees of the trust. Moreover, the settlor may name the same two or three parties as trustees and as beneficiaries of the trust. The same person may not, however, serve as sole trustee and sole beneficiary. See Restatement (Second) of Trusts, sec. 99.”-449

(vii) One’s legal and equitable title “merge” and the trust terminates where, for example, one is the life beneficiary and sole trustee, and the remainder beneficiary dies.-449

(viii) “The merger doctrine does not apply if the sole beneficiary is under an incapacity, or if the settlor has created a spendthrift trust for the sole beneficiary, who has become the sole trustee by reason of death, resignation, or removal of a co-trustee. In these circumstances, a court will simply appoint a co-trustee rather than terminating the trust. See Restatement (Second) of Trusts, sec. 341.”-449

(ix) “[I]f a trust imposes no active duties on the trustee, the trust is invalid as a ‘passive’ trust, and the beneficiary acquires legal as well as equitable title. . . . American statutes and cases typically hold passive trusts invalid whether the subject matter is land or personalty.”-450

(x) “Why would anyone care whether a trust is passive? A beneficiary might want to challenge a trust as passive simply to avoid paying commissions to the named trustee. Perhaps most often, the beneficiary’s challenge rests on a desire to own the property outright – in particular, to have control over the trust property, rather than merely beneficial ownership. Finally, a beneficiary might want to challenge a trust as passive to avoid having the trust invalidated on some other ground – for indefiniteness, or, perhaps, for violation of the Rule Against Perpetuities.”-450

d. The Need for Identifiable Beneficiaries(i) “No trust fails for want of a trustee, but no trust

exists in the absence of a beneficiary.”-450Moss v. Axford—S. Ct. of Mich., 1929

Issue: Whether a will provision directing the executor of a testator’s estate to “give, devise, and bequeath all the rest, residue, and remainder of [the testator’s property] . . . to the same person who has given me the best care in my declining years and who in his opinion is the most worth of my said property” is invalid for failure to specify a beneficiary? NOHolding: “We do not read the will as conferring on Mr. Axford unrestrained discretion or right of personal opinion in the designation of a beneficiary.”-452

65

Page 66: Wills, Trusts

“We think the clause a valid devise in trust, and that the designation of Mrs. Piers by Mr. Axford constituted her the residuary legatee.”-453Rule: The purpose of a will provision directing the executor to distribute property to an unnamed beneficiary is lawful and will be enforced “unless there is such an uncertainty that the law is fairly baffled. . . . It is not necessary that a beneficiary be designated by name, or by a description which makes identification automatic. . . . Nor that the testator have in mind the particular individual upon whom his bounty may fall. . . . It is enough if the testator uses language which is sufficiently clear to enable the court by extrinsic evidence to identify the beneficiary. If by such evidence the court can make the identification necessary to give effect to the intention of the testator, the devise will be sustained.”-452“A trust is not invalidated by the fact that the trustee is vested with discretion.”-452

e. Notes(i) “The requirement that trusts have identifiable

beneficiaries is designed, in part, to assure that someone has power to enforce the trust.”-453

(ii) In Clark v. Campbell, the court held that “the word ‘friends’ unlike ‘relations’ has no accepted statutory or other controlling limitations, and in fact has no precise sense at all.”-454

(iii) “In California, it seems to be possible to create a trust for the benefit of settlor’s friends. By statute, the legislature has decreed that ‘a trust created for an indefinite or general purpose is not invalid for that reason if it can be determined with reasonable certainty that a particular use of the trust property comes within that purpose.”-454

(iv) Assuming Franklin is the testator and Fala is a pet: “So long as Fala has no capacity to sue, a trust for Fala’s benefit creates no enforceable duties in the trustee, and hence would appear to fail as a trust.”-455

(v) “Although case law on the issue is sparse, there have been suggestions that a trust for Fala’s benefit could be treated as an ‘honorary trust’, which operates like a power or a conditional gift. That is, a trust for Fala would not fail outright, but the trustee would be entitled to take the trust funds so long as the trustee used them for Fala’s benefit. If the trustee failed to use the funds for Fala’s benefit, the honorary trust would fail, and the property would pass back into the residue of Franklin’s estate or, if the honorary trust were the residuary bequest, by intestate succession.”-455

(vi) “Section 2-907 of the UPC . . . explicitly validates trusts for the benefit of pets. The statute also authorizes the settlor to designate an individual with power to enforce trust obligations, thus moving a step beyond the honorary device.”-456

f. Trust PropertyBrainard v. Commissioner—7th Cir., 1937

66

Page 67: Wills, Trusts

Issue: Whether “under the circumstances set forth in the findings of the Board of Tax Appeals, the taxpayer created a valid trust, the income of which was taxable to the beneficiaries under section 162 of the Revenue Act of 1928”? NORule: “In the Restatement of the Law of Trusts, vol. 1, § 75 it is said that an interest which has not come into existence or which has ceased to exist can not be held in trust. It is there further said: ‘If a person gratuitously declares himself trustee of such shares as he may thereafter acquire in a corporation not yet organized, no trust is created. The result is the same where instead of declaring himself trustee, he purports to transfer to another as trustee such shares as he may thereafter acquire in a corporation not yet organized. In such a case there is at most a gratuitous undertaking to create a trust in the future, and such an undertaking is not binding as a contract for lack of consideration.’”-457Restatement § 26k provides: “If a person manifests an intention to become trustee at a subsequent time, his conduct at that subsequent time considered in connection with his original manifestation may be a sufficient manifestation of intention at that subsequent time to create a trust. . . . [T]he act of acquiring the property coupled with the earlier declaration of trust may be a sufficient manifestation of an intention to create a trust at the time of the acquisition of the property.”-458

(i) “In Speelman v. Pascal, . . . a theatrical producer who held the rights to make a musical version of George Bernard Shaw’s Pygmalion, assigned to his secretary (who was also his lover) a percentage of the profits to be derived from the musical stage version of the play – a version which had not yet been written. Did the secretary acquire an enforceable right to the future profits from what became My Fair Lady? The Court of Appeals held that she did.”-459

(ii) “On occasion, testator’s lawyer might, as part of an estate plan, draft a trust instrument designed to receive testator’s assets at death, and then to distribute those assets to designated beneficiaries. These trusts, sometimes called ‘standby trusts’ may not actually be funded with any assets during testator’s lifetime. When unfunded standby trusts are challenged as testamentary, courts, with the aid of statutes, usually uphold the trusts.”

(iii) “The UPC (section 2-511) and statutes of many other states, explicitly reject the argument that standby trusts cannot receive estate assets unless they are executed in accordance with testamentary formalities.”-460

2. Intent to Create a Trust: The Precatory Language Problema. “To create a private express trust, a settlor must express an

intent to impose an enforceable duty on the trustee.”-460b. “On occasion, . . . testators annex ‘precatory words’ – words of

request or entreaty – to devises or bequests, and thus create doubt whether an absolute gift or trust is intended.”-460

Spicer v. Wright—S. Ct. of Va., 1975Issue: Whether “the language of Mrs. Spicer’s will, read in context with the extrinsic evidence, is sufficient to establish an intent to create an express trust” where she said, in a holographic will, “My estate . . . I give to my sister, Anne Beecher Wilson to be disposed of as already agreed between us”? NHolding: “We hold that the language Mrs. Spicer employed is precatory, that the extrinsic evidence is insufficient to render that language imperative or to establish a testamentary intent to impose a legal obligation to make a particular disposition of property, that no express trust was intended or created, and that the language constitutes an absolute testamentary grant to Miss Wilson.”-462Rule: “Precatory words are prima facie construed to create a trust when they are directed to an executor . . . but no trust is created by precatory language directed to a legatee unless there is

67

Page 68: Wills, Trusts

testamentary intent to impose a legal obligation upon him to make a particular disposition of property.”-461

Levin v. Fisch—Tx. Ct. of Civ. App., 1966Issue: Whether the appellants-children-executors of the testator’s estate are required to contribute $2400/year to their mother’s sister in keeping with a will provision that it is the testator’s “desire” that such money be paid to the sister? YESHolding: “Based upon the rule announced in the above cited cases and authorities it is our opinion that it was the intention of the testatrix that the words ‘desire’ as used in the will were a positive directive and imposed an obligation on appellants to comply therewith.”-464Rule: “The word ‘desire’ in its ordinary and primary meaning is precatory, but is often construed when used in a will as directive or mandatory when it clearly appears that such was the intention of the testator from a consideration of the instrument as a whole and the surrounding circumstances.”-463“In determining whether particular words are to be construed as precatory or mandatory, the court will look to the expressed intent of the testator, as found from the context of the will and surrounding circumstances; and words which, in their ordinary meaning, are precatory will be construed as mandatory only when it is evident that such was the testator’s intent.”-464“The word ‘desire’ in its ordinary and primary meaning is precatory, but is often construed when used in a will as directive or mandatory when it clearly appears that such was the intention of the testator from a consideration of the instrument as a whole and the surrounding circumstances.”-463“In determining whether particular words are to be construed as precatory or mandatory, the court will look to the expressed intent of the testator, as found from the context of the will and surrounding circumstances; and words which, in their ordinary meaning, are precatory will be construed as mandatory only when it is evident that such was the testator’s intent.”-464

3. Trust FormalitiesGoodman v. Goodman—S. Ct. of Wash., 1995

Issue: Whether the trial court erred in deciding as a matter of law that the statute of limitations had run on the Ps’ claim that Gladys was intended, by Clive (testator) to hold Clive’s property for the benefit of his children until they were old enough to handle the responsibility? YESRule: “A constructive trust is an equitable remedy imposed by courts when someone should not in fairness be allowed to retain property.”-467“An express trust, on the other hand, arises because of expressed intent and involves a fiduciary relationship in which the trustee holds property for the benefit of a third party.”-468“The statute of limitations on an express trust action begins to run when the beneficiary of the trust discovers or should have discovered the trust has been terminated or repudiated by the trustee.”-468

a. Notes(i) “[L]egislatures and courts worry a lot about

formalities when a decedent seeks to make a transfer by will. Yet, in Goodman v. Goodman, and other trust cases, courts are willing to enforce a trust agreement without any written evidence of the terms of the trust, without any signature, and without any witnesses.”-469

(ii) “In all but a few American states, no writing is necessary to establish a trust of personal property. (That does not, of course, mean that no writing is desirable.)”-469

(iii) “A trust settlor may create an inter vivos trust by declaring that she holds the trust property, as trustee, in trust for named beneficiaries, or by transferring the trust property to someone else as

68

Page 69: Wills, Trusts

trustee for the benefit of the named beneficiaries.”-470

(iv) “When settlor transfers the property in trust to someone else as trustee, the settlor must deliver the trust property (or a suitable instrument representing the trust property) to the named trustee. . . . The delivery requirement for transfers in trust, like the delivery requirement for inter vivos gifts, is designed to assure that settlor truly intended to make a transfer . . . .”-470

(v) “[C]ourts generally uphold declarations of trusts of real property even if the settlor has not recorded the declaration or any instrument of transfer.”-470

(vi) Fla. Stat. 737.111 – Execution Requirements for Express Trusts – “(1) The testamentary aspects of a trust defined in section 731.201(33), are invalid unless the trust instrument is executed with the formalities required for the execution of a will. (2) The testamentary aspects of a trust created by a nonresident of Florida, either before or after this law takes effect, are not invalid because the trust does not meet the requirements of this section, if the trust is valid under the laws of the state or country where the settlor was at the time of execution. (3) The testamentary aspects of an amendment to a trust are invalid unless the instrument is executed by the settlor with the same formalities as a will. (4) For the purposes of this section, the term ‘testamentary aspects’ means those provisions of the trust that dispose of the trust property on or after the death of the settlor other than to the settlor’s estate.”-471

(vii) “The private express trust . . . shares the ‘trust’ label with two other, significantly different, devices: the constructive trust and the resulting trust. The constructive trust is not really a trust at all, but a remedial device used by courts to achieve results which do not easily fit within other doctrinal frameworks. No one ‘intends’ to create a constructive trust. Instead, the constructive trust . . . is a flexible remedial device often used to prevent unjust enrichment. In Judge Cardozo’s words: ‘A constructive trust is the formula through which the conscience of equity finds expression. When property has been acquired in such circumstances that the holder of the legal title may not in good conscience retain the beneficial interest, equity converts him into a trustee.’”-471

(viii) “A resulting trust generally arises when a settlor intends to create a trust, but the trust fails for some reason. . . . Practically, all that means is that once the court concludes that the express trust fails, the

69

Page 70: Wills, Trusts

‘trustee’ must transfer the property back to settlor. Similarly, if the trust has more property than necessary to accomplish its purposes, the trustee is said to hold the remainder on a resulting trust for the benefit of settlor or settlor’s successors-in-interest. In a sense, the resulting trust is an implied equitable reversion in the settlor.”-472

(ix) “The other important category of resulting trust is the ‘purchase money resulting trust.” A purchase money resulting trust is illustrated by the following: if X paid for land, but arranged for the seller to transfer title to Y, the presumption is not that X made a gift to Y, but that Y held the title in trust for X. “In many jurisdictions, the purchase money resulting trust has been abolished by statute.”-472

C. Using Trusts as an Estate Planning Tool1. Avoiding Probate

a. “[P]robate often entails costs and delays. The principal costs are (1) commissions paid to the executor or administrator selected to administer the estate; and (2) legal fees paid to the lawyer who steers the estate through the probate costs. These costs vary significantly from state to state.”-472

b. “By transferring essentially all of one’s assets into an inter vivos trust, one can effectively arrange to have no probate estate at death, and hence, nothing to probate. Moreover, the settlor of the trust can often retain nearly complete control of the trust property.”-473

c. “Because the trust device permits the settlor to retain so much control, most people prefer trusts to outright gifts, which might eliminate the need for probate, but put the donor at the mercy of donees who might be none too eager to care for the donor in her time of need.”-473

d. Is the Trust Invalid As “Testamentary”?(i) “When a settlor creates an inter vivos trust, but

retains virtually unlimited control, the beneficiaries of settlor’s estate sometimes contend that the trust was an invalid attempt to make a will without testamentary formalities.”-473

Green v. Green—S. Ct. of R.I., 1989Issue: Whether the trial court erred in holding that the testator had created valid totten trusts, and that the funds in the trust accounts were properly distributed to each of the named beneficiaries, where there was evidence that the testator sought to avoid probate, and also that the testator intended that the beneficiaries receive the value in each of the eight bank accounts? NOHolding: “We believe that Green specifically intended to dispose of his property through totten trusts. Therefore, the beneficiaries are entitled to the full amount of these trusts.”-476Rule: “A totten trust is defined as a deposit in trust by the settlor of his own money for the benefit of another. . . . The creation of a valid totten trust requires retention of the subject matter of the trust by the settlor or the trustee for the benefit of a named beneficiary. . . . The settlor may be the named trustee, but this is not necessary to the validity of the trust. . . . During the settlor’s lifetime the trust is revocable, and the settlor may use the funds for his own benefit. . . . Upon the settlor’s death, however, the trust becomes irrevocable and is the exclusive property of the beneficiary.”-475

70

Page 71: Wills, Trusts

“Because of the distinction between a trust and a gift, the settlor’s intention is critical to the validity of a totten trust. The intention to create a trust must be made during his lifetime. . . . The settlor is not required to use any particular form of words to create a trust but must convey the property to another in trust or unequivocally declare that he holds it in praesenti in trust or as trustee for another.”-475“[A]ctual notice to the beneficiary is not essential to the creation, existence, or validity of a trust. . . . Nor is an acceptance of or assent to a trust by the beneficiary necessary. . . . Finally, . . . the creation of a valid totten trust is a question of intention and, therefore, a question of fact.”-476“The factfinder may look to the settlor’s statements and conduct regarding the account in discerning the settlor’s intent. . . . The form of the account creates a rebuttable presumption that a valid trust exists for the named beneficiary. . . . Merely making a deposit in trust for another, however, is not conclusive evidence of the intent to create a trust. . . . Nevertheless, if the depositor dies before the beneficiary, leaving an unexplained bank account in the form of a trust and not revoking or disaffirming the trust during his lifetime, then a prima facie case supporting the creation of a valid trust exists.”-476“Once the prima facie case is established, the burden of going forward shifts to the party opposing the trust. If that party presents satisfactory evidence to rebut the prima facie case, then the presumption of a valid trust becomes inoperative. The case will then proceed as if no presumption had been invoked. . . . However, if the prima facie case is not overcome by any reliable testimony and/or substantive evidence, then the trust will be upheld as valid.”-476

(ii) “[B]ank account trusts are often called ‘Totten’ trusts after In re Totten, . . . in which the New York Court of Appeals first sustained a bank account trust against the claim that it constituted an invalid testamentary transfer. Most, but not all states, have rejected the argument that Totten trusts are unenforceable as testamentary transfers.”-478

(iii) “Because bank account trusts create no enforceable obligations in the settlor-trustee, and because the settlor can revoke the trust by withdrawing money, closing the account, or changing the name of the beneficiary, Totten trusts have also been called ‘tentative’ trusts.”-478

(iv) “What happens to the funds on deposit If Jane dies [where Jane and her husband John have a joint checking account]? During the lifetime of Jane and John, either has power to withdraw all of the funds, but, unknown to most depositors, each generally has a claim against the other for withdrawals in excess of the amount deposited by the withdrawing account holder! That is, Jane and John do not jointly ‘own’ the money in the account; John has a claim only to his contribution to the present balance.”-478

(v) “Typically, the documents creating a joint account include a survivorship feature, and statutes in many states expressly authorize banks to pay over all of the funds in such an account to either holder of the account. Thus, if one of the accountholders were to die, the other could withdraw the funds without worrying about probate.”-478

(vi) “Suppose Jane Smith opens an account in her own name, but provides that the funds will be ‘payable

71

Page 72: Wills, Trusts

on death to John Smith.’ John has no right to the funds during Jane’s lifetime, but Jane has expressed an intention that John take the funds at Jane’s death. On occasion, courts have held that John is not entitled to the funds because the POD provision constitutes a testamentary transfer without testamentary formalities. The trend is toward enforcement, and the UPC moves substantially in that direction.”

(vii) “The 1989 revision of article VI of the UPC ‘consolidates treatment of POD accounts and trust accounts so that the same rules apply to both, since both types of account operate identically and serve the same function of passing property to a beneficiary at the death of the account owner.’”-479

(viii) “Section 6-101 provides explicitly that a wide variety of nonprobate transfers on death – including those in account agreements – are nontestamentary. Section 6-204 provides a form contract of deposit which would enable depositors to choose between a ‘single-party account’ and a ‘multiple-party account.’ On a single-party account, the depositor would then choose whether to include a POD designation, or, alternatively to have assets pass to the depositor’s estate at death. On a multiple-party account, the depositors could choose whether to create a right of survivorship, and whether to include a POD designation at the death of the surviving depositor. The UPC includes no separate form for totten trusts, because a single-party account with a POD designation is the functional equivalent of a Totten trust.”-479

Westerfeld v. Huckaby—S. Ct. of Tx., 1971Issue: Whether “Virginia B. Miller, dealing with her own property, could create valid trusts even though she reserved in herself broad beneficial rights, as well as the right to revoke the trusts and the right to control or manage the acts of the trustee”? YES Holding: “[T]he court of civil appeals correctly held that an application of the illusory trust doctrine, as enunciated in Land v. Marshall, . . . is limited to instances in which a non-consenting spouse’s property is used to fund a trust.”-481Rule: Section 57 of the Restatement of Trusts provides: “Where an interest in the trust property is created in a beneficiary other than the settlor, the disposition is not testamentary and invalid for failure to comply with the requirements of the Statute of Wills merely because the settlor reserves a beneficial life interest or because he reserves in addition a power to revoke the trust in whole or in part, and a power to modify the trust, and a power to control the trustee as to the administration of the trust.” Comment b provides: “Where settlor reserves power of control. Where the owner of property transfers it inter vivos to another person in trust, the fact that he reserves not only a power to revoke and modify the trust but also power to control the trustee as to the administration of the trust does not make disposition testamentary and invalid for failure to comply with the requirements of the Statute of Wills.”-482Comment h provides: “Declaration of trust. The rule stated in this Section is applicable not only where the owner of property transfers it to another as trustee, but also where he declares himself trustee of the

72

Page 73: Wills, Trusts

property. The disposition is not testamentary and invalid for failure to comply with the requirements of the Statute of Wills merely because the settlor-trustee reserves a beneficial life interest and power to revoke and modify the trust. The fact that as trustee he controls the administration of the trust does not invalidate it.”-482

(ix) “[W]hen settlor’s spouse challenges a revocable inter vivos trust as illusory because the settlor retained full control over the trust’s assets until the moment of death, the spouse often prevails.”-486

(x) “Under modern elective share statutes (including the UPC), the revocable trust will be included in the ‘augmented estate’ or will otherwise be available to satisfy the surviving spouse’s claims.”

(xi) “In evaluating the validity of revocable trusts as a device to cut off the surviving spouse, some courts focus on settlor’s intent . . . while others, including Newman v. Dore, . . . focus on the degree of control reserved by settlor.”-486

(xii) “In Sullivan v. Burkin, . . . the Massachusetts Supreme Judicial Court announced that it would subsequently adopt another rule protecting the surviving spouse against disinheritance by use of revocable trusts: ‘The rule we now favor would treat as part of the estate of the deceased for purposes of [the forced heirship statute] assets of an inter vivos trust created during the marriage by the deceased spouse over which he or she alone had a general power of appointment, exercisable by deed or will. This objective test would involve no consideration of the motive or intention of the spouse in creating the trust. We would not need to engage in a determination of ‘whether the spouse has in good faith divested himself of ownership of his property or has made an illusory transfer’ . . . or with the factual question whether the spouse ‘intended to surrender complete dominion over the property.’ . . . Nor would we have to participate in the rather unsatisfactory process of determining whether the inter vivos trust was on some standard, ‘colorable,’ ‘fraudulent,’ or ‘illusory.’”-487

(xiii) “Because many jurisdictions do not have statutes expressly including revocable trusts in an augmented estate for elective share purposes, the illusory trust doctrine remains important.”-487

e. Marketing Revocable Trusts: Professional Responsibility Issues

Committee on Professional Ethics v. Baker—S. Ct. of Iowa, 1992Issue: Whether certified financial planner Voegtlin and/or attorney Baker engaged/assisted in the unauthorized practice of law where Voegtlin would encourage clients to use living trusts as a means of avoiding probate, and Baker would draft the appropriate legal documents after nearly all of the clients were referred to him through Voegtlin? YES

73

Page 74: Wills, Trusts

Holding: “We reprimand Baker for aiding Voegtlin in the unauthorized practice of law and for allowing Voegtlin to direct or regulate Baker’s professional judgment in rendering legal services to Baker’s clients.”-495Rule: DR 5-107(B) provides: “A lawyer shall not permit a person who recommends, employs, or pays him to render legal services for another to direct or regulate his professional judgment in rendering such legal services.”-494EC 5-1 provides: “The professional judgment of a lawyer should be exercised, within the bounds of the law, solely for the benefit of his client and free of compromising influences and loyalties. Neither his personal interests, the interests of other clients, nor the desires of third persons should be permitted to dilute his loyalty to his client.”-494

f. Life Insurance Trusts as a Probate Avoidance Device(i) “When a person buys life insurance, she has a

number of options for designating beneficiaries. She can simply make the insurance proceeds payable to a named beneficiary, she can make the proceeds payable to her estate, or she can create an inter vivos trust, and name the trustee as beneficiary of the life insurance policy.”-496

(ii) “Unfunded life insurance trusts have survived attacks contending that they are invalid as ‘testamentary’ because no property passes into the trust until the beneficiary’s death.”-496

(iii) “Other life insurance trusts are funded; the settlor transfers to the trustee not only the policies, but resources sufficient to pay the premiums. The trustee typically covenants to manage the securities and to apply the income to the insurance premiums.”-496

(iv) “In general, life insurance trusts, if funded both with life insurance policies and with assets to allow for the payment of premiums, are not suited to individuals with small estates because of the trustee’s fees that are likely to be required.”-496

2. Building Flexibility into the Estate Plan: Support Trusts and Discretionary Trusts

a. “Support Trusts and Discretionary Trusts enable the testator to account for events not known at the time the testator dies or at a time a settlor creates an inter vivos trust. They also enable a testator to provide for the unknown needs of incapacitated beneficiaries.”-497

b. “When settlor creates a discretionary trust, the settlor imposes no mandatory obligation on the trustee. Instead, the settlor gives the trustee discretion to pay income (and/or to invade principal) for the benefit of one or more described beneficiaries.”-497

c. “By contrast, settlor creates a support trust by giving the trustee power to pay income for the support of a named beneficiary. Settlor need not simply use the word ‘support’; the trust instrument might authorize payment of income for the ‘support and education,’ or for the ‘maintenance’ of the beneficiary’s

74

Page 75: Wills, Trusts

lifestyle. The trustee is then obligated to pay the beneficiary the amounts necessary for support.”-497

d. Ginsberg Abravanel – Discretionary Support Trusts – “Frequently, the discretionary element of a support trust is made explicit, as when discretionary language is combined with language that, if taken alone, would be deemed to create a pure support trust. For example, a property owner transfers certain assets ‘to T, in trust, to pay to W or apply so much of the income or principal of the trust, from time to time, as T shall, in his uncontrolled discretion, deem necessary for the support and maintenance of W,’ with remainder over to some third person upon the termination of the life tenant’s interest. This language creates a species of trust termed a discretionary support trust.”-498

Wells v. Sanford—S. Ct. of Ark., 1984Issue: Whether “the assets of a testamentary trust should be used to support a woman who has been declared incompetent before her own assets, as controlled by her guardian, are used”? YESHolding: “Our decision today directs that the trust assets be used to support Nora Wells, irrespective of her own assets, and authorizes the sale of those assets, if necessary, to comply with our decision.”-501Rule: “[U]ness something appears in the will indicating a different purpose, it is ordinarily presumed that the testator intended the beneficiary to be supported and maintained from estate income or . . . from sale of a part of the corpus.”-500“We have held that when there is nothing in the will to indicate that the testator did not understand the meaning of the words he used, we must presume that he did.”-500“When technical phrases or terms of art are used, it is fair to presume that the testator understood their meaning, and that they expressed the intention of his will, according to their import and signification. When certain terms or words have by repeated adjudication received a precise, definite and legal construction, if the testator in making his will uses such terms or similar expressions, they shall be construed according to their legal effect.”-500“It is axiomatic that a testator can only convey by will such property as he owns and that he cannot, through his will, control the estate of another.”-500

Marsman v. Nasca—App. Ct. of Mass., 1991Issue: “Does a trustee, holding a discretionary power to pay principal for the ‘comfortable support and maintenance’ of a beneficiary, have a duty to inquire into the financial resources of that beneficiary so as to recognize his needs?” YES“If so, what is the remedy for such failure?” Holding: “The conclusion of the trial judge that, had he exercised ‘sound judgment,’ he would have made such payments to Cappy ‘as to allow him to continue to live in the home he had occupied for many years with the settlor’ was warranted.”-506“The remedy in such circumstances is to impress a constructive trust on the amounts which should have been distributed to Cappy but were not because of the error of the trustee.”-506“[W]e hold that, since there was no evidence that the insertion of the clause was an abuse of Farr’s fiduciary relationship with Sara at the time of the drawing of her will, the clause is effective.”-508Rule: “Prudence and reasonableness, not caprice or careless good nature, much less a desire on the part of the trustee to be relieved from trouble . . . furnish the standard of conduct.”-505“That there is a duty of inquiry into the needs of the beneficiary follows from the requirement that the trustee’s power ‘must be exercised with that soundness of judgment which follows from a due appreciation of trust responsibility.’”-506

e. Dunkley v. Peoples Bank & Trust Co.—In this case, the trustee bank distributed to the trust beneficiary nearly all of the principal ($140,000) for the purchase of a house, allegedly for

75

Page 76: Wills, Trusts

medical reasons. It was alleged by the remaindermen that the trustee breached his duty to them. “The court directed the bank to make the trust whole by returning to the trust the amount wrongfully paid to Mr. Rushke, and indicated that the bank could recover from Mr. Rushke any amounts wrongfully paid to him.”-509

f. Restatement (Second) of Trusts § 187 provides: “Where discretion is conferred upon the trustee with respect to the exercise of a power, its exercise is not subject to control by the court, except to prevent an abuse by the trustee of his discretion.”-510

g. “If the trust had been a pure discretionary trust, without any ascertainable standards, the trustee would have been entitled, but not required, to consider outside income.”-510

h. “In other trusts, the trustee has discretion to pay the income to one or more named beneficiaries. Thus, a settlor could create a trust giving the trustee discretion to pay income to any of settlor’s children, as their needs might appear. Because trustee has discretion to ‘spray’ the income among the various beneficiaries, this form of discretionary trust is sometimes called a ‘spray’ trust or a ‘sprinkle’ trust.”-511

i. “Often, a testator will create a trust for the benefit of a spouse as part of an estate plan designed to make maximum use of settlor’s unified credit. A trust created for this purpose is often called a ‘credit-shelter’ trust. Testator will often want to make the credit-shelter trust a discretionary trust.”-511

j. Hess – The Federal Taxation of Nongeneral Powers of Appointment – “For the most part, the courts have no trouble finding that a power to invade for both permissible and impermissible purposes is a general power and not one subject to the appropriate ascertainable standard. Thus, a power to invade for ‘support, maintenance, comfort and welfare’ or ‘not only for her support and maintenance but also for her comfort and pleasure’ or ‘for her comfort, maintenance and support’ or ‘for his care, comfort and enjoyment’ have been found to grant the powerholder a general power of appointment rather than one limited by an ascertainable standard.”-512

3. Protecting Beneficiaries from Creditors: More on Support and Discretionary Trusts, and an Introduction to Spendthrift Trusts

a. In nearly every state, a trust beneficiary may assign his or her interest in the trust income. However, “[t]he result is that settlor’s intention has been frustrated: she has not assured a steady income for her son.”-514

b. “Now suppose the son made no voluntary assignment to Boatseller, but instead gave Boatseller a note for $200,000 in exchange for the yacht. If the son defaulted on the note, and Boatseller obtained a judgment against the son, Boatseller, as judgment creditor, would (in most states) be entitled to garnish the trust income in the hands of the trustee, forcing the trustee to pay the trust income to Boatseller rather than the son. Again, the settlor’s intention would have been frustrated.”-514

76

Page 77: Wills, Trusts

c. “Suppose, however, the settlor had created a support trust for her son’s benefit, using the following language: ‘I direct my trustee to pay to my son such sums as shall be necessary to provide for his support, education, and maintenance.’ . . . If the yacht is not necessary for the son’s support, trustee is not obligated (or permitted) to pay Boatseller trust income or principal. At the same time, despite the assignment, trustee may pay trust income directly to the son to use for his support, or to suppliers of goods and services necessary for the son’s support, education, or maintenance.”-514

d. “Would Boatseller have any rights against the trustee? Again, the answer is no; because settlor has authorized the trustee to use the trust’s funds only for the support of the beneficiary, a creditor who is not providing support is not entitled to garnish the beneficiary’s interest in the trust.”-514

e. “A support trust, then, provides a beneficiary with substantial protection against creditor claims. At the same time, however, the support trust limits the trustee’s power to use the money for any purposes other than support of the beneficiary.”-515

Wilcox v. Gentry—S. Ct. of Kan., 1994Issue: Whether the trial court erred in holding that only funds paid directly to a discretionary trust beneficiary are subject to garnishment by a creditor, where the trustee also had discretion to provide funds for the support of the trust beneficiary? YESHolding: “We adopt the Restatement (Second) of Trusts § 155(2) and find it determinative of this issue. The district court and the Court of Appeals erred in holding that only funds paid directly to a discretionary trust beneficiary are subject to garnishment by a creditor.”-518Rule: Restatement of Trusts (Second) § 155(2): “Unless a valid restraint on alienation has been imposed in accordance with the rules stated in §§ 152 and 153, if the trustee pays to or applies for the beneficiary any part of the income or principal with knowledge of the transfer or after he has been served with process in a proceeding by a creditor to reach it, he is liable to such transferee or creditor.”-516

f. Spendthrift Trusts(i) “The most common mechanism for keeping trust

assets away from a beneficiary’s creditors is inclusion of a ‘spendthrift’ provision in the trust instrument. For instance, the trust instrument might provide: ‘The interests of my trust beneficiary, whether in trust income or trust principal, shall not be capable of assignment, anticipation, or seizure by legal process.’”-518

(ii) “Spendthrift provisions are designed both to prevent voluntary assignments, and to prevent the beneficiary’s creditors from garnishing the beneficiary’s trust interest.”-518

(iii) “Unlike support trusts, spendthrift trusts permit a trust settlor to insulate trust assets from creditors while permitting the beneficiary to enjoy trust assets for purposes other than support or education. Unlike discretionary trusts, spendthrift trusts permit the trustee to make payments to the beneficiary while withholding them from the beneficiary’s

77

Page 78: Wills, Trusts

creditors. Since their introduction, however, spendthrift trusts have remained controversial.”-519

Broadway National Bank v. Adams—S. Jud. Ct. of Mass., 1882Issue: “Whether the founder of a trust can secure the income of it to the object of his bounty, by providing that it shall not be alienable by him or be subject to be taken by his creditors”? YESHolding: “[T]he income of the trust fund created for the benefit of the defendant Adams cannot be reached by attachment, either at law or in equity, before it is paid to him.”-521Rule: “The court will not compel the trustee . . . to do what the will forbids him to do, unless the provisions and intention of the testator are unlawful.”-519“[T]he rule of the common law is, that a man cannot attach to a grant or transfer of property, otherwise absolute, the condition that it shall not be alienated; such condition being repugnant to the nature of the estate granted.”-519“Under our system, creditors may reach all the property of the debtor not exempted by law, but they cannot enlarge the gift of the founder of a trust, and take more than he has given.”-521“The rule of public policy which subjects a debtor’s property to the payment of his debts does not subject the property of a donor to the debts of his beneficiary, and does not give the creditor a right to complain, that, in the exercise of his absolute right of disposition, the donor has not seen fit to give the property to the creditor, but has left it out of his reach.”-521“[W]e are of opinion that any other person, having the entire right to dispose of his property, may settle it in trust in favor of a beneficiary, and may provide that it shall not be alienated by him by anticipation, and shall not be subject to be seized by his creditors in advance of its payment to him.”-521

Bacardi v. White—S. Ct. of Fla., 1985Issue: Whether “disbursements from spendthrift trusts can be garnished to satisfy court ordered alimony and attorney’s fee payments before such disbursements reach the debtor-beneficiary”? YESHolding: “We quash the decision of the district court and hold that disbursements from spendthrift trusts, in certain limited circumstances, may be garnished to enforce court orders or judgments for alimony before such disbursements reach the debtor-beneficiary. We also hold that an order or judgment for attorney’s fees awarded incident to the divorce or the enforcement proceedings may be collected in the same manner.”-524Rule: “[W]e conclude that in these types of cases the restraint of spendthrift trusts should not be an absolute bar to the enforcement of alimony orders or judgments.”-526“This enforcement alternative should be allowed only as a last resort.”-526“We further limit this right of garnishment to disbursements that are due to be made or which are actually made from the trust.”-526“We agree that the continuing aspect of such orders may be sustained . . . as necessary to secure payment of alimony. It should be remembered, however, that a continuing garnishment against a spendthrift trust . . . is also a ‘last resort’ remedy that is available only when the traditional methods of enforcing alimony arrearages are not effective.”-527

g. “[A] few courts have held that a spendthrift trust is not subject to garnishment for payment of claims for alimony or child support. The result in Bacardi, however, represents the great weight of authority.”-528

h. “In general, a provider of necessary services may enforce her claim against the beneficiary’s interest in a spendthrift trust. Similarly, if a trust beneficiary hires a lawyer to protect his interest in a spendthrift trust, the lawyer has a claim against the trust proceeds.”-528

i. Restatement of Trusts (Second) § 157 provides: “Although a trust is a spendthrift trust or a trust for support, the interest of the beneficiary can be reached in satisfaction of an enforceable claim against the beneficiary, (a) by the wife or child of the

78

Page 79: Wills, Trusts

beneficiary for support, or by the wife for alimony; (b) for necessary services rendered to the beneficiary or necessary supplies furnished to him; (c) for services rendered and materials furnished which preserve or benefit the interest of the beneficiary; (d) by the United States or a State to satisfy a claim against the beneficiary.”-528

j. “A number of states have enacted statutes purporting to give creditors the right to reach a debtor’s interest in a spendthrift trust to the extent that the debtor’s interest exceeds the debtor’s needs for education and support.”-528

k. “In most jurisdictions, a trust settlor must include a spendthrift provision to insulate trust interests from creditor claims. In New York, however, all trusts are spendthrift unless the trust instrument expressly confers on the trust beneficiary a power to transfer rights under the trust.”-528

l. “[T]he bankruptcy act insulates spendthrift trusts from claims by the trustee in bankruptcy. 11 U.S.C. § 541(c)(2) provides: ‘A restriction on the transfer of a beneficial interest of the debtor in a trust that is enforceable under applicable nonbankruptcy law is enforceable in a case under this title.’”-529

m. “[C]ourts and legislatures . . . uniformly hold that a settlor may not create a spendthrift trust for his own benefit. If a settlor does attempt to create a ‘self-settled’ spendthrift trust, creditors are entitled to reach the settlor-beneficiary’s interest.”-529

n. “Similarly, if two people create reciprocal spendthrift trusts for each other’s benefit, both trusts will be treated as self-settled, and the spendthrift provisions will be ineffective.”-530

o. “More than a half-dozen countries permit settlors to create valid self-settled spendthrift trusts as a way to compete for foreign investment. That is, these countries promise ‘rich’ prospective debtors – often lawyer sand physicians concerned about the prospect of malpractice liability – that they can free themselves of the claims of prospective judgment creditors.”-530

4. Planning for the Costs of Institutional CareCohen v. Commissioner—S. Jud. Ct. of Mass., 1996

Issue: Whether individuals in nursing homes who have sufficient financial means, via trust income, to support themselves, may preserve their trust assets, and at the same time qualify for Medicaid, which is then used as a means of satisfying their financial obligations to medical or nursing institutions? NORule: “[I]f, in any circumstances any amount of money might be paid to a beneficiary, the maximum of such amount is deemed to be available to the beneficiary.”-534“[W]e interpret the statute to define what is an MQT. . . . And that is any trust established by a person (or that person’s spouse) under which that person may receive any payments.”-535“[I]f there is a peppercorn of discretion [on the part of the trustee], then whatever is the most the beneficiary might under any state of affairs receive in the full exercise of that discretion is the amount that is counted as available for Medicaid eligibility.”-536“It is true that a trust might be written to deprive the trustee of any discretion (for instance allowing the payment only of income) and that such a limitation would be respected.”-538

a. Rule 1.2(d) of the MRPC provides: “A lawyer shall not counsel a client to engage, or assist a client, in conduct that the lawyer

79

Page 80: Wills, Trusts

knows is criminal or fraudulent, but a lawyer may discuss the legal consequences of any proposed course of conduct with a client and may counsel or assist a client to make a good faith effort to determine the validity, scope, meaning or application of the law.”-541

b. “Medicaid regulations invariably require consideration of the assets of both partners to a marriage in determining whether either spouse is eligible for Medicaid benefits.”-542

c. “The Medicaid statute does provide limited protection to the community spouse. Although the resources of both spouses are considered in determining eligibility for Medicaid, after one spouse is institutionalized, the statute provides the community spouse some protection against claims by the state. The statute precludes imposition of a lien on the couple’s home . . ., and provides that during periods of institutionalization, the community spouse’s income shall not be deemed available to the institutionalized spouse. . . . In particular, if trust income is paid solely to the community spouse, the income is not treated as available to the institutionalized spouse.”-542

d. “In a number of cases, as in the case of Kokoska, when disabled persons have received tort recoveries of settlements, persons acting on their behalf have attempted to create trusts to supplement public assistance, while preserving principal for estate beneficiaries. Courts have consistently thwarted such efforts, treating the trusts as if they had been created by the beneficiary.”-542

e. “Someone other than the settlor-beneficiary can, however, create a supplemental needs trust with her own assets. Thus, parents of a developmentally disabled adult child can create such a trust and still preserve the child’s eligibility for government programs.”-543

f. “[I]f a person of means becomes disabled – of if a person collects a tort judgment or a settlement – the statute permits establishment of a supplemental needs trust to supplement state-supplied benefits, so long as the state succeeds to the trust principal (or the amount of state payments for medical assistance) when the beneficiary ultimately dies.”-543

XIX. The Government’s Share – A Brief Introduction to Estate TaxationA. Introduction

1. “Since 1916, . . . a federal estate tax has been in effect in the United States.”-430

2. “[U]nder the current statute, estates in excess of $3,000,000 are taxed at a marginal rate of 55 percent, while most of the middle class escapes estate taxation altogether. In 1998, estates under $625,000 are, in effect, not subject to the tax, and that amount is scheduled to increase to $1,000,000 by 2006.”-430

3. “In addition to the federal estate tax, most states have death or inheritance taxes of one sort or another. For the very wealthy, those taxes generally pale in significance when compared with the federal estate tax. For the less wealthy, those taxes may ultimately be more significant, because smaller estates may be subject to the tax.”-431

80

Page 81: Wills, Trusts

B. A Bit of History1. In 1931, Congress enacted a gift tax to reduce opportunities for tax

avoidance.-4312. “In 1976, . . . Congress reformed the estate and gift tax system by

‘unifying’ the federal estate and gift tax – substituting a ‘unified credit’ against the estate and gift taxes for the prior separate exemptions, and creating a common rate structure for the two taxes. At the same time, Congress enacted the first ‘generation-skipping tax,’ designed to prevent very wealthy people from avoiding a generation of estate taxation by devising property to their grandchildren or other remote descendants instead of to their children.”-431

3. In 1981, the “maximum marginal [tax] rate was reduced to 55 percent, while the unified credit essentially eliminated the estate tax for people with less than $600,000 in assets. Perhaps the most significant structural reform in 1981 was the enactment of an unlimited marital deduction. . . . After 1981, no tax is paid by an estate that passes entirely to a surviving spouse. As a result, the estate tax now views married couples as a unit, and no taxes are owed until property passes out of the unit’s hands. Finally, the 1981 statute increased the annual gift-tax exclusion from $3000 to $10,000; a donor may give $10,000 each year to as many individuals as she chooses without incurring any gift tax.”-432

4. “In 1986, Congress enacted a ‘new and improved’ tax on generation-skipping transfers. This tax applies only to the most wealthy segment of society, as each taxpayer is entitled to a $1 million exemption.”-432

5. Through the Taxpayer Relief Act of 1997, “Congress increased, in phases, the size of an estate that would pass free of tax – from $625,000 in 1998 to $1,000,000 in 2006. Congress also ‘indexed’ the generation-skipping transfer tax exemption, so that the $1,000,000 exemption will rise with inflation. In addition, Congress enacted an exclusion that permits owners of family-owned businesses to escape estate taxes on up to $1,300,000 of the value of those businesses.”-432

C. The Mechanics of the Federal Estate Tax1. “The federal estate tax is designed to be a once-a-generation tax on

transmission of wealth. The tax is imposed at death, and it is a progressive tax: decedents who die with taxable estates of less than the ‘exclusion amount’ ($625,000 in 1998, rising to $1,000,000 by 2006) incur no tax liability at all, while decedents with extraordinarily large estates will be taxed at a rate of 55 percent.”-432

2. “To compute estate tax liability, we must proceed in three steps:”a. Start by computing the tax base – what is subject to tax? . . . [T]he tax

base includes the taxable estate, together with all taxable gifts made during decedent’s lifetime. . . . Under section 2033 of the Code, all property owned by decedent at death is included in decedent’s gross estate, but sections 2035 through 2044 include a variety of other property interests that may not pass through decedent’s probate estate, including life insurance proceeds, . . . employee death benefits, and the decedent’s interest in joint tenancies. Once we have computed decedent’s gross estate, sections 2053 through 2056 provide deductions against the gross estate – most significantly the marital deduction and the charitable deduction. . . . Finally, section 2001(b)(2) directs us to subtract from the taxable estate the gift tax payable during

81

Page 82: Wills, Trusts

decedent’s lifetime. The subtraction provision is designed to avoid double taxation.”-432

b. “Next, proceed to section 2001(c), which gives us the tax on that base.”-432

c. “Proceed to section 2010(a), which instructs us to allow a credit against the tax we have computed in steps (1) and (2). The amount of the credit is specified in 2010(c), set forth below. . . . The basic scheme is this – the credit available to a decedent who dies in a given year is precisely equal to the tentative tax that would be due on the ‘exclusion amount’ for that year. . . . The effect of the credit, then, is to permit an estate equal to the exclusion amount to pass tax-free, and to tax all transfers in excess of that amount at marginal rates beginning at 37 to 41 percent, depending on decedent’s year of death.”-433

3. Example: “Now let’s examine the statute in operation: Example 1: Decedent dies in 1999 with a taxable estate of $1,200,000, and has made no gifts during his lifetime. What is the estate tax due on that estate? Solution: We start with the $1,200,000 base, and look to the table. The table tells us that the tax on $1,200,000 base is $345,800 (the tax on $1,000,000), plus 41% of the balance over $1,000,000. The balance over $1,000,000 is $200,000. 41% of $200,000 is $82,000. The total tax, then, is equal to $427,800 ($345,800 plus $82,000). Now, subtract the 1999 unified credit of $211,300. The tax payable is $216,500.”-435

D. The Annual Gift Exclusion1. “Section 2503(e) explicitly excludes payments made as tuition to

educational institutions, and payments made to providers of medical care. Note that to qualify for the exclusion, the payments must be made directly to providers; if your mother pays you $20,000 to reimburse you for amounts you paid to your law school, your mother has made a gift that does not qualify for the tuition exclusion.”-435

2. Section 2503(b) provides: “In the case of gifts (other than gifts of future interests in property) made to any person by the donor during the calendar year, the first $10,000 of such gifts to such person shall not . . . be included in the total amount of gifts made during such year.”-436

3. “Example 2: Jones has three children, five grandchildren, and assets worth $1,000,000. How much estate tax would be saved if Jones gave $10,000 to each of his children and grandchildren in each of the three years before his 1999 death? Solution: If Jones made no gifts, and died with an estate worth $1,000,000, the estate tax would be $134,500 ($345,800 minus the unified credit of $211,300). If Jones gave away $80,000 during each of the three years before his death, for a total of $240,000, his estate would be reduced to $760,000. The estate tax would be $59,400, computed as follows: start by adding $248,300 [the tax on $750,000] to $3,900 [39% of $10,000, the estate’s excess over $750,000]. The total is $252,200. Then subtract the unified credit of $211,300, for an estate tax of $40,900. Hence, Jones would save $93,600 by making the lifetime gifts.”

E. Integrating the Gift Tax and the Estate Tax1. “[A] donor does not have to file a gift tax return for any gift that qualifies

for the $10,000 annual exclusion, or for the tuition and medical care exclusion. For other gifts, the donor must file a return, reporting the taxable gift. The donor will, however, apply part of the unified credit

82

Page 83: Wills, Trusts

against the gift tax due, and will not therefore be obligated to pay any gift tax until the donor has used up the entire unified credit.”-437

2. “[E]ach time the donor files a gift tax return, the donor must report the cumulative total of her taxable gifts. She must then compute the tax on that cumulative total. (Of course, she will then subtract from the tax due any tax payments made with respect to prior gifts). Each gift is effectively taxed only once, and the donor, not the donee, is liable for the gift tax.”-437

3. “First, a donor does not become liable for any gift tax until the donor has used up all of the donor’s unified credit. Second, in spite of the ‘unification’ of the estate and gift taxes, a significant advantage to lifetime gifts remains: the amount of gift tax actually paid on lifetime gifts is excluded from the estate tax base. Put in tax jargon, the gift tax is ‘tax exclusive’ while the estate tax is ‘tax inclusive.’”-439

4. “Brown is deciding whether to make a lifetime gift. Suppose Brown has $150,000 available to give. Case 1: Brown makes a lifetime gift of $100,000 to his daughter. Brown pays tax of $50,000, - 50% of the value of the gift – and the daughter receives $100,000. Case 2: Brown does not make a lifetime gift. Instead he retains the $150,000 in his estate. His estate will be liable for $75,000 (50%) in additional estate tax, and the daughter, if she is the estate beneficiary, receives only $75,000.”-440

5. “Section 2035(b) of the Code mandates that gift taxes paid on gifts made within three years of death be added back into decedent’s estate for purposes of computing the estate tax. Second, most people want or need their assets during their own lives; lifetime gifts diminish the security they may want during their old age. Third, there is a significant income tax advantage to passing property through one’s estate rather than by inter vivos gift: the ‘stepped-up basis.’ The donor’s cost basis travels with the gift when the donor makes an inter vivos gift; by contrast, when a testator passes an asset through the estate, the estate beneficiary’s basis is the value of the asset at the time of transfer. . . . The stepped-up basis is a significant tax advantage, and often makes it preferable to pass assets through one’s estate rather than by inter vivos gift.”-440

F. The marital Deduction1. “The estate tax is treated as a tax on transmission of wealth between

generations; Congress does not require taxation of the husband’s estate at his death, only to have the property taxed again at the wife’s death. . . . Section 2056(a) of the Code provides: ‘For purposes of the tax imposed by section 2001, the value of the taxable estate shall, except as limited by subsection (b), be determined by deducting from the value of the gross estate an amount equal to the value of any interest in property which passes or has passed from the decedent to his surviving spouse, but only to the extent that such interest is included in determining the value of the gross estate.’”-441

2. “The marital deduction permits the married couple to avoid taxation on transfers to each other, whether during their lifetimes or at death, and also to defer taxation until the death of the surviving spouse.”-441

3. “To minimize estate taxation, the married couple will typically use the marital deduction in conjunction with the unified credit. By making full use of both the deduction and the credit, a married couple can avoid estate

83

Page 84: Wills, Trusts

taxation altogether so long as their joint assets do not exceed $1.25 million in 1998 (and as much as $2,000,000 by 2006).”-441

G. Note: Gifts Between Spouses1. “Just as a disposition to a spouse by will qualifies for the marital

deduction, a gift by one spouse to the other generates no tax consequences. Section 2523(a) allows a deduction for the full value of gifts made between spouses. Hence, if one spouse transfers assets to the other in order to allow both spouses to take advantage of the unified credit, the transfer has no gift tax consequences.”-443

2. “[I]f one spouse makes a gift of $20,000, the gift will be treated as two $10,000 gifts, one by each spouse, so long as the other spouse consents. . . . Hence, if one spouse owns the bulk of the couple’s assets, that spouse’s $20,000 gifts fully qualify for the annual exclusion (so long as the other spouse agrees and does not make any gifts to the same donee).”-443

XX. Trusts (Continued)A. Minimizing Taxation

1. Inter Vivos Trustsa. “[I]nter vivos trusts create few tax advantages, and the primary

motivations for creating them are usually non-tax concerns about management of trust property and protection of beneficiaries.”-543

b. “Revocable inter vivos trusts provide little opportunity for tax savings. The income of the trust is taxed to the settlor, and on the settlor’s death, the assets are included in his estate for estate tax purposes as though the settlor owned the trust property outright.”

c. “So long as the settlor of an irrevocable trust parts with all interest and control over the transferred property, . . . creation of the trust will be taxed just like an outright gift of the property, with one exception. Creation of the trust will be subject to the gift tax, and the trust will not be an asset of the settlor’s estate at death. Nevertheless, because only present interests in property qualify for the annual exclusion from gift tax, transfers in trust will not generally qualify for the annual exclusion. Section 2503(c) allows the annual exclusion for certain trusts created for the beneficiary of a minor. A judicial rule has developed which generously allows the annual exclusion for transfers to a ‘Crummey Trust’ . . . .”-544

d. “Irrevocable trusts do provide the opportunity for limited income tax savings. If the beneficiary is in a lower tax bracket than the settlor, some amount of income tax can be saved by transferring property into trust. If, however, the beneficiary is a child under the age of 14, the beneficiary’s income will be taxed at her parents’ rate, resulting in no tax savings.”-544

e. “Tax avoidance, then, rarely provides the sole justification for creation of inter vivos trusts.”-544

f. Crummey Trusts(i) “Can the property owner simply create $10,000 irrevocable

trusts for each of the beneficiaries, postponing the beneficiaries’ right to trust principal until after the property owner’s death? The answer is no – unless the settlor takes care in creating the trusts.”-544

84

Page 85: Wills, Trusts

(ii) “Section 2503(b) of the IRC provides that the annual gift exclusion shall apply ‘in the case of gifts (other than gifts of future interests in property).’ That is, if a property owner makes a gift of a future interest, the gift will be taxed even if its value is below the amount of the annual exclusion.”-544

(iii) “The statute does include an exception permitting a property owner to give a future interest to a minor and still qualify for the annual exclusion – but only if the income is available for use by the minor before he reaches the age of 21, only if the principal becomes wholly available to the minor when he reaches age 21, and only if the principal will be payable to the minor’s estate if the minor dies before the age of 21.”-544

(iv) A Crummey trust is one in which “a trust . . . gives the ultimate beneficiary the right to withdraw the property owner’s annual contribution to the trust for a limited period – say, 30 days – upon which the right to withdraw lapses[.] The gift is arguably not a gift of a future interest, because the beneficiary has an absolute right to withdraw the money during the year in which the ‘gift’ is made. But the limited time for withdrawal, together with the limited savvy of the beneficiary, makes it extremely unlikely that the money will actually be withdrawn. Moreover, the property owner has a stick to make sure the beneficiary doesn’t withdraw the money: if the beneficiary withdraws this year’s shares, there won’t be a share next year.”-545

(v) After Crummey v. Commissioner, “the IRS issued Rev. Rul. 73-405, which approved the use of the Crummey device to obtain the benefit of the annual exclusion while limiting the beneficiary’s practical access to the proceeds of the gift.”-545

Estate of Kohlsaat—U.S. Tax Ct., 1997Issue: Whether, “in the computation of petitioner’s Federal estate tax, decedent’s inter vivos transfer of property to an irrevocable trust is eligible under section 2503(b) for the annual gift tax exclusion with respect to each of 16 contingent beneficiaries of the trust”? YESHolding: “For the reasons stated above, the contingent beneficiaries’ unrestricted rights to demand immediate distributions of trust property are to be treated as present interests in property. Decedent’s transfer of the commercial building to the trust qualifies for 16 annual gift tax exclusions under section 2503(b) with regard to the present interests of the 16 contingent beneficiaries therein.”-548Rule: “The annual exclusion provides that gifts made to beneficiaries during a calendar year shall be excluded from taxable gifts to the extent they do not exceed $10,000 per beneficiary per year. . . . Gifts qualifying for the annual exclusion are not counted in the computation of an estate’s Federal estate tax liability.”-547“Only gifts of present interests in property qualify for the annual gift tax exclusion. Gifts of future interests in property . . . do not qualify for the annual exclusion.”-547“Generally, interests in property qualify as present interests in property where they represent the unrestricted right to immediate use, possession, or enjoyment of property or income from property.”-547“Where trust beneficiaries, including minor and contingent beneficiaries, are given unrestricted rights to demand immediate distributions of trust property, the beneficiaries generally are treated, under section 2503(b), as possessing present interests in property.”-547

g. Notes(i) In Estate of Holland, T.C. Memo, the Tax Court indicated that

“an express agreement would disqualify the withdrawal rights,

85

Page 86: Wills, Trusts

but [found], on the facts of that case, that there had been no express agreement.”-548

(ii) “[M]any lawyers recommend that a Crummey trust be funded, at least in significant part, with liquid assets.”-549

(iii) “Notice is critical in the Crummey trust scheme; the holder of the right to withdraw must receive adequate notice of the withdrawal right, and must enjoy a reasonable time to exercise that right once notice has been received. The beneficiary should always receive notice in writing, and, for evidentiary purposes, the beneficiary should be required to acknowledge receipt of the notice. The IRS has issued a number of private letter rulings indicating that a 30-day withdrawal period – after receipt of the notice – suffices to qualify trust contributions for the annual gift tax exclusion.”-549

2. Testamentary Trusts: More on Marital Deduction Planning, and an Introduction to the Generation-Skipping Taxa. “Section 2056(a) . . . provides that property which passes to decedent’s

surviving spouse shall reduce the decedent’s taxable estate. Section 2010(a) provides each estate with a unified credit, and section 2010(c) provides for a computation of the amount of the unified credit, depending on the year of death. . . . The effect of the unified credit is to insulate from estate taxation the first $650,000 of the estate of any decedent who dies in 1999.”-549

b. “Any tax due on the couple’s combined wealth will be postponed until the death of the surviving spouse. If the survivor’s taxable estate turns out to be less than $650,000, then no tax will be due at all. But what if the survivor’s estate proves to be more than $650,000? In that event, there will be a tax due on the death of the survivor. . . . If all of that spouse’s property is left outright to the surviving spouse, the couple has made no use of the unified credit in the estate of the first spouse to die because the marital deduction reduces the taxable estate to zero.”-550

c. “Example 1: Ms. Smith has $1,500,000 in assets; Mr. Smith, her husband, has none. Ms. Smith’s will leaves all of her property to Mr. Smith. Ms. Smith dies in January 1999; Mr. Smith dies in December 1999. What estate tax is due and when? Solution: Ms. Smith’s estate qualifies for the marital deduction. No tax is due (and hence, Ms. Smith does not use her unified credit). At Mr. Smith’s death, the estate tax on his $1,500,000 estate is $555,800, minus the $211,300 unified credit, for a total tax due of $344,500.”-550

d. “Example 2: Same facts as Example 1, except that Ms. Smith’s will leaves $650,000 to her daughter, and the rest to Mr. Smith. What estate tax is due and when? Solution: Ms. Smith’s taxable estate is equal to $650,000 (the remaining $850,000 qualifies for the marital deduction). The estate tax due of $211,300 is entirely offset by Ms. Smith’s unified credit. At Mr. Smith’s death, the tax on his $850,000 taxable estate (remember, $650,000 passed to the daughter) is $287,300, minus a unified credit of $211,300, for a total tax of $76,000.”-550

86

Page 87: Wills, Trusts

e. “In examples 1 and 2, then, the Smiths would save $268,500 in estate taxes, by making full use of the unified credit and marital deduction.”-550

f. “When married couples seek tax advice, two situations typically arise: (1) Each spouse wants to give the other spouse all of the couple’s property, and to trust the surviving spouse to dispose of whatever is left at the survivor’s death. This situation is common in the so-called ‘traditional’ family, where both spouses share the same children. . . . (2) The decedent wants to provide for the surviving spouse, but to control the ultimate distribution of the property after the surviving spouse’s death. This situation is common in a multiple-marriage family, where decedent’s spouse is not necessarily the parent of decedent’s children, and where the spouse may have children who are not also the decedent’s children.”-551

g. Maximizing the Surviving Spouse’s Control Over the Couple’s Property(i) “Suppose . . . settlor understands that his own foresight is

limited, and wants to entrust to someone else the power to decide how much (if anything) particular beneficiaries should receive. Settlor can confer on that person a Power of Appointment over the trust property.”-552

(ii) “Example 3: Settlor creates a trust, naming his wife as trustee, directing wife to pay income to herself during life, and conferring on the wife a power to appoint the trust principal, by will, among the settlor’s three children. The wife has a power of appointment, and may, in her will, provide that the trust principal should be divided equally among the children, but may also provide for unequal distribution.”-552

(iii) “In Example 3, the wife’s power of appointment is a special power of appointment because the class of people to whom she is permitted to appoint excludes herself. She may only appoint a limited class – settlor’s children. By contrast, if settlor gave his wife a power to appoint to anyone – including herself or her own estate – we would say that settlor had given his wife a general power of appointment.”-552

(iv) “If a decedent creates a trust, and gives his spouse a general power of appointment over the trust principal, that trust principal will be included in the spouse’s taxable estate. . . . By contrast, if a decedent gives his spouse only a special power, as in Example 3, the trust principal will not be included in the spouse’s taxable estate.”-552

(v) “Decedent can create a trust, and give his spouse a power to appoint the remaining principal among his children . . . . The trust property will be included in the decedent’s estate, using his unified credit, but will not be included in his spouse’s estate. Decedent will have achieved most of his objectives by use of the trust – often called a ‘credit shelter trust’ or a ‘bypass trust’ because the trust ‘bypasses’ the spouse’s estate.”-552

(vi) “If . . . the spouse is decedent’s principal concern, but the decedent is concerned about the spouse’s ability to manage

87

Page 88: Wills, Trusts

property, decedent might, instead, want to devise property in trust, giving the trustee broad powers to invade principal for the spouse’s benefit, and, giving the spouse a general power to appoint the trust property at her death. Such a trust would clearly qualify for the marital deduction, and is often called a marital deduction trust.”-554

h. Limiting the Spouse’s Power to Distribute Trust Assets(i) The qualified terminable interest property trust is known as

a QTIP trust.(ii) “In 1981, Congress created the qualified terminable interest

property provision embodied in section 2056(b)(7). The statute permits a decedent to create a trust which gives her surviving spouse only a life interest, and to have the trust qualify for the marital deduction. So long as the trust mandates annual (or more frequent) payment of income to the surviving spouse, and assures that no one else will acquire any power to invade the trust principal during the spouse’s lifetime, decedent’s executor may elect to have the trust qualify for the marital deduction. Section 2044 assures that QTIP property will be taxed at least once, by providing that if property qualifies for the marital deduction under section 2056(b)(7), the value of that property is included in the estate of the surviving spouse.”

(iii) “For the decedent who wants to provide her spouse with a lifetime interest in her property, while assuring that the principal is ultimately distributed to her children, a typical estate plan will include two trusts: first, a QTIP trust designed to take advantage of the marital deduction, and second, a credit shelter trust designed to assure that decedent uses her full unified credit. . . . So long as decedent does not confer on the spouse too broad a power to consume or dispose of the trust principal (which would cause the trust to be included in both decedent’s estate and the spouse’s estate), she is free to structure the credit shelter trust to achieve non-tax objectives.”

i. Generation-Skipping Trusts(i) “Generation-skipping trusts create the potential for

enormous tax savings.”-559(ii) “In the Tax Reform Act of 1986, Congress and the Reagan

administration, by enacting the Generation-Skipping Transfer Tax, significantly curtailed the tax benefits of generation-skipping trusts.”-560

(iii) “The thrust of the tax is this: in cases like Example 7, even though no assets pass through decedent’s child’s estate, we must treat the distribution of trust principal as if the principal were distributed through the child’s estate. That is, the generation-skipping tax is designed to assure that wealth is taxed once each generation.”-560

(iv) “[U]ntil 1997, each individual transferor enjoyed a $1,000,000 exemption from the Generation-Skipping Transfer Tax. The Taxpayer Relief Act of 1997 retains the

88

Page 89: Wills, Trusts

exemption, but indexes it for inflation. In practice, this means that an individual can create, in 1997 dollars, a $1,000,000 trust, providing income to children for life and ultimate distribution to grandchildren, without incurring generation-skipping tax liability. The result will be to insulate that $1,000,000 from tax in the estates of decedent’s children – a considerable tax savings if the children are not likely to need the money. Since each spouse in a married couple can use the exemption, the couple can insulate $2,000,000 (again, indexed for inflation) from tax in the estates of their children.”-560

D. Trust Modification and Termination1. Modification or Termination by Direction of the Trust Settlor

a. “The trust settlor – who created the trust in the first place – has considerable power over trust termination and modification. First, the settlor can, by appropriate language in the instrument creating the trust, reserve to herself the power to revoke or modify the trust. Second, the trust instrument will almost invariably specify a time for trust termination, and will include directions for distribution of trust assets upon termination.”-561

Connecticut General Life Ins. Co. v. First National Bank of Minneapolis—S. Ct. of Minn., 1977Issue: Whether the trial court erred in holding that that a revocable trust naming the settlor’s ex-wife and three children as beneficiaries was revoked by a subsequently executed will, which purported to “supercede and cancel any previous wills or trusts established” by him? NORule: “It is the general rule that where a settlor reserves the power to revoke a trust by a transaction inter vivos, as for example by notice to the trustee, he cannot revoke the trust by his will.”-562

2. Modification or Termination by ConsentAdams v. Link—S. Ct. of Errors of Conn., 1958

Issue: Whether the trial court erred in refusing to approve an agreement between the beneficiaries of a trust that would have altered the provisions in the settlor’s will? NOHolding: “Here the provisions of the will itself are being drastically changed so as to abolish a trust contrary to our rule. This cannot be done.”-567Rule: “Our rule as to the right of the beneficiaries of a testamentary trust to have it terminated has been set forth in a number of cases . . . .”-565“The rule has also in effect been applied to the right of the beneficiaries to terminate an inter vivos trust.”-565“[A] testamentary trust may be terminated only by a decree of a court of equity, regardless of any stipulation by all parties in interest.”-565“Conditions precedent which should concur in order to warrant termination of a testamentary trust by judicial decree are (1) that all the parties in interest unite in seeking the termination, (2) that every reasonable ultimate purpose of the trust’s creation and existence has been accomplished, and (3) that no fair and lawful restriction imposed by the testator will be nullified or disturbed by such a result.”-565

American National Bank of Cheyenne v. Miller—S. Ct. of Wyo., 1995Issue: Whether “the beneficiaries of a trust can compel its early termination on the grounds it lacks any remaining material purpose”? YESWhether “a trustee (who is not also a beneficiary) has standing to challenge the early termination of a trust, and whether the cost of a supersedeas bond was properly allocated to the trustee individually”? YRule: “For a party to have standing to sue means that he has sufficient stake, in an otherwise justiciable controversy, to obtain a judicial resolution.”-569“A trustee, however, acts on behalf of both the beneficiaries and the grantor of the trust. A fundamental duty of a trustee is to carry out the terms of the trust.”-569

89

Page 90: Wills, Trusts

“A trustee has an obligation to defend the trust in order to carry out the material purposes of the trust.”“The Restatement (Second) of Trusts § 337 (1959) summarizes a general rule as to when beneficiaries can compel the early termination of a trust: ‘(1) Except as stated in Subsection (2), if all of the beneficiaries of a trust consent and none of them is under an incapacity, they can compel the termination of the trust. (2) If the continuance of the trust is necessary to carry out a material purpose of the trust, the beneficiaries cannot compel its termination.’ We find the principles of § 337 to be sound, and adopt them as the law of Wyoming.”-570

a. Notes(i) “[T]here is universal agreement that the beneficiaries may not

compel termination of a spendthrift trust, because termination would interfere with a material purpose of the settlor. Thus, in New York, where all trusts are spendthrift unless the settlor provides otherwise, the beneficiaries may not consent to trust termination once the settlor has died – unless the settlor has explicitly made the trust non-spendthrift.”-572

(ii) “If all of the beneficiaries of a trust consent to termination, the only reason for keeping the trust in effect is deference to the wishes of the settlor. If, however, the settlor consents to the termination, the only reason for maintaining the trust disappears. See Restatement (Second) of Trusts, section 338(1): ‘If the settlor and all of the beneficiaries of a trust consent and none of them is under an incapacity, they can compel the termination or modification of the trust, although the purposes of the trust have not been accomplished.’”

(iii) “[I]n New York, for instance, a court may terminate a trust so long as all living beneficiaries consent. . . . In other jurisdictions, courts may appoint a guardian ad litem to represent the interests of unborn beneficiaries, and then require consent of the guardian before permitting trust termination.”-574

(iv) “The California statute provides that ‘in determining whether to give consent, the guardian ad litem may rely on general family benefit accruing to living members of the beneficiary’s family as a basis for approving a modification or termination of the trust.’”-575

3. Modification or Termination Without Consent of All Beneficiariesa. “What happens, . . . when a court becomes convinced that enforcing

the trust according to its terms would frustrate settlor’s intentions?”-575

b. MistakeSimches v. Simches—S. Jud. Ct. of Mass., 1996

Issue: Whether the settlor and sole trustee of an irrevocable trust may reform the trust by changing the remaindermen so as to conform to the settlor’s intentions with respect to tax avoidance? YESRule: “In determining whether a mistake has been made, we must determine the settlor’s intent and then decide whether the questioned provision satisfies that intent . . . . It is firmly established that considering tax implications is proper when inferring intent.”-576

(i) “Reformation for mistake is not limited to the tax context, and courts are willing to admit extrinsic evidence to establish mistake. Nevertheless, application of the mistake doctrine is particularly appropriate in the tax context because the settlor’s tax objectives are often clear, and the

90

Page 91: Wills, Trusts

language used in the instrument could have been designed for no purpose other than achieving those objectives.”-577

c. Impracticability of Accomplishing Trust Purposes(i) Restatement (Second) of Trusts § 335 provides: “If the purposes for which

a trust is created become impossible of accomplishment or illegal, the trust will be terminated.”-578

(ii) “In California, a statute gives the trustee authority to terminate a trust – without even obtaining court consent – if the trust principal falls below $20,000.”-578

d. Modification or Termination to Provide for Needy Income Beneficiaries: New York’s EPTL Section 7-1.6(B): “Notwithstanding any contrary provision of law, the court having jurisdiction of an express trust, hereafter created or declared, to receive income from property and apply it to the use of or pay it to any person, unless otherwise provided in the disposing instrument, may in its discretion make an allowance from principal to any income beneficiary whose support or education is not sufficiently provided for, whether or not such person is entitled to the principal of the trust or any part thereof; provided that the court, after a hearing on notice to all those beneficially interested in the trust in such manner as the court may direct, is satisfied that the original purpose of the creator of the trust cannot be carried out and that such allowance effectuates the intention of the creator.”-578-79

E. Charitable Trusts1. Introduction

a. “[S]uppose a philanthropist wants to impose a continuing obligation to use her money for a particular philanthropic purpose: scholarships for law students, or maintenance of a soup kitchen. How can the philanthropist assure that her instructions will continue to be followed?”-579

b. “The private express trust . . . will not help our philanthropist, for two reasons. First, as we have seen, a private express trust must have identifiable beneficiaries – people who have a right to enforce duties imposed on the trustee. . . . Second, as we shall see when we discuss the Rule Against Perpetuities, private express trusts may not endure forever. If the philanthropist wants to assure that scholarship funds are permanently available, a private express trust will not accomplish his objective.”-579

c. “If the settlor’s objectives do sufficient social good to be deemed ‘charitable,’ a trust for those charitable purposes will be enforced even without identifiable beneficiaries. Who will enforce the trust obligations? The state, through its Attorney General.”-579

d. “[N]either legislatures nor courts have imposed any limits on the duration of charitable trusts.”-579

2. Tax Incentives for Charitable Givinga. “The federal estate and gift tax laws permit a 100% deduction for gifts

to qualifying charities.”-580b. “If testator has any desire to benefit particular charities, then, the tax

system provides a significant incentive for charitable giving.”-580c. “Moreover, if testator wants to postpone the effective date of the

charitable gift until after the lifetime of a spouse, the Internal Revenue

91

Page 92: Wills, Trusts

Code will oblige. By placing property in trust for the life of a spouse, followed by a charitable remainder and using an ‘annuity trust’ or ‘unitrust,’ the transfer can be made free of federal estate tax. There is a 100% marital deduction for the life interest in the spouse and a 100% charitable deduction for the remainder.”-580

3. Charitable PurposesShenandoah Valley National Bank of Winchester v. Taylor—S. Ct. of App. of Va., 1951

Issue: Whether a will providing that a trustee shall invest and reinvest the trust principle and distribute the net income among the first through third grade students at the John Kerr School of the City of Winchester “to be used by such child in the furtherance of his or her obtainment of an education” creates a valid charitable trust? NOHolding: “Payment to the children of their cash bequests on the two occasions specified would bring to them pleasure and happiness and no doubt cause them to remember or think of their benefactor with gratitude and thanksgiving. That was, we think, Charles B. Henry’s intent. Laudable, generous and praiseworthy though it may be, it is not for the relief of the poor or needy, nor does it otherwise so benefit or advance the social interest of the community as to justify its continuance in perpetuity as a charitable trust.”-586Rule: “If [a settlor’s] dominant intent as expressed was charitable, the trust should be afforded efficacy and sustained.”-582“But on the other hand, if the testator’s intent as expressed is merely benevolent, though the disposition of his property be meritorious and evince traits of generosity, the trust must nevertheless be declared invalid because it violates the rule against perpetuities.”-582“A charitable trust is created only if the settlor properly manifests an intention to create a charitable trust.”-582“In the law of trusts there is a real and fundamental distinction between a charitable trust and one that is devoted to mere benevolence. The former is public in nature and valid; the latter is private and if it offends the rule against perpetuities, it is void.”-583“A trust from which the income is to be paid at stated intervals to each member of a designated segment of the public, without regard to whether or not the recipients are poor or in need, is not for the relief of poverty, nor is it a social benefit to the community. It is a mere benevolence – a private trust – and may not be upheld as a charitable trust.”-585

Estate of Robbins—S. Ct. of Cal., 1962Issue: Whether the trial court erred in holding invalid a trust that provided the trustee sole discretion to use the trust income or principal “for the care, comfort, support, medical attention, education, sustenance, maintenance or custody of such minor Negro child or children, whose father or mother, or both, have been incarcerated, imprisoned, detained or committed in any federal, state, county or local prison or penitentiary, as a result of the conviction of a crime or misdemeanor of a political nature”? YHolding: “Assistance to the minor beneficiaries of the trust in this case is a valid charitable purpose.”-2Rule: “Relief of poverty is not a condition of charitable assistance. If the benefit conferred has a sufficiently widespread social value, a charitable purpose exists.”-2“In short, as the word ‘charity’ is commonly understood in modern usage, it does not refer only to aid to the poor and destitute and exclude all humanitarian activities . . . which are maintained to care for the physical and mental well-being of the recipients, and which make it less likely that such recipients will become burdens on society.”-2“It is the purpose for which the property is to be used, however, not the motives of the testator that determines whether a trust is a valid charitable trust.”-2

a. Notes(i) “A philanthropist seeking to make a charitable bequest need

not use the trust form. The philanthropist can instead create a charitable corporation, or make a gift to an existing charitable corporation. . . . As with a charitable trust, the philanthropist

92

Page 93: Wills, Trusts

can impose restrictions on the corporation’s use of the funds. St. Joseph’s Hospital v. Bennett . . . is the leading case establishing that restricted gifts to charitable corporations should be treated, for perpetuities purposes, like charitable trusts.”-587

(ii) “Suppose . . . settlor creates a trust, with the income to be used for the education of her descendants. Is the trust charitable? . . . . No.”-587

(iii) “Trusts for the relief of poverty are charitable.”-588(iv) “Although religion is clearly one of the categories of charity,

courts sometimes find pretexts to invalidate trusts for unorthodox beliefs.”-588

(v) In re Estate of Breeden – “Although a trust to promote the success of a specific political party is not charitable, . . . the promotion of a particular cause or doctrine remains charitable regardless whether it is embraced as well by a political party.”-588

(vi) “The Attorney General in each state has standing to enforce charitable trusts.”-589

4. The Cy Pres DoctrineEstate of Crawshaw—S. Ct. of Kan., 1991

Issue: Whether the application of the cy pres doctrine is appropriate where the settlor created a charitable trust for the benefit of the nursing students at Marymount College, reserving 85% of the trust proceeds for this purpose, but also reserved 15% of the trust proceeds for the benefit of the Salvation Army? YESRule: “In Kansas, a lapsed or void bequest falls into the residuum and will be disposed of by the residuary clause, if one has been provided.”-593“The doctrine of cy-pres permits a court to implement a testator’s intent and save a gift to charity by substituting beneficiaries when the named charitable beneficiary is unable to take the gift. In order for the doctrine to apply, several conditions must be met. First, the gift must be to a charitable organization for a charitable purpose. Second, it must be impossible, impractical or illegal to carry out the donor’s stated charitable purpose. Finally, it must appear that the donor had a general charitable intent. . . . Therefore, when it becomes impossible for the gift to take effect exactly as the donor specified, the court must look for another agent, as nearly like the designated one as possible, that will receive the gift and effectuate the general charitable intent expressed in the will or gift instrument.”-595“The cy pres doctrine should not be applied if the testator has: (1) manifested a specific charitable intent; (2) has anticipated possible failure of the trust; or (3) has made an alternate disposition of the property if the charitable gift should fail.”-595“In determining whether the testator had a general charitable intent, courts may consider all available, admissible evidence, both intrinsic and extrinsic, which is indicative of the testator’s intent. Among the factors which may be considered are the existence of a reversionary or gift-over provision, the existence of a limitation or reservation on the use of the gift, whether the bulk of the estate was bequeathed to charity, and whether specific devises and bequests were made to individuals who would have taken the estate by intestacy.”-597

a. Notes(i) “In general, academic commentators favor broad application of

cy pres.”-601(ii) “Once it becomes clear that settlor’s instructions cannot

feasibly be followed, each cy pres dispute raises two distinct but related questions: first, should the doctrine be applied at all (that is, did settlor have a general charitable intent); second,

93

Page 94: Wills, Trusts

how should the doctrine be applied – who should take the settlor’s funds, and on what terms?”-601

(iii) “[C]ourts use their own judgment in choosing the substitute taker most likely to approximate settlor’s wishes.”-602

Estate of Wilson—N.Y. Ct. of App., 1983Issue: Whether “the equal protection clause of the Fourteenth Amendment is violated when a court permits the administration of private charitable trusts according to the testator’s intent to finance the education of male students and not female students”? NOHolding: “Accordingly, the proper means of continuing the Johnson Trust would be to replace the school district with someone able and willing to administer the trust according to its terms.”-606“Therefore, a deviation from the Wilson Trust’s administrative terms by eliminating the certification requirement would be the appropriate method of continuing that trust’s administration.”-606Rule: “When a court applies trust law that neither encourages, nor affirmatively promotes, nor compels private discrimination but allows parties to engage in private selection in the devise or bequest of their property, that choice will not be attributable to the State and subjected to the Fourteenth Amendment’s strictures.”-602“When a court determines that changed circumstances have rendered the administration of a charitable trust according to its literal terms either ‘impracticable or impossible,’ the court may exercise its cy pres power to reform the trust in a matter that ‘will most effectively accomplish its general purposes.’ . . . In reforming trusts pursuant to this power, care must be taken to evaluate the precise purpose or direction of the testator, so that when the court directs the trust towards another charitable end, it will ‘give effect insofar as practicable to the full design of the testator as manifested by his will and codicil.’”-604“A provision in a charitable trust . . . that is central to the testator’s or settlor’s charitable purpose, and is not illegal, should not be invalidated on public policy grounds unless that provision, if given effect, would substantially mitigate the general charitable effect of the gift.”-605“A testamentary trust will not fail for want of a trustee, . . . and, in the event a trustee is unwilling or unable to act, a court may replace the trustee with another.”-606“When an impasse is reached in the administration of a trust due to an incidental requirement of its terms, a court may effect, or permit the trustee to effect, a deviation from the trust’s literal terms.”-606“The strictures of the equal protection clause are invoked when the State engages in invidious discrimination. . . . Indeed, the State itself cannot, consistent with the Fourteenth Amendment, award scholarships that are gender restrictive. . . .”-606“The Fourteenth Amendment, however, ‘erects no shield against merely private conduct, however discriminatory or wrongful.’”-607“Private discrimination may violate equal protection of the law when accompanied by State participation in, facilitation of, and, in some cases, acquiescence in the discrimination. . . . Although there is no conclusive test to determine when State involvement in private discrimination will violate the Fourteenth Amendment, . . . the general standard that has evolved is whether ‘the conduct allegedly causing the deprivation of a federal right [is] fairly attributable to the state.’ . . . Therefore, it is a question of ‘state responsibility’ and ‘[only] by sifting facts and weighing circumstances can the involvement of the State in private conduct be attributed its true significance.’”-607“[W]hen the State regulates private dealings it may be responsible for private discrimination occurring in the regulated field only when enforcement of its regulation has the effect of compelling the private discrimination.”-607“[W]hen a court applies this trust law and determines that it permits the continued existence of private discriminatory trusts, the Fourteenth Amendment is not implicated.”-607“This court holds only that a trust’s discriminatory terms are not fairly attributable to the State when a court applies trust principles that permit private discrimination but do not encourage, affirmatively promote, or compel it.”-608

XXI. Classification and Construction of Future InterestsA. Classification of Future Interests

94

Page 95: Wills, Trusts

1. Future Interests and Present Interestsa. “A future interest is one that does not become possessory upon its

creation.”-670b. “Even if a future interest will not become possessory for a long time,

the future interest is a legally recognized and enforceable relationship from the moment it is created. It does not spring into existence for the first time on the future date when the owner of the future interest becomes entitled to enjoyment of the property. The word ‘present’ or ‘future’ immediately preceding the word ‘interest’ refers to the time of beneficial enjoyment, not to the existence of the legal relationship.”-670

c. “What rights does this future interest give to C? First, C may bring an action against T for any acts by T that would impair C’s interest . . . .”-671

d. “If the future interest follows a legal life estate in land, the holder of a future interest may bring an action for ‘waste’ if the life estate holder takes actions that impair the value of the future interest. Second, the holder of the future interest, C, may generally sell or otherwise dispose of that future interest even before the interest becomes possessory. Thus, a future interest has a present value to its holder.”-671

2. The Categories of Future Interestsa. The Basic Classification System

(i) “The first step in classifying a future interest is to determine who holds the future interest at the moment the future interest is created. If the future interest is held by the grantor of the property, the future interest must be one of the following three interests: possibility of reverter, right of entry (sometimes called a ‘right of re-entry’ or a ‘power of termination’), or reversion. On the other hand, if the grantor creates a future interest in someone other than himself, the future interest is one of the following two interests: remainder, or executory interest.”-672

b. Future Interests in the Grantor(i) “A possibility of reverter is the future interest remaining in a

grantor who has created a determinable interest – generally a fee simple determinable, but sometimes a determinable life estate or a determinable term of years – in someone else. A right of entry is the future interest remaining in a grantor who has created, in someone else, a possessory interest (fee, life estate, or term of years) subject to a condition subsequent. Generally, all other future interests reserved by a grantor are reversions.”-672

(ii) “[A] determinable interest is generally conceptualized as an interest that will end automatically upon the happening of a particular event.”-673

(iii) “By contrast, an interest subject to condition subsequent is conceptualized as an interest that does not automatically terminate upon the happening of a particular event, but which the grantor may elect to terminate upon the happening of that event.”-673

95

Page 96: Wills, Trusts

(iv) “The supposed difference between the fee simple determinable and the fee simple subject to condition subsequent does not amount to much in practice. [For example], if A never takes any action to reclaim Blackacre, B will continue to use Blackacre for whatever purposes please him; the ‘automatic’ termination will have little practical consequence.”-673

Mahrenholz v. County Board of School Trustees—Ill. App. Ct., 1981Issue: Whether “a transfer of property was effective in light of an Illinois statute barring transfers of both rights of entry and possibilities of reverter”? NOHolding: “We believe that a close analysis of the wording of the original grant shows that the grantors intended to create a fee simple determinable followed by a possibility of reverter.”-674

(v) “[N]either the possibility of reverter nor the right of entry is terribly important in estate planning. Only rarely will a person seeking to transmit wealth use either form of defeasible fee. As a result, when the grantor does create a future interest in herself, that future interest will almost always be a reversion. Finally, if a grantor creates a reversion in herself, her subsequent conveyance of the reversion does not convert the reversion into a remainder; it remains a reversion.”-674

c. Future Interests in Persons Other Than the Grantor(i) “When future interest problems arise in the estate planning

context, the issues generally involve future interests in persons other than the grantor. As we have seen, those future interests are remainders and executory interests. There are four different types of remainders: indefeasibly vested remainders, vested remainders subject to open (or subject to partial divestment, or subject to partial defeasance), vested remainders subject to complete divestment (or subject to complete defeasance), and contingent remainders. Most future interests in persons other than the grantor are remainders . . . .”-675

(ii) Distinguishing Among Remainders(A) “Not all remainders will become possessory estates. Suppose,

for instance, T’s will devises Blackacre ‘to A for life, remainder to B if B survives A, and otherwise to C.’ If B dies before A, B’s interest will not become possessory at all. We call B’s remainder a contingent remainder. How can we tell whether a remainder is contingent? Generally, by asking two questions. One must first ask whether the remainder is held by some ascertained person? If the answer is no, the remainder is contingent. . . . Second, if the remainder is held by an ascertained person, one must ask whether the remainder is subject to a condition precedent. If the remainder is subject to a condition precedent, it will be classified as a contingent remainder even if it is held by an ascertained person. A vested remainder cannot be subject to a condition precedent. A vested remainder can be subject to a condition subsequent. If it is, we say that the remainder is a vested remainder subject to complete divestment.”-675

(B) “If the conditional element is incorporated into the description of, or into the gift to, the remainder-man, then the remainder is

96

Page 97: Wills, Trusts

contingent; but if, after words giving a vested interest, a clause is added divesting it, the remainder is vested [subject to complete divestment].”-676

(C) Example 7: “T devises Blackacre ‘to A for life, remainder to B if B survives A, otherwise to C.’ According to Gray, B has a contingent remainder because the condition – surviving A – appears in the same clause as the grant to B.”-676

(D) Example 8: T devises Blackacre ‘to A for life, remainder to B, but if B fails to survive A, then to C.’ Gray’s formulation would make B’s remainder in this case a vested remainder subject to complete divestment. Why? Because in this case, the survivorship condition comes not in the same clause as the grant to B, but in a clause that follows the grant to B. Another way to say that the clause ‘follows’ the grant to B is that the clause is ‘subsequent’ to the grant to B. Hence, the condition is called a ‘condition subsequent.’ By contrast, if the conditional language had preceded the grant, the condition would generally be called a ‘condition precedent.”-676

Webb. V. Underhill—Ct. of App. of Or., 1994Issue: Whether “the children’s and grandchildren’s interests in the property are vested, and whether resolution of that question involves a factual determination that precludes summary judgment”?Holding: “We hold that the determinative issue in the case, whether alternative beneficiaries’ remainder interests are contingent or vested, is a question of law that is amenable to resolution on summary judgment.”-676Rule: “To maintain an action to partition property, a plaintiff must be a tenant in common, with a vested remainder in the property.”-677“When the language of a will is unambiguous, there is no basis for resorting to extrinsic evidence to ascertain the testator’s intent.”-678“When a life estate is followed by two alternative remainder interests, and ‘the vesting of the second depends upon the failure of the first, and the same contingency decides which one of the two alternative remainders shall take effect in possession,’ both interests are alternative contingent remainders.”-679

(E) “The indefeasibly vested remainder is the easiest remainder to understand. Subject to some unimportant qualifications, a remainder is indefeasibly vested when it is certain to become possessory whenever and however the preceding estates end.”-679

(F) “[T]here are no events which might act to diminish or divest D’s interest, either before or after the interest becomes possessory. This, too, is a characteristic of the indefeasibly vested remainder.”-680

(G) “The vested remainder subject to open or subject to partial divestment is a remainder which, like an indefeasibly vested remainder, is certain to become possessory. But the holder of a vested remainder subject to open, unlike the holder of an indefeasibly vested remainder, may find his share of the property reduced as more beneficiaries become eligible to share the property. Because the class of possible takers is still open, his share may be partially divested by new members of the class.”-680

97

Page 98: Wills, Trusts

(H) “Vested remainders subject to open are most common when the grantor creates a ‘class gift,’ such as a gift ‘to the children of A’ or to the ‘nephews and nieces of B.’”-680

(iii) Distinguishing Remainders from Executory Interests(A) “Executory interests typically arise in two situations. . . . First,

by definition, a remainder cannot cut short, or ‘divest’ a vested remainder in fee simple. If a future interest in someone other than the grantor would operate to cut short a vested remainder in fee, that future interest must be an executory interest. Typically, executory interests follow vested remainders subject to complete defeasance.”-681

(B) “A future interest is an executory interest even if it only partially divests a vested remainder in fee. Thus, if a decedent creates a vested remainder subject to open – ‘to my daughter, for life, remainder to my grandchildren’ – decedent’s unborn grandchildren have an executory interest because their interest would partially divest the interests of living grandchildren.”-681

(C) “Second, executory interests arise when there is certain to be a time gap between the end of the prior estate and the time the future interest will become possessory. For a future interest to be a remainder, the interest must be one that may become possessory immediately upon the natural expiration of a prior estate created in the same instrument. If the interest cannot become possessory until after the ‘gap,’ the interest cannot be a remainder.”-681

(D) “The difference between a contingent remainder and an executory interest is generally of limited importance today. Indeed, in some states, the distinction has been abolished by statute.”-681

B. Construction of Future Interests: Gifts to Individuals1. Introduction

a. “Remember that a will typically has no effect until testator’s death. The beneficiaries of a will have no property interest in testator’s estate until testator’s death. By contrast, when settlor has created a trust – and for this purpose it doesn’t matter whether the trust was created by will or by inter vivos instrument – the remainder beneficiaries of the trust have a property interest (albeit a future interest) from the moment the trust becomes effective.”-683

b. “You should not assume, therefore, that an antilapse statute will save a gift of a future interest to a remainderman who dies before her interest becomes possessory.”-683

2. Should We Imply a Condition of Survival?In re Estate of Ferry—S. Ct. of Cal., 1961

Issue: Whether “the testator intended that Mary Silva must survive the trust beneficiary, Joseph J. Ferry, before her interest could be transmissible to her heirs”? NOHolding: “[A]bsent a clear intent deducible from the words of the codicil or decree we are unwilling to imply a condition that Mary, as executory devisee or alternative contingent remainderman, must survive Joseph, the first-named and primary taker under the instrument.”-686Rule: “Certainly there is no rule of law that a condition precedent of survivorship is implied wherever a gift is subject to another condition precedent.”-685

98

Page 99: Wills, Trusts

“Where the requirement of survival is found to exist, it is usually applicable only to the first name takers. The persons who are the alternative takers need not survive unless such a requirement is specifically expressed.”-685“In a limitation purporting to create a remainder or an executory interest in a person, which is ambiguous as to whether such interest is, or is not, subject to a requirement of survival to some particular time, the absence, with respect to such particular time, or both an alternative and supplanting limitation tends to establish that such interest is free from any requirement of survival to such time whenever such construction results in more interests being disposed of than would otherwise be disposed of by the conveyance in question.”-686“The term ‘heirs,’ or other words of inheritance, are not requisite to devise a fee, and a devise of real property passes all the estate of the testator, unless otherwise limited.”-686

a. “Most often, when a testator or a trust settlor leaves a remainder interest to someone, that someone is a family member.”-686

b. Note on the Presumption in Favor of Early Vesting(i) “Common law courts developed a presumption in favor

of early vesting for several reasons. . . . First, at common law, vested interests were alienable; contingent interests were not. . . . Today, in the large majority of states (although by no means in all), contingent interests are freely alienable, reducing the pressure for a presumption in favor of early vesting. Moreover, to the extent that, today, most property subject to future interests is property held in trust, with a trustee who enjoys power to sell the property, concerns about alienability provide even less support for the presumption in favor of early vesting.”-688

(ii) “Second, . . . the Rule Against Perpetuities invalidates future interests if they do not ‘vest’ within the Rule’s period. As a result, a presumption in favor of early vesting operated to save some number of future interests from invalidity under the Rule. This reason for the presumption in favor of early vesting still exists today – but only in those cases where there is some prospect that an interest might otherwise be invalid under the Rule.”-688

(iii) “Third, vested remainders accelerated into possession upon premature termination of preceding estates; contingent remainders did not. . . . As a result, by construing a remainder as vested, courts could avoid the prospect of undisposed of property. . . . Today, disclaimer statutes generally deal with this problem by treating the disclaiming beneficiary as if she predeceased testator.”-688

(iv) “Finally, vested remainders were not subject to the common law rule that contingent remainders were destructible – a rule abolished virtually everywhere.”-688

c. Susan F. French, Imposing a General Survival Requirement on Beneficiaries of Future Interests: Solving the Problems Caused

99

Page 100: Wills, Trusts

by the Death of a Beneficiary Before the Time Set for Distribution(i) “Although distributing the property to the donee’s

estate presents many attractive features, it is both cumbersome and costly. It is cumbersome because the probate process itself is cumbersome, and to make matters worse, the beneficiary may die long before the property becomes distributable. Too often, the future interest was not recognized as an asset of the beneficiary’s estate, and a probate must be opened or reopened years later to receive and then distribute the property.”-689

(ii) “The process is costly for several reasons. First, the administrative expense of the probate must be paid. Second, the value of the future interest is taxable as part of the beneficiary’s estate. Third, it exposes the property to the claims of the beneficiary’s creditors and the claims of forced or pretermitted heirs.”-689

d. “Transmissible” Remainders and the Estate Tax(i) “[T]he presumption in favor of early vesting can have

disastrous tax consequences for testator’s family if testator’s lawyer isn’t careful about the drafting of future interests.”-689

(ii) “The problem is this: if, at decedent’s death, decedent holds a ‘transmissible’ future interest, that future interest will be included in the decedent’s estate for federal estate tax purposes, even if decedent never enjoyed use or possession of the property during her lifetime. If, on the other hand, decedent does not hold a ‘transmissible’ future interest, the future interest will not be included in decedent’s estate.”

(iii) “How do we determine whether a future interest is transmissible? The question is whether decedent’s interest terminated with his death (in which case it is not taxable as an asset of his estate), or whether he can pass it on to others, in which case it will be taxed in his estate.”-689

(iv) “Not all remainders classified as vested will be considered transmissible. Some remainders may be vested, but not transmissible. . . . Conversely, it is possible for a contingent remainder to be transmissible.”-690

(v) “The general drafting lesson . . . is to avoid transmissible remainders whenever possible.”-691

(vi) “There is, however, one caveat to the general principle that testators should avoid transmissible remainders: beware the Generation-Skipping Transfer Tax. Because GST Tax often has a marginal rate even higher than the estate tax, a testator must be careful, while avoiding transmissible remainders, also to avoid creating ‘skip’ transactions in excess of the exemption

100

Page 101: Wills, Trusts

($1,000,000 in 1997, and subsequently indexed for inflation) from the GST Tax.”-691

3. Express Conditions of SurvivalMatter of Krooss—Ct. of App. of N.Y., 1951

Issue: Whether the Appellate Division erred in holding that each of the remaindermen or their estates, in order to recover, must survive the life beneficiary? YESRule: “The law has long favored a construction of language in deed and will that accomplishes the vesting of estates; such a result is preferred because, among other things, it enables property to be freely transferred at the earliest possible date.”-692“Accordingly, the courts are intent upon restricting defeating events to the exact circumstances specified.”-692“When a will contains language that has acquired, through judicial decision, a definite and established significance, the testator is taken to have employed that language in that sense and with that meaning in mind.”-693“Over the years, the courts have uniformly held that language such as that used by the testator here, or language substantially identical, creates a vested remainder in fee subject to be divested by the remainderman’s failing to survive the life beneficiary, if, but only if, such remainderman leaves issue or descendants surviving.”-693

Browning v. Sacrison—S. Ct. of Or., 1974Issue: Whether “the remainder devised to plaintiff’s husband, Franklin Browning, now deceased, and his brother, Robert Sacrison, the defendant, was vested or contingent at the time of Mrs. Webb’s death”? Contingent.Rule: “In modern law it is felt that when a devise is made to a life tenant with a remainder conditioned upon an ambiguous form of survivorship, the intent ‘commonly prevalent among conveyors similarly situated’ is deemed to require that the remainderman survive the life tenant rather than the testator.”-698

C. Construction of Class Gifts1. “Oscar might want to make a class gift – a gift to all members of a

designated group – rather than making separate gifts to individually named members of the group. And he might want to designate multi-generational class (issue of my brothers and sisters, rather than nieces and nephews).”-706

2. Increase in Class Membershipa. “By using the word ‘grandchildren’ rather than naming individual

grandchildren, testator expressed an intention that the devise not be limited to grandchildren living at the time of will execution. Moreover, no good practical reason exists for excluding grandchildren born by the time of testator’s death.”-707

b. “[G]randchildren born after testator’s death . . . will not be entitled to share in the $20,000. . . . When testator makes a devise of $20,000 to her grandchildren, she intends that her grandchildren will receive money at that time – not years later when it is no longer physiologically possible for additional grandchildren to be born.”-707

c. “[W]hen a grantor makes a class gift, membership in the class may continue to increase until at least one member of the class becomes entitled to possession of the property that is the subject of the class gift. Put another way, the class ‘closes’ when one member of the class becomes entitled to take her share of the property. . . . This rule is often called the ‘class-closing rule’ or the ‘rule of convenience.’”-707

d. The common law recognizes one significant exception to the class-closing rule: if, at the time an interest is intended to become possessory, no member of the class has yet been born, then the class

101

Page 102: Wills, Trusts

closing rule will not apply, and we wait for the class to close naturally.”-708

In re Evans’ Estate—S. Ct. of Wis., 1957Issue: Whether “or not the judgment entered April 17, 1942, is res judicata as to grandchildren born after the date of death of the testator”? NOHolding: “[T]he final judgment in the original probate proceedings was not binding on the afterborn grandchildren, because they were not represented therein by either a guardian ad litem or the existing members of the class (grandchildren already born who had antagonistic interests); hence the defense of res judicata was without merit.”-710Rule: “The maximum membership in the class is determined when the time for distribution has arrived. The class may increase until that time, and persons born thereafter are excluded.”-710“The time for distribution arrived when the first member of the class attained the designated age.”-710“When a class gift is postponed until the occurrence of some event, such as the attainment of age twenty-one, the class does not normally close until the first member of the class attains the designated age.”-710“The gift may be so made that the class may either increase or decrease after the death of the testator until the arrival of the fixed point of time or the happening of the specified event. It may be so worded that there may be an interval of time during which the class may increase but during which it cannot decrease.”-710

e. Notes(i) “[T]ypically, . . . a devise to a person ‘to be paid upon reaching

the age of 30’ or ‘when she reaches the age of 30’ is not construed to require survivorship until the age of 30. Instead, the interest is construed as a vested interest, and if the devisee does not reach the prescribed age, the devise is payable to the devisee’s estate.”-711

(ii) “The class-closing rule applied in the Evans case is a rule of construction. If, in a particular case, the grantor manifests an intent that the class close earlier – or not until the class closes physiologically – the intent of the grantor will control.”-711

(iii) Sherrod v. Cooper – “In a proceeding to construe the will, the court embraced the proposition, derived from a 1901 case, that with gifts of real estate, if there is no intervening life estate, the class closes at testator’s death unless there is clear language to the contrary. The court concluded that the language of the will was not sufficiently clear to establish that the class should remain open until the oldest granddaughter reached age 30.”-712

3. Decrease in Class Membership: Survivorship Againa. Notes

(i) “A lawyer drafting a will or trust instrument should never draft a class gift to ‘children’ without explanation or clarification.”-713

In re Trust of Walker—S. Ct. of Wis., 1962Issue: Whether “the bequest of the trust corpus to the grandnephews and grandnieces is conditioned upon their survival until the date of distribution”? YESHolding: “[W]e determine that it was testatrix’ intent that survival of a member of the class to the date of distribution is a requisite condition to sharing in such distribution.”-717Rule: “If testator has made a gift to a class, and one of the class dies before testator, or even after testator, while the class is not yet determined there is no lapse or failure of any part of such gift; but, in the absence of statute, no interest under such gift passes to the heirs or next of kin of the deceased

102

Page 103: Wills, Trusts

beneficiary. The entire gift passes to the members of the class who were in existence at the time that the class was determined.”-715

b. Notes on Express Survivorship Conditions(i) “One way to make it clear that testator intends to require

survival is to include express language to that effect.”-7194. UPC § 2-707: A Reprise

a. “UPC section 2-707 . . . revolutionize[s] the law of future interest by creating an implied survivorship requirement in all future interests. . . . In its application to class gifts, the statute imposes a survivorship requirement, but then creates, in section 2-707(b)(2), a ‘substitute gift’ in the surviving descendants of deceased class members.”-721

b. “In addition, the statute provides that ‘words of survivorship attached to a future interest are not, in the absence of additional evidence, a sufficient indication of an intent contrary to application of this section.’”-722

5. Adopted Members of the Classa. “It is well established in most jurisdictions today that a will or trust

provision leaving a future interest to a person’s children or grandchildren includes that person’s adopted children or grandchildren. Even when the gift is to ‘issue,’ which, perhaps, has a more biological connotation even than children, courts typically hold that adopted members of the class are entitled to take on an equal footing with natural-born class members.”-722

b. “In more recent years, a number of courts have permitted inheritance from and through natural parents after an adoption by the spouse of one of the natural parents.”-723

Newman v. Wells Fargo Bank—S. Ct. of Cal., 1996Issue: Whether, “in looking to the law of intestacy as a guide to a testator’s presumed intent when a will provision is ambiguous, a court should consider the law in effect at the time the will or testamentary trust was executed to determine if a child adopted out of a designated ancestor’s family is among ‘issue’ and ‘children’ the testator intended to benefit, or should apply the law in effect at the death of the ancestor through whom the child may take”?Rule: “A testator is presumed to be aware of the public policy reflected in the statutory definitions of the terms used in a will at the time the will is executed and to intend that those definitions be followed in construction of the will unless a contrary intent is expressed in the will. This presumption is strongest when an attorney has drafted the will because ‘where an instrument is drawn by one skilled in the law, the presence of legal and technical terms is an indication that the legal term of art has been used, and therefore is to be accepted, in accordance with its legal definition.”-726“[T]he law to which the court must look to find the presumed intent of a testator with regard to adopted children is the law in effect when a will is executed.”-727

c. Notes on Adult Adoptions(i) “In most states, adoption is not restricted to minor children;

adults may adopt other adults.”-730(ii) Davis v. Nielson – “Nielson adopted six people – including his

secretary, his secretary’s adult son, a nephew, and some acquaintances. Several of the adoptees were older than Nielson. When the trustee refused to distribute the trust principal – valued at between $1,000,000 and $2,500,000 – to the adopted ‘children,’ litigation resulted. The court held that the adult adoptees were not entitled to inherit, citing an article by Professor Rein: ‘Common sense tells us that a donor would

103

Page 104: Wills, Trusts

normally expect anyone partaking of his bounty to be a true family member and not just some willing adult adopted for the purpose of reducing or defeating a gift-over to others.’”-730

(iii) “Courts are more likely to permit adult adoptees to inherit when the relationship between the ‘parent’ and the ‘child’ appears to track a traditional parent-child relationship, as when a person adopts an adult stepchild.”-730

6. Gifts to “Heirs”a. “When settlor makes a future gift to ‘heirs,’ settlor leaves open an

important question: as of what date should the heirs be determined – the date of the testator’s death, or the date upon which the interest in heirs should become possessory?”-731

Harris Trust and Savings Bank v. Beach—S. Ct. of Ill., 1987Issue: Whether “the settlor intended that his heirs be ascertained at his death, or whether he desired that they be determined after the death of his wife, who was the life tenant”? WifeHolding: “We hold that because the primary reason for early vesting is no longer as important as it formerly was, proof by the preponderance of the evidence that the settlor, testator, or donor intended to use the term ‘heirs’ in its nontechnical sense is sufficient to delay the vesting of a gift to a time other than at the grantor’s death.”-733“[W]e conclude that the preponderant proof favors the position of the grandchildren and great-grandchildren and Hixon’s heirs should therefore be determined at Alice’s death.”-734“We conclude that the remainder in the heirs should be distributed per stirpes with the three great-grandchildren each taking one-ninth of the estate and the two grandchildren each taking one-third.”-737Rule: “The word ‘heirs’ refers to ‘those persons appointed by the law to inherit an estate in case of intestacy.’ . . . When used in its technical sense, the testator’s or settlor’s heirs are, of course, determined at the time of his or her death. . . . We have observed that ‘’heirs’ when used in a will does not necessarily have a fixed meaning. It may mean children or, where there are no children, it may mean some other class of heirs . . . if the context of the entire will plainly shows such to have been the intention of the testator.’ . . . A determination of the class of heirs, therefore, is governed by the settlor’s or testator’s intention rather than by a fixed rule of law. The rule in Illinois, however, has been that, unless the settlor’s intention to the contrary is ‘plainly shown’ in the trust document, courts will rely upon the technical meaning of the term ‘heirs’ by applying it as a rule of construction.”-733“If a gift to a class is vested, it is considered a true legal estate once the trust is executed. On the other hand, if the gift is a contingent, it ‘is not an estate, but merely the possibility of acquiring one.’”-734

b. Notes on the Common Law Rule(i) “[A]t common law, courts generally held that a future interest

in A’s ‘heirs’ vested at A’s death.”-737c. Notes on Tax Consequences

(i) “If a future interest is held by heirs determined at the time of testator’s death, those heirs are likely to hold a vested remainder which will be a ‘transmissible remainder’ for federal estate tax purposes. . . . [T]he transmissible remainder will be taxed and probated in the heir’s estate if the heir dies before the interest becomes possessory. By contrast, if the determination of heirs is postponed until the death of the income beneficiary, an heir who dies in the meantime will not hold an interest subject to estate tax.”-738

(ii) “[A] number of states (and the drafters of the UPCC) have promulgated statutes codifying the result in the Harris Trust case.”-738

104

Page 105: Wills, Trusts

(iii) UPC § 2-711 provides: “If an applicable statute or a governing instrument calls for a present or future distribution to or creates a present or future interest in a designated individual’s ‘heirs,’ ‘heirs at law,’ or ‘relatives,’ or ‘family,’ or language of similar import, the property passes to those persons, including the state under Section 2-105, and in such shares as would succeed to the designated individual’s domicile if the designated individual died when the disposition is to take effect in possession or enjoyment. If the designated individual’s surviving spouse is living but is remarried at the time the disposition is to take effect in possession or enjoyment, the surviving spouse is not an heir of the designated individual.”-738

d. Notes on the “Divide and Pay Over” Rule(i) “The court in Harris Trust purports to rely on the ‘divide and

pay over rule’ to support its construction of the gift to heirs as contingent on survival to the time of distribution. The rule has been applied not only to gifts to heirs, but to all sorts of future interests. Unfortunately, however, the rule is of little help. The rule was first articulated in Matter of Crane: ‘Where the only words of gift are found in the direction to divide or pay over at a future time the gift is future, not immediate; contingent and not vested.’ So articulated, the rule simply adopts the preference for construing class gifts to be contingent on survival to the time of distribution. But, as the court in Harris Trust also notes, the rule is said not to apply when distribution is postponed only for the purpose of creating an intervening estate.”-739

e. Notes on the Doctrine of Worthier Title(i) “In the Harris Trust case, the charities sought to invoke the

Doctrine of Worthier Title to support their position. At common law, when a grantor made a transfer in the form ‘to A for life, remainder to my heirs,’ courts often construed the transfer not to create a remainder in grantor’s heirs but instead to create a reversion in grantor. . . . In the words of the New York Court of Appeals in In re Burchell’s Estate: ‘The doctrine had its origin in the feudal custom of awarding certain valuable incidents to the overlord upon the descent of property held by a feoffee. These incidents did not accrue if the property was acquired through purchase, and, in order to obviate this means of curtailing that payments of incidents, title by descent was declared to be more worthy than title by purchase. If a gift over might pass to an heir by descent rather than by gift, he took his title through inheritance.’”-739

(ii) “In Doctor v. Hughes, . . . the New York Court of Appeals, in an opinion by Judge Cardozo, transformed the doctrine from a rule of law into a rule of construction – the doctrine would now be used as an aid in ascertaining grantor’s intent, nothing more.”-739

7. Class Gifts of IncomeDewire v. Haveles—S. Jud. Ct. of Mass., 1989

105

Page 106: Wills, Trusts

Issue: Whether “the class gift of income to grandchildren calls for the payment of income equally to those grandchildren living from time to time (as joint tenants with rights of survivorship) or whether the issue of any deceased grandchild succeeds by right of representation to his income interest”?Holding: “Where, among other things, every other provision in the will concerning the distribution of trust income and principal (after the death of the testator and his wife) points to equal treatment of the testator’s issue per stirpes, there is a sufficient contrary intent shown to overcome the rule of construction that the class gift of income to grandchildren is given to them as joint tenants with the right of survivorship.”-743Rule: “The general rule is that, in the absence of a contrary intent expressed in the will or a controlling statute stating otherwise, members of a class are joint tenants with rights of survivorship.”-741“[A] provision void for remoteness is still to be resorted to for construing the rest of the will.”-742

XXII. Estate and Trust AdministrationA. Introduction

1. “Oversimplifying, we are concerned with two kinds of fiduciaries: the people in charge of probate estates (personal representatives, or administrators or executors) and the people in charge of trusts (trustees).”-896

2. “In a sentence, personal representatives wind up a decedent’s affairs and trustees administer ongoing trusts. Executors are liquidators and trustees manage and dispense portolios.”-896

B. Estate Administration1. “In a sentence, an executor or an administrator has to collect (or marshal)

the decedent’s assets, pay his bills and taxes, and deliver the net estate to the devisees or heirs.”-896

2. Why Do We Administer Grandpa’s Estate?a. “There are two basic answers [to the question of why we have to

probate wills]: to start the statute of limitations running on the decedent’s debts and be sure that all of his property can be transferred to his successors.”-897

b. Nev. Rev. Stat. Ann. Sec. 147.040 – “(1) All persons having claims, due or to become due, against the deceased must file their claims with the clerk of the court within 90 days after the mailing, for those required to be mailed, or 90 days after the first publication of the notice to creditors pursuant to NRS 155.020. (2) If a claim is not filed with the clerk within 90 days after the first publication or mailing of the notice, the claim is forever barred, but when it is made to appear, by the affidavit of the claimant or by other proof to the satisfaction of the court, that the claimant did not have notice as provided in NRS 155.020, the claim may be filed at any time before the filing of the final account.”-898

c. Notes(i) “Simply put, only the simplest estates can, or should, be wound

up without administration of some sort. The presence of real estate, or a meaningful number of current or potential creditors, strongly suggests that an administration be undertaken. Usually, administration must be undertaken to pursue claims of the estate.”-898

(ii) “Many states provide simplified provisions of one sort or another to wind up small estates expeditiously. The statutes vary meaningfully from state to state, and often the procedures are limited or not as simple as they appear.”

106

Page 107: Wills, Trusts

(iii) “Basically, when one is trying to transfer a decedent’s property one asks the various bureaucracies that control either the property or the title what they require to make the transfer. Each will have their own procedures. If the property is solely in the decedent’s name the gatekeeper may well require proof of administration or compliance with the local small estate statute. Virtually all will require proof of death.”-898

3. Where Do We Administer Grandpa’s Estate – Domiciliary and Ancillary Jurisdiction Over Probatea. Questions

(i) “[A] domiciliary administration [is] the winding up of the affairs of a domiciliary of the county, in the county.”-899

(ii) “Ancillary means subsidiary or supplementary.”-899b. Notes

(i) “An ancillary administration is conducted the same way as a regular administration. The fiduciary collects the local assets, pays the local taxes and creditors and then usually sends the assets to the domiciliary executor where the assets are treated like any other assets – first they are used to pay the decedent’s debts, administration expenses and taxes and then what’s left is distributed under the terms of the will. Although one hopes for coordination, the two administrations are separate in every way.”-899

(ii) “It is always possible that foreign assets can be obtained without an ancillary administration. You should always inquire of competent local counsel before hiring local counsel to begin a proceeding.”-900

(iii) “Some banks, etc., especially in retirement states and in cities that function as ‘little Switzerlands’ for out of state and foreign national investors, often don’t get overinvolved with red tape when a customer dies.”-900

(iv) “Collecting assets in foreign countries can often be byzantine. Often the law on the books differs from the law in action.”-900

(v) “Similar issues of jurisdiction affect trustees of trusts. The law of the trust’s situs controls legal questions about the administration of the trust.”-900

(vi) “A trust settlor can always pick the law affecting a trust. In the absence of an express choice the situs is determined by an analysis of factors including: domicile of the trustee, location of the trust corpus, and the like.”-900

4. Who Should Be, and Who Will Be, the Executor With Some Thoughts About Trusteesa. “A fiduciary has to be an institution with fiduciary powers under state

law or an individual who is an adult of sound mind. If a court is to appoint the fiduciary, the nominee must be subject to the court’s jurisdiction. The probate court always appoints a personal representative (or guardian or conservator), sometimes appoints a testamentary trustee and rarely appoints the original trustee of an inter vivos trust . . . .”

107

Page 108: Wills, Trusts

b. “An executor should be someone who is able to wind up a decedent’s affairs, or who is able to supervise others whom she hires to accomplish that windup.”-900

c. “A trustee should be someone who is able to invest assets, or who is able to supervise others who he hires, and, if he has discretion over how much, and when, beneficiaries get income or principal from the trust, then he should have compassion and wisdom as well.”-901

d. UPC § 3-203 provides: “(a) Whether the proceedings are formal or informal, persons who are not disqualified have priority for appointment in the following order: (1) the person with priority as determined by a probated will including a person nominated by a power conferred in a will; (2) the surviving spouse of the decedent who is a devisee of the decedent; (3) other devisees of the decedent; (4) the surviving spouse of the decedent; (5) other heirs of the decedent; (6) 45 days after the death of the decedent, any creditor. (b) An objection to an appointment can be made only in formal proceedings. In case of objection the priorities stated in (a) apply except that (1) if the estate appears to be more than adequate to meet exemptions and costs of administration but inadequate to discharge anticipated unsecured claims, the Court, on petition of creditors, may appoint any qualified person.”-901

e. Notes(i) “Most clients find their fiduciaries in their immediate circle of

family, friends and advisers or turn to the trust department of a financial institution. Compassion, intelligence, ability, comfort with financial issues, honesty, freedom from conflicts of interest, energy, empathy and good communication skills are some of the characteristics of a good fiduciary.”-902

(ii) “[C]ourts are more willing to remove capable and honest trustees than capable and honest executors.”-902

(iii) UPC § 3-611 provides: “(a) A person interested in the estate may petition for removal of a personal representative for cause at any time. (b) Cause for removal exists when removal would be in the best interests of the estate, or if it is shown that a personal representative or the person seeking his appointment intentionally misrepresented material facts in the proceedings leading to his appointment, or that the personal representative has disregarded an order of the Court, has become incapable of discharging the duties of his office, or has mismanaged the estate or failed to perform any duty pertaining to the office.”-902

(iv) “[M]any courts are slow to let unhappy trustees resign, and even when they do agree they will likely insist on a replacement. The best explanation seems to be that the beneficiary who’s hard to get along with is the very one who most needs a trustee and the fiduciary who would now resign should have thought of this earlier.”-903

5. How the Executor Probates the Willa. “After a person dies owning probate property, someone will likely

begin a proceeding in the appropriate court. That court might be called the probate court, or the surrogate’s court, or the orphan’s court,

108

Page 109: Wills, Trusts

or in some states it might be a court of general jurisdiction. If the decedent died with a will, that someone, usually referred to as the petitioner, will likely offer the will for probate. The petitioner is likely to be the executor nominated in the will.”-903

b. “Simplifying, the will is offered for probate, in a judicial proceeding, with notice to persons financially interested in the death of the decedent and the transfer of his property. They include the decedent’s creditors, devisees under the will and the persons who would be hurt financially if the will was admitted to probate (intestate successors or devisees under prior wills who would otherwise take the decedent’s property). After the will is admitted to probate, and the executor appointed, the executor administers the estate, in some states independently, and in other states under close judicial supervision. After the administration is complete the executor distributes the assets and accounts for his actions as fiduciary.”-903

c. “Simplifying, there are two kinds of probate. The first kind is informal, or common form, or ex parte, probate. Simplifying again, when the probate is ex parte, the will is tentatively admitted into probate when the proponent offers it. Notice and the passage of time are not conditions precedent to admitting the will in an ex parte probate. They come after the will is probated. Only after this tentative probate does the executor give notice to all interested parties. . . . If proper notice is given, and if no one with standing successfully objects, then after the passage of a specified period of time the decree admitting the will to probate becomes final. At that point, the court order is an order in rem and is good against the whole world as a declaration that the will is the last will and testament of the decedent and that the petitioner is the executor of the estate.”-903

d. “The second kind of probate is more formal. It may be labeled solemn form or inter partes probate (between the parties). . . . The proponent who offers the will for probate is a notional plaintiff. She petitions for admission of the will to probate and serves a copy of her petition on the persons who are financially better off if the will is denied probate. . . . The heirs are the notional defendants. They are required to show cause why the will should not be admitted to probate. If they don’t successfully oppose the probate then the will is admitted. Their failure to appear is similar to defaulting in a lawsuit.”-904

e. “Some states have only one kind of probate. Some states allow the proponent of the will to choose between the two forms. Choice is the UPC approach. Informal probates are provided for, beginning at section 3-301 and formal probates are dealt with beginning at section 3-401.”-904

f. “Under either form of probate, a day arrives, early in the administration of the typical estate, when the court determines that it has jurisdiction and that the instrument before it is the last will and testament of the decedent. . . . The executor is appointed and issued letters testamentary. They are, if you wish, the rough equivalent of a business license for the estate.”-904

6. Marshalling Estate Assets – Gathering Estate or Trust Propertya. “A fiduciary has to take possession of her trust or estate property

within a reasonable period of time. . . . When the administration is of a

109

Page 110: Wills, Trusts

decedent’s estate we say there is a duty to marshall the decedent’s assets. This includes not only physical security, but also retitling assets, and pursuing the decedent’s claims. The personal representative is personally liable for assets lost due to bad faith or imprudence in collecting them. At the same time the fiduciary is under a duty not to waste assets foolishly pursuing small claims or claims of dubious value.”-904

b. “Ultra-cautious personal representatives may insist on obtaining court permission to abandon claims.”-904

c. “In theory, the personal representative should review all inter vivos transfers made by the decedent to see if any are ineffective.”-904

d. “If the fiduciary is a trustee he has a duty to be sure he is receiving everything that belongs to the trust.”-905

7. Creditors’ Claimsa. Pre-Death Creditors

(i) “First, many states have complex, local procedures for dealing with creditors. . . . Second, in some estates there may be genuine problems identifying all of a decedent’s creditors. . . . Third, a wise fiduciary will be concerned about unfounded or unenforceable claims or claims that have been overstated or inflated. Fourth, in the interest of efficient and timely winding up of a decedent’s affairs and out of a populist dislike of creditors and a sentimental concern for widows and orphans a number of jurisdictions have short statutes of limitation that cancel the rights of creditors.”-905

(ii) “These statutes come in two basic forms. So-called nonclaim statutes quickly cancel creditor’s rights after probate, so long as the specific statutory procedures are followed. . . . If a creditor gets notice of the death and probate fails to put in a claim, she can lose her rights in a matter of a few months. . . . The second type of statute of limitations extinguishes claims not presented within a longer period of time from the decedent’s death, with no notice required.”-906

Tulsa Professional Collection Services, Inc. v. Pope—S. Ct., 1988Issue: Whether an Oklahoma statute setting a two-month time period within which creditors are required to file claims against an estate admitted to probate, (with the two-month period beginning after publication of notice advising creditors of the commencement of the proceedings), satisfies the Due Process Clause of the Fourteenth Amendment? NOWhether the State’s involvement is substantial enough to implicate the Due Process Clause of the Fourteenth Amendment? YESHolding: “Appellant’s claim . . . is properly considered a protected property interest.”-909“In sum, the substantial involvement of the probate court throughout the process leaves little doubt that the running of Oklahoma’s nonclaim statute is accompanied by sufficient government action to implicate the Due Process Clause.”-910“Nor can there be any doubt that the nonclaim statute may ‘adversely affect’ a protected property interest.”-910“We hold that Oklahoma’s nonclaim statute is not a self-executing statute of limitations. Rather, the statue operates in connection with Oklahoma’s probate proceedings to ‘adversely affect’ appellant’s property interest. Thus, if appellant’s identity as a creditor was known or ‘reasonably ascertainable,’ then the Due Process Clause requires that appellant be given ‘notice by mail or other means as certain to ensure actual notice.’”-911

110

Page 111: Wills, Trusts

Rule: “[S]tate action affecting property must generally be accompanied by notification of that action: ‘An elementary and fundamental requirement of due process in any proceeding which is to be accorded finality is notice reasonably calculated, under all the circumstances, to apprise interested parties of the pendency of the action and afford them an opportunity to present their objections.’”-908“[A]ctual notice is a minimum constitutional precondition to a proceeding which will adversely affect the liberty or property interests of any party, whether unlettered or well versed in commercial practice, if its name and address are reasonably ascertainable.”-908“Private use of state-sanctioned private remedies or procedures does not rise to the level of state action. . . . Nor is the State’s involvement in the mere running of a general statute of limitations generally sufficient to implicate due process. . . . But when private parties make use of state procedures with the overt, significant assistance of state officials, state action may be found.”-909

U.S. Trust Co. of Fla. Savings Bank v. Haig—D. Ct. of App. of Fla., 1997Issue: Whether the trial court erred in granting a motion to extend time for filing a claim against an estate where the claim was “contingent” upon the finding of a defect covered by a guaranty? YESHolding: “Because we find that Appellees were contingent claimants, they were not entitled to actual notice.”-913“Because Appellees’ claim was contingent, they were not entitled to actual notice of the claims period, and must have filed their claim within 90 days after publication notice. Since the claim was not filed in a timely manner, the trial court was without discretion to grant Appellee’s motion for an extension of the claims period.”-914Rule: “A contingent claim is one where the liability depends upon some future event, which may or may not happen, which renders it uncertain whether there ever will be a liability. Nonclaim statutes have been applied to claims arising from a breach of covenant in a deed; to guaranty, suretyship and endorsement contracts; obligations contained in a lease; breach of contract to convey land.”-913“Nor do we consider it unreasonable for the State to dispense with more certain notice to those beneficiaries whose interests are either conjectural or future or, although they could be discovered upon investigation, do not in due course of business come to the knowledge of the common trustee.”-913

b. Notes(i) “In today’s world, many ‘estates’ are settled by settling the

testator’s funded revocable trusts.”-915(ii) “People who don’t plan to pay their bills sometimes establish

so-called asset protection trusts in friendly jurisdictions, usually foreign. These arrangements rely on local laws and customs designed to eliminate, or severely reduce, the rights and advantages of creditors. They offer no legitimate federal income or estate tax advantage.”-915

c. Post-Death Creditors(i) “Without statutory relief, or contractual setting aside of the

default regime, a fiduciary is personally liable on his contracts, even if he signs in a fiduciary capacity. Without statutory relief, the only escape from personal liability is a contractual provision, in the fiduciary/creditor contract, excluding personal liability of the fiduciary. At that point, the contract creditor is limiting himself to a recovery against the estate or trust assets. All is not lost for the fiduciary in a silent contract, however. She still can be indemnified from estate or trust assets, to the extent any such assets remain, and to the extent she property entered into the contract.”-916

(ii) “The important point is this: the estate or trust is not an entity that provides limited liability for fiduciary and beneficiaries.”-916

111

Page 112: Wills, Trusts

(iii) “The rules are the same for tort liability, except there it’s against public policy to contractually limit the rights of one’s tort creditors, if they can even be identified before the injury. So, at common law there is no escaping personal liability for the fiduciary, in tort. All the fiduciary can do is buy casualty insurance.”-916

(iv) “The UPC, in sections 3-808 and 7-306, sets the old rule aside and provides ‘a personal representative is not individually liable on a contract properly entered into in his fiduciary capacity . . . unless he fails to reveal his representative capacity and identify the estate in the contract.’ As to torts, the section provides a ‘personal representative is individually liable for . . . torts committed in the course of administration of the estate only if he is personally at fault.’”-916

d. CERCLA(i) “CERCLA is the Comprehensive Environmental Response

Compensation and Liability Act of 1980. . . . Fiduciaries feared, not without reason, that they would become personally liable for a decedent’s pollution of Blackacre. Banks often insisted that estate property be inspected for pollution, at estate expense, before they would qualify as an executor or trustee.”-917

(ii) “Congress . . . provided for fiduciary relief from CERCLA and RCRA (The Resource Conservation and Recovery Act of 1976) in the Asset Conservation, Lender Liability and Deposit Insurance Protection Act of 1996. . . . The 1996 exemption specifically allows the following ‘safe harbor’ acts on the part of the fiduciary: (1) Undertaking or directing another person to undertake a clean-up. (2) Responding to an agency enforcement order. (3) Terminating the fiduciary relationship. (4) Incorporating special environmental provisions (including indemnification of the fiduciary) into the fiduciary agreement or enforcing such provisions. (5) Inspecting the contaminated property. (6) Providing financial or other advice to the settlor or beneficiaries. (7) Altering or renegotiating the terms of the fiduciary relationship. (8) Administering property which was contaminated before the fiduciary relationship began. (9) Declining to take any listed action.”-918

(iii) “If, however, a fiduciary continues to knowingly operate a polluting dump, as was the case in City of Phoenix, it will still be held liable.”-918

C. The Duty of Loyalty1. “The fiduciary has the duty to be loyal, to be reasonably prudent in all

things, to delegate appropriately, wisely and well, to account for her acts, to safeguard and earmark (or keep separate) fiduciary assets, to be watchful, to seek information and to deal impartially with beneficiaries.”-942

2. “The nature of the fiduciary relation requires the fiduciary to act only for the benefit of the real owner. The fiduciary may not deal with the property so as to benefit directly or indirectly. . . . An important element of

112

Page 113: Wills, Trusts

the duty of loyalty is a flat prohibition against any transactions involving self-dealing.”-943

3. “The simplest application of the prohibition against self-dealing comes in sales by the fiduciary of her own property to the trust, or the converse, the purchase of estate or trust property by the fiduciary.”-943

4. “[I]t is the duty of the fiduciary to use reasonable efforts to obtain the highest price possible.”-943

Matter of Kinzler—S. Ct., App. Div., N.Y., 1993Issue: Whether the Surrogate’s Court erred in awarding the objectant’s attorney $7000 for legal fees, in finding that the sale of the decedent’s residence to one of the beneficiaries, who was also a testamentary trustee, constituted a conflict of interest and self-dealing, and in holding the executor’s payment of advance legal fees, without prior court order, to be improper? NORule: “The general rule is that, where legal services have been rendered for the benefit of the estate as a whole, resulting in the enlargement of all the shares of all the estate beneficiaries, reasonable compensation should be granted from the funds of the estate.”-944“A person ‘standing in the relation of a fiduciary capacity, cannot deal with or purchase the property, in reference to which he holds that relation.”-944“An executor must at all times discharge his fiduciary duties so that all legatees are treated in like manner and without prejudice or discrimination.”-945

Matter of Estate of Rothko—Ct. of App. of N.Y., 1977Issue: Whether the trial court erred in holding that the executors had violated their fiduciary duties to the estate by selling all of the decedent’s 798 paintings for less than their estimated value, many of which being sold to a dealer with ties to one of the executors, who profited as a result of the sale? NORule: “The duty of loyalty imposed on the fiduciary prevents him from accepting employment from a third party who is entering into a business transaction with the trust . . . . While he [a trustee] is administering the trust he must refrain from placing himself in a position where his personal interest or that of a third person does or may conflict with the interest of the beneficiaries.”-948

5. Questionsa. UPC § 3-713 provides: “Any sale or encumbrance to the personal

representative, his spouse, agent or attorney, or any corporation or trust in which he has a substantial beneficial interest, or any transaction which is affected by a substantial conflict of interest on the part of the personal representative, is voidable by any person interested in the estate except one who has consented after fair disclosure, unless (1) the will or a contract entered into by the decedent expressly authorized the transaction; or (2) the transaction is approved by the Court after notice to interested persons.”-950

6. Notesa. “There are degrees of conflict of interest. A direct conflict is involved

where there is self-dealing – the fiduciary buying from or selling to the estate or trust. Here the courts have developed a ‘no further inquiry rule,’ making the transaction voidable even though the fiduciary may have acted fairly under the circumstances.”-951

b. “As the Rothko case illustrates, where self-dealing is not involved, there still may be a conflict of interest in a less direct sense. Here the rules are not as rigid. There is no flat prohibition, but the fiduciary has the burden of showing that the transaction was completely fair to the beneficiaries, despite the taint of possible conflict.”-951

c. “The duty of loyalty to the beneficiaries of an estate or trust applies directly to the situation where the fiduciary wants to obtain consent or release from the beneficiaries to a proposed action by the fiduciary,

113

Page 114: Wills, Trusts

such as a sale, an investment, approval of accounts, etc. . . . Sometimes the fiduciary also occupies a position as a beneficiary. If so, the fiduciary may not favor herself over other beneficiaries.”-952

d. “Rothko also illustrates the problem of ‘appreciation damages.’ If a fiduciary negligently causes a loss in the performance of his duties then he is charged (or surcharged) for that loss. Simply put, the question is what is the measure of damages when a fiduciary has breached a duty to the beneficiaries and in the process sold something at too low a price. The usual answer is the fiduciary is charged with the price he should have gotten for the item plus interest on the amount he didn’t get, and he is credited with what he did get at the sale plus whatever income came in before the imperfect sale. The appreciation after the sale is lost to the beneficiaries.”-952

e. “When a bank trust department is a fiduciary it may ‘buy’ services from other units of the bank. For example, if the estate needs a savings account why not buy it from the commercial side of the bank? . . . [T]his kind of self-dealing is often authorized by statute, regulation or the boilerplate in the will or trust.”-952

D. The Duty of Care1. “Common law courts invented a somewhat different character – the

‘prudent person’ – to serve as a model for fiduciary behavior. The behavior of a trustee or executor must measure up to the prudent person standard. That is, a trustee who avoids ordinary negligence nevertheless breaches her duty of care if she acts imprudently.”-953

2. The Duty of Care in GeneralAllard v. Pacific National Bank—S. Ct. of Wash., 1983

Issue: Whether the trial court erred in holding that the Ps’ claims are primarily equitable in nature, and that they were therefore not entitled to a jury trial? NOWhether the trial court erred in dismissing the action against Pacific Bank for breach of fiduciary duties as trustee of the Stone Trusts? YESHolding: “We agree with the Superior Court that plaintiffs’ claims are primarily equitable in nature and we affirm its decision that plaintiffs have no right to jury trial. We conclude, however, that Pacific Bank breached its fiduciary duties regarding management of the Stone trusts. We also find the Superior Court incorrectly awarded attorney fees and costs to Pacific Bank.”-953Rule: “Except where impossible, illegal, or where a change of circumstances occurs which would impair the purposes of the trust, the nature and extent of the duties and powers of a trustee are determined by the trust agreement.”-957“[T]he trustee’s fiduciary duty includes the responsibility to inform the beneficiaries fully of all facts which would aid them in protecting their interests. . . . That the settlor has created a trust and thus required the beneficiaries to enjoy their property interests indirectly does not imply the beneficiaries are to be kept in ignorance of the trust, the nature of the trust property, and the details of its administration.”-958“The trustee must inform beneficiaries . . . of all material facts in connection with a nonroutine transaction which significantly affects the trust estate and the interests of the beneficiaries prior to the transaction taking place.”-958“[A] trustee may determine the best possible price for trust property either by obtaining an independent appraisal of the property or by ‘testing the market’ to determine what a willing buyer would pay.”-959

3. Notesa. “According to Allard there is a duty to inform the beneficiary under

circumstances faced by the fiduciary and a duty to inform can also be found in UPC § 7-303.”-960

114

Page 115: Wills, Trusts

4. Delegation of Fiduciary ObligationsShriners Hospitals for Crippled Children v. Gardiner—S. Ct. of Ariz., 1987

Issue: Whether Mary Jane’s delegation of investment power to Charles was a breach of Mary Jane’s fiduciary duty? YESWhether Mary Jane’s delegation to Charles of investment power was the proximate cause of the loss of $317,234.36?Whether Robert can properly continue to act as successor trustee and as guardian and conservator for the predecessor trustee Mary Jane? Holding: “Together, the accounting and admissions establish that Charles was functioning as a surrogate trustee. Mary Jane was not exercising any control over the selection of investments. She clearly breached her duties to act prudently and to personally perform her duties as a trustee.”-962“A delegation of investment authority is unreasonable and therefore Mary Jane’s delegation is a breach of trust.”-963Rule: “In Arizona, a trustee has the duty to ‘observe the standard in dealing with the trust assets that would be observed by a prudent man dealing with the property of another.’ . . . If the trustee breaches that responsibility, he is personally liable for any resulting loss to the trust assets. . . . A trustee breaches the prudent man standard when he delegates responsibilities that he reasonably can be expected personally to perform.”-961“[A] trustee lacking investment experience must seek out expert advice. Although a trustee must seek out expert advice, ‘he is not ordinarily justified in relying on such advice, but must exercise his own judgment.’”-962“The standard of care required . . . is measured objectively.”-963“The trustee must be reasonable in her delegation.”-963“A trustee is not permitted to delegate his responsibilities as a co-trustee. . . . Certainly, then, a trustee is subject to liability when she improperly delegates her investment responsibility to an alternate trustee. . . . A trust document may allow a trustee to delegate powers ordinarily non-delegable.”-963

a. Note(i) “The Restatement (Third) of Trusts has a new section 171.

Comment (h) to that section provides: ‘The trustee is not required personally to perform all aspects of the trust’s investment activities. The qualities and qualifications for which trustees are properly selected for fiduciary roles, and the scope and complexity of the investment programs of some trusts, are so diverse that prescriptions for prudent behavior in the delegation of investment functions cannot be expressed in simple and precise legal rules. With professional advice as needed, the trustee personally must at least define the trust’s investment objectives. In addition, the trustee must personally either formulate or approve the trust’s investment strategies and programs. Even these limited generalizations are necessarily and desirably couched in terms that are less than self-defining. In other matters, the trustee must exercise care, skill, and caution in determining what investment responsibilities to delegate. Fiduciary prudence must then be exercised as well in the selection of an agent, in establishing the terms of the delegation, and in monitoring or supervising the agent’s performance in a manner appropriate to the circumstances and conditions of the delegation.’”-965

b. John H. Langbein, The Uniform Prudent Investor Act and the Future of Trust Investing

115

Page 116: Wills, Trusts

(i) “The rule against delegating investment functions was a branch of the general nondelegation rule of trust law. As formulated in the 1959 Restatement, the nondelegation rule places the trustee ‘under a duty to the beneficiary not to delegate to others the doing of acts which the trustee can reasonably be required personally to perform.’. . . The core notion is to protect the settlor’s reliance when the personality of the trustee is a vital component of the settlor’s intention.”-966

(ii) “The traditional nondelegation rule was, however, overbroad. Courts tended to read the requirement that the trustee not delegate ‘acts which the trustee can reasonably be required personally to perform’ as a prohibition on delegating any function that looked to be important.”-966

(iii) “The second Restatement was crystal clear about investing: ‘A trustee cannot properly delegate to another power to select investments.’”-966

(iv) “In recent decades a variety of special-purpose statutes reversed the nondelegation rule for investment and other specialized functions, the Uniform Trustees’ Powers Act in 1964, the Uniform Management of Institutional Funds Act in 1972, and ERISA, the federal pension reform law, in 1974. . . . The mutual fund industry responded by securing legislation that remains in force in most states expressly authorizing trustees to invest in mutual funds.”-967

(v) “The 1992 Restatement achieves a major reform of the nondelegation rule. Nominally, the Restatement leaves the general nondelegation principle intact, but effectively reduces it to a subrule of the duty of prudent administration and makes it easy to overcome. The new rule reads: ‘A trustee has a duty personally to perform the responsibilities of the trusteeship except as a prudent person might delegate those responsibilities to others.’”-967

(vi) “The Uniform Act follows the Restatement in crafting a delegation regime. Section 9(a) empowers the trustee to ‘delegate investment and management functions that a prudent trustee of comparable skills could properly delegate under the circumstances.’”-967

(vii) “The Act provides that the trustee who complies with these standards ‘is not liable . . . for the decisions or actions of the agent to whom the function was delegated.’ Instead, an aggrieved beneficiary must look exclusively to the agent, who ‘owes a duty to the trust to exercise reasonable care to comply with the terms of the delegation.’”-967

5. Portfolio Managementa. A Bit of History

(i) “Courts have historically assumed that the settlor wanted, first, to preserve the corpus of the trust intact for the ultimate remainderman, and second, to generate as much income as possible consistent with safety of the corpus.”

(ii) “To implement this preference for safety, courts articulated the prudent person rule set out in Harvard College v. Armory: ‘All

116

Page 117: Wills, Trusts

that can be required of a trustee to invest, is, that he shall conduct himself faithfully and exercise a sound discretion. He is to observe how men of prudence, discretion and intelligence manage their own affairs, not in regard to speculation, but in regard to the permanent disposition of their funds, considering the probable income, as well as the probable safety of the capital to be invested.’”-969

(iii) “In a number of states, the prudent person rule evolved into a flat prohibition against certain assets – sometimes including common stocks – deemed too speculative for trust investments. Statutes and cases created laundry lists – often very short lists – of permissible trust investments.”-969

(iv) “Modern Portfolio Theory rejects the notion that risky investments are imprudent investments. Instead, Modern Portfolio Theory emphasizes that any asset, if bought at the right price, and if properly placed in a portfolio, with an understanding of diversification and risk, can be a wise investment. . . . That is, investors should think in portfolio terms, not in terms of individual investments.”-970In re Estate of Janes—Ct. of App. of N.Y., 1997

Issue: Whether the trial court erred in holding that the petitioner had acted imprudently and should have divested the estate of a high concentration of Kodak stock by August 9, 1973? YESHolding: “In our view, a fiduciary’s duty of investment prudence in holding a concentration of one security may not be so rigidly limited.”-972Rule: “No precise formula exists for determining whether the prudent person standard has been violated in a particular situation; rather, the determination depends on an examination of the facts and circumstances of each case. . . . In undertaking this inquiry, the court should engage in ‘a balanced and perceptive analysis of [the fiduciary’s] consideration and action in light of the history of each individual investment, viewed at the time of its action or its omission to act.’ . . . And, while a court should not view each act or omission aided or enlightened by hindsight . . ., a court may, nevertheless, examine the fiduciary’s conduct over the entire course of the investment in determining whether it has acted prudently.”-973“The inquiry is simply whether, under all the facts and circumstances of the particular case, the fiduciary violated the prudent person standard in maintaining a concentration of a particular stock in the estate’s portfolio of investments.”-974“[T]he amount of the trust estate, the situation of the beneficiaries, the trend of prices and the cost of living, the prospect of inflation and deflation.’ . . . Other pertinent factors are the marketability of the investment and possible tax consequences. . . . The trustee must weigh all of these investment factors as they affect the principal objects of the testator’s or settlor’s bounty, as between income beneficiaries and remainder persons, including decisions regarding ‘whether to apportion the investments between high-yield or high growth securities.’”-974“[T]he various factors affecting the prudence of any particular investment must be considered in the light of the ‘circumstances of the trust itself rather than [merely] the integrity of the individual investment.’”-974“The record of any individual investment is not to be viewed exclusively, of course, as though it were in its own water-tight compartment, since to some extent individual investment decisions may properly be affected by considerations of the performance of the fund as an entity, as in the instance, for example, of individual security decisions based in part on considerations of diversification of the fund or of capital transactions to achieve sound tax planning for the fund as a whole. The focus of inquiry, however, is nonetheless on the individual security as such and factors relating to the entire portfolio are to be weighed only along with others in reviewing the prudence of the particular investment decisions.”-975

117

Page 118: Wills, Trusts

“What constitutes a reasonable time will vary from case to case and is not fixed or arbitrary. . . . The test remains ‘the diligence and prudence of prudent and intelligent [persons] in the management of their own affairs.’”-976“Where, as here, a fiduciary’s imprudence consists solely of negligent retention of assets it should have sold, the measure of damages is the value of the lost capital.”-976

In re Estate of Cooper—Ct. of App. of Wash., 1996Issue: Whether “the prudent investor rule limits the court’s consideration to the overall performance of the trust, or whether, instead, the court may consider the performance of specific assets in the trust”?Holding: “We hold the prudent investor rule focuses on the performance of the trustee, not the results of the trust. The trial court here then appropriately considered individual assets, and groups of assets, in finding that the trustee had improperly weighed trust assets in favor of himself, the income beneficiary.”-977“The trial court here properly applied the prudent investor standard, as set forth above, and its findings of fact on that issue are supported by substantial evidence.”-980“The court did not abuse its discretion in removing Mr. Cooper as trustee.”-981“The statement attributed to Joyce does not unambiguously renounce her right to prudent trust management. Her statement was not a waiver.”-981Rule: “All that can be required of a trustee to invest, is, that he shall conduct himself faithfully and exercise a sound discretion. He is to observe how men of prudence, discretion and intelligence manage their own affairs, not in regard to speculation, but in regard to the permanent disposition of their funds, considering the probable income, as well as the probable safety of the capital to be invested.”-980“A waiver is an intentional relinquishment of a known right.”-981

b. Notes(i) “One of the classic questions in this area of the law is what to

do about ‘inception’ or ‘retention’ assets – assets received at the beginning, or inception, of the fiduciary relationship.”-983

(ii) “The typical trustee must sell improper investments with reasonable diligence, unless the instrument expressly or impliedly authorizes retention. What’s reasonable depends on factors such as current and expected market prices, and tax considerations.”-983

(iii) Bank common trust funds “are in-house bank mutual funds available only to trusts and estates of which the bank is a fiduciary. Traditionally, they are thought to be for small trusts, although historically they often outperform trusts directly invested by the bank’s trust officers.”-984

6. Safeguarding Property – The Duty Not to Comminglea. “A fiduciary must take measures to secure property under her control. .

. . The fiduciary must use reasonable prudence in selecting a bank for deposit of the estate or trust funds. She must take reasonable care of tangible personal assets, such as farm equipment and motor vehicles; and she must insure buildings and equipment against loss by fire and other hazards if reasonably prudent persons insure under the same circumstances.”-987

b. “A difficult problem that often arises concerning security of the estate or trust is a tendency by nonprofessional personal representatives and trustees to merge estate or trust assets, for long or short intervals, with their own. The fiduciary is under a duty at all times to keep such property separate from his individual property and clearly identified.”-987

118

Page 119: Wills, Trusts

c. “[C]ash funds of an estate or trust must be deposited not in the fiduciary’s individual bank account but in a separate account identified as a fiduciary account.”-987

d. “Similarly, documents of title such as a stock certificate should reveal the existence of the fiduciary relation. This rule has a two-fold purpose. It serves not only to identify the estate or trust property as against individual creditors of the fiduciary, but also to prevent unauthorized transfer.”-988

119